Nothing Special   »   [go: up one dir, main page]

Surgery Made Easy V1 PDF

Download as pdf or txt
Download as pdf or txt
You are on page 1of 125

Surgery Made Easy

Version 1

١ ‫صفحة‬
‫‪Surgery Made Easy‬‬


‪ ‬‬

‫بسم اهلل الرحمن الرحيم‬


الشكر هلل أوالً ثم ملن قام بتجميع وتصحيح األسئلة من السنوات السابقة‬
‫األسئلة تحتمل الصح والخطأ

‫تم اإلعتماد على مصادر رئيسية ومقررة

‫للتصحيح او اإلستفسار على اإليميل


‪medsmle@gmail.com‬‬

‫بالتوفيق …

‫صفحة ‪٢‬‬
Surgery Made Easy

Surgery
Made Easy

Contents
Cell biology ٥
Fluid and electrolytes ٦
Shock ١٦
Hematology ١٩
Surgical infection and antibiotics ٢٤
Trauma and Critical care ٢٩
Burn ٤١
Postoperative and Surgical complication ٤٣
Nutrition/TPN ٤٦
Wound healing ٥٢
Pre-operative assessment, Anesthesia and pain management ٥٥
Plastics, Skin, and Soft Tissues ٦٣
Surgical oncology ٦٦
The breast ٦٧
Thyroid, Parathyroid and Adrenal glad and other glands ٦٩
Hernia ٧٣
Head and Neck ٧٨
Upper GIT and Lower GIT ٨٠
Oesophagus ٨٠
Stomach ٨٣
٣ ‫صفحة‬
Surgery Made Easy

Small intestine ٨٨
Appendix ٩٠
Colon ٩٢
Rectum ٩٦
Anus ٩٦
Gastrointestinal Bleeding and acute Abdomen ٩٨
Hepatobiliary ١٠٢
Liver ١٠٢
Gallbladder ١٠٣
Pancreas ١٠٨
Spleen ١١٠
Research, Ethics & Professionalism and patient safety ١١٣
Transplant ١١٩
Urology ١٢٢
Vascular ١٢٣

٤ ‫صفحة‬
Surgery Made Easy

Cell biology
1. Activated protein C working on?
A. Fibrinolysis /Inhibit thrombin
B. Activate protein S
C. Inhibit fibrinolysis

- Thrombin-TM complexes induce an anticoagulant state through activation of protein


C and enhancement of fibrinolysis. Schwartz
- Protein C and Protein S:
- Both synthesized in the liver and dependent on vitamin K.
- Protein C degrades Factors Va and VIIIa, and promotes fibrinolysis by inactivating
plasminogen-activator inhibitor 1.
- Protein S is a cofactor for protein C and enhances its activity.
- and Inhibit thrombin

٥ ‫صفحة‬
Surgery Made Easy

Fluid and electrolytes


1. Patient for OR for elective surgery you give him dextrose 5% where will be ?
A. interstitial
B. Intravascular
C. Third space
D. Intracellular

- TBW = ( ECF + ICF ( interstitial fluid + intravascular fluid )


- Plasma Protein Solution distributed only to the intravascular fluid
- Normal Saline distributed mainly interstitial fluid and intravascular
- Dextrose 5% distributed intracellularly

2. Patient on warfarin he develop abdominal pain and intraperitoneal collection CT


abdomen confirmed hematoma what is next ?
A. Vit K
B. FFP
C. percutaneous drainage
D. Exploratory laparotomy

- Patients with serious or life-threatening bleeding and a prolonged INR (eg, >2) should
have warfarin withheld and should receive vitamin K (10 mg) by slow intravenous
infusion, along with a rapid reversal agent. We suggest a 4-factor prothrombin
complex concentrate (PCC) rather than Fresh Frozen Plasma (FFP). uptodate
- When contrast-enhanced CT detects coagulopathy-associated active extravasation,
this is more frequently venous than arterial, usually not requiring surgery or
embolization. Treatment is mainly conservative and based on withholding of
anticoagulant medications. link
- Patients who develop retroperitoneal hematomas as a result of anticoagulation are
best managed by the restoration of circulating blood volume and correction of the
underlying coagulopathy. In rare circumstances, arteriography with embolization of a
bleeding artery or operative exploration is required to stop the bleeding.

3. Case scenario of pt on NGT for 7 days what is acid base suspected?


A. metabolic alkalosis
B. metabolic acidosis
C. compensated metabolic acidosis
D. metabolic alkalosis with respiratory compensated

- Metabolic alkalosis can be generated by a shift of hydrogen ions into the cells. This
most often occurs in patients with potassium deficits and hypokalemia. This may be
an important pathophysiologic mechanism in patients with metabolic alkalosis due to
vomiting or nasogastric suction.

4. case scenario of chronic ulcerative colitis has watery diarrhea for 14 days what
is acid base suspected

٦ ‫صفحة‬
Surgery Made Easy

A. metabolic alkalosis
B. metabolic acidosis
C. compensated metabolic acidosis
D. compensated metabolic alkalosis

- Intestinal inflammatory processes reduce the absorption of sodium, chloride and


calcium, while they increase potassium secretion. In addition, mild to severe
metabolic alkalosis may occur in IBD patients, mainly depending on the severity of
the disease and the part of the gastrointestinal tract being affected. PMC3959504
- but here chronic UC but history of watery diarrhea making metabolic acidosis
- compensated depend on lab

5. Case scenario of high output entreocutenous fistula what is acid base


suspected?
A. metabolic alkalosis
B. metabolic acidosis
C. compensated metabolic acidosis
D. compensated metabolic alkalosis

- Duodenal or pancreatic fistulas may require bicarbonate replacement because of the


development of metabolic acidosis.

6. patient chronic renal failure on dialysis complain of abdominal pain and vomiting
vital stable ECG picture peaked - T wav ? 2018
A. IV calcium gluconate
B. Dialysis
C. Dextrose with insulin
D. Kayexalate oral

- Hyperkalemia is the most common clinically significant electrolyte abnormality in


chronic renal failure
- If there ECG change ( first sign peaked T then QRs wider) give Calcium gluconate
to protect heart then D50% Insulin

7. Early sign of malignant hyperthermia is?


A. Sinus tachycardia
B. Increase end tidal CO2
C. Fever
D. Sweating

- Caused by a defect in calcium metabolism


- Calcium released from sarcoplasmic reticulum causes muscle excitation –
contraction syndrome (ryanodine receptor defect)
- Side effects: 1st sign is ↑ end-tidal CO2, then fever, tachycardia, rigidity, acidosis,
hyperkalemia, rhabdomyolysis
- Tx: dantrolene (10 mg/kg) inhibits Ca release and decouples excitation complex;
cooling blankets, HCO3, glucose, supportive care

٧ ‫صفحة‬
Surgery Made Easy

8. A 42-year-old man with marked ascites is being treated for hepatic


encephalopathy secondary to alcoholic cirrhosis. What is the most likely acid-
base abnormality found in this patient?
A. Normal anion-gap metabolic acidosis
B. Metabolic alkalosis
C. high Anion-gap metabolic acidosis
D. Respiratory alkalosis

- Metabolic acidosis :
- Increased anion gap—ketoacidosis, alcohol intoxication, lactic acidosis, renal failure,
toxin ingestion (salicylates, paraldehyde, ethylene glycol, methanol)
- Normal anion gap (hyperchloremic)—renal tubular acidosis, potassium-sparing
diuretics, hypoaldosteronism, diarrhea, biliary or pancreatic fluid losses, small bowel
fistulas, dilutional acidosis, carbonic anhydrase inhibitors, ureteral diversions

9. Trauma pt in the ICU with PH 7.29, Which of the following will aggravate his acid-
base abnormality?
A. 25% albumin
B. NS
C. RL
D. D5 1/2 normal saline

- Fluids of choice: normal saline (in case of metabolic alkalosis > 7.45) and Ringer’s
Lactate (in trauma or metabolic acidosis < 7.35 )

10. Maintenance fluid equation, 67 kg ?


A. =107ml/h

11. patient post LAR with abdominal drain on the 5th day post op the drain amount
is 20ml, most appropriate IVF?
A. LR
B. NS
C. D5 1/2 NS

- IV maintenance fluids after major adult gastrointestinal surgery


- During operation and 1st 24 hours, use LR
- After 24 hours, switch to D5 ½ NS with 20 mEq K+
- 5% dextrose will stimulate insulin release, resulting in amino acid uptake and
protein synthesis (also prevents protein catabolism)
- D5 ½ NS @ 125/h provides 150 g glucose per day (525 kcal/day). Absite
- In general, gastric losses should be replaced with D5 ½ NS with 20 mEq K and
pancreatic, biliary, largest intestine losses (e.g., diarrhea) and small intestine losses
should be replaced with Lactated Ringer (LR) solution. Washington

12.Most sensitive test for diagnosis acid base imbalance?


A. ABG

13.67-year-old male pt c/o RUQ pain, Jaundice, clay stool with high fever what's is
the initial test?
٨ ‫صفحة‬
Surgery Made Easy

A. Urine culture
B. blood culture
C. tumor marker
D. lactate level

- shock or sepsis

14.about corrected sodium, Glucose 520 Na 118 ?


A. 124.6
B. 124.4
C. 126.4

- (glucose - 100 ) X 1.6 = A


- A + Na = corrected NA
15.Signs persistent hypocalcemia Chovestek sign Ca Normal?
A. Hypomagnesemia

- Hypomagnesemia Symptoms are similar to those of calcium deficiency, including


hyperactive reflexes, muscle tremors, tetany, and positive Chvostek’s and
Trousseau’s signs

16. Case of perforated appendix presented with picture of sepsis he has


hypokalemia What is the case?
A. Intracellular shift
B. Intraluminal shift

- Hypokalemia is much more common than hyperkalemia in the surgical patient. It may
be caused by inadequate potassium intake; excessive renal potassium excretion;
potassium loss in pathologic GI secretions, such as with diarrhea, fistulas, vomiting,
or high nasogastric output; or intracellular shifts from metabolic alkalosis or insulin
therapy. Schwartz
- Hyperkalemia = Intraluminal shift

17. Pt known to have chronic renal failure, after massive blood transfusion. His K
was 6.7 what is the management?
A. Urgent Dialysis
B. IV Ca gluconat 


- 1st drug to give; membrane stabilizer for heart


- then Calcium gluconate (1st drug to give; membrane stabilizer for heart)
- then Sodium bicarbonate, insulin, Kayexalate, Lasix, Albuterol, Dialysis if refractory.
Fiser

18. Post breast cancer surgery 2 years ago presented with thirst, lethargy and other
hypercalcemia symptoms first to give ?
A. IV fluid
B. Calcium gluconate

٩ ‫صفحة‬
Surgery Made Easy

C. Calcitonin 


- Malignancy is the most common cause of hypercalcemia in hospitalized patients and


is due to increased bone resorption or decreased renal excretion. Bone destruction
occurs from bony metastasis as seen in breast or renal cell cancer but also can occur
in multiple myeloma.
- The treatment of hypercalcemia of malignancy should begin with saline volume
expansion, which will decrease renal reabsorption of calcium as the associated
volume deficit is corrected. Once an adequate volume status has been achieved, a
loop diuretic may be added. Unfortunately, these measures are only temporary, and
additional treatment is often necessary. A variety of drugs are available with varying
times of onset, durations of action, and side effects.Calcitonin is effective, inhibiting
bone resorption and increasing renal excretion of calcium. It acts quickly, within 2 to 4
hours, but its use is limited by the development of tachyphylaxis. Schwartz
- NO RL No Thiazides
- give NS and lasix

19. Following right hemicolectomy for cecal cancer. The patient had illeus and was
th
kept on NGT suction. On the 5 day the pt. was clinically dehydrated with the
following parameters: Ph: 7.56 Po2: 85 Pco2: 50 Na: 132 K: 3 Cl: 80

urine analysis: Na: 2 K: 5 Cl: 6, what do these values reflect?
A. Uncomplicated metabolic alkalosis
B. Respiratory acidosis with metabolic compensation
C. Combined metabolic and respiraty alkalosis
D. Metabolic alkalosis with respiratory compensation

20. Small bowel fistula what is the best fluid for replacement?
A. RL
B. NS

- FLUID RESUSCITATION (FOR SIGNIFICANT DEHYDRATION):


- Sweat loss (eg marathon runner) – Tx: normal saline
- Gastric fluid loss (eg gastric outlet obstruction) – Tx: normal saline
- Pancreas, biliary, or small bowel fluid loss – Tx: lactated ringers
- Large bowel (eg massive diarrhea) – Tx: lactated ringers. Fiser 


21. Pt with Enterocutaneous fistula what is the acid base balance?


A. Compensated Metabolic acidosis
B. Compensated metabolic alkalosis
C. Metabolic acidosis. 


- Cause of MA = Mnemonic—USEDCRAP:
- Ureterostomy, Small bowel fistulas, Extra chloride, Diarrhea, Carbonic anhydrase
inhibitors, Renal tubular acidosis, Adrenal insufficiency, Pancreatic fistulas.

22. Pt with hypocalcemia tried to be corrected with calcium but not corrected what
you will check? 2018
A. Mg

١٠ ‫صفحة‬
Surgery Made Easy

- Hypocalcemia may occur in conjunction with Mg2+ depletion, which simultaneously


impairs PTH secretion and function. Washington

23. Pt, with pancreatic fistula what is appropriate IVF?


A. RL
B. NS
C. 5% Dextrose 


- see Q 20

24. 72 yrs old woman underwent a subtotal gastrectomy for gastric ca. she was
previously healthy. The operation time was 4 hours. And the estimated blood
loss was 600ml. she received 2 liters of normal saline during the operation. In the
recovery room she produced 10 ml of urine output in the first postoperative hour.
Abd examination was normal. Bp: 120/70 HR: 85 RR: 17 temp: 36.7 WBC: 18.2
Hb: 9 which of the following is the appropriate initial step in management?
A. Furosemide
B. Blood transfusion
C. Re-exploration of abdominal
D. Rapid infusion of NS

- Once a volume deficit is diagnosed, prompt fluid replacement should be instituted,


usually with an isotonic crystalloid, depending on the measured serum electrolyte
values. Patients with cardiovascular signs of volume deficit should receive a bolus of
1 to 2 L of isotonic fluid followed by a continuous infusion. Close monitoring during
this period is imperative. Schwartz
- Oliguria
- 1st – make sure patient is volume loaded (CVP 11–15 mm Hg)
- 2nd – try diuretic trial → furosemide (Lasix)
- 3rd – dialysis if needed. Fiser

25. Five days after an eventful open chole. An asymptomatic middle aged woman is
found to have a serum sodium level of 150 what is the proper next step in
management?
A. Restriction of free water
B. Plasma filtration
C. Hemodialysis
D. Dieresis with furosemide

- Hypernatremia – usually from poor fluid intake; restlessness, irritability, seizure


- Correct with D5 water slowly to avoid brain swelling. Fiser 


26. 28 yrs old man underwent abd. Exploration for perforated appendicitis postop.
Days 5 he was having diffuse abd. Distension. Constipation, sluggish bowel
sound with high output in the NGT. which of the following is the most
appropriate initial step in management?
A. Re exploration of the abd.
B. Abd CT

١١ ‫صفحة‬
Surgery Made Easy

C. Remove of NGT
D. Check serum electrolyte level

- Dx Hypokalemia. In surgical patient, GI (diarrhea, gastric drainage: vomiting/


nasogastric tube), diuretics, and insulin administration.

27. Patient with signs of hypocalcemia, positive chvostek's sign, Lab works showed
normal calcium and low Mg, what is management?
A. Ca gluconate
B. Mg sulfate

- Dx Hypomagnesemia. For those with severe deficits (<1.0 mEq/L) or those who are
symptomatic, 1 to 2 g of magnesium sulfate may be administered IV over 15 minutes.

28. Flat T wave in ECG?


A. Hypokalemia
B. hypomagnesaemia

29. Patient with enterocutaneous jejunal fistula ,what is appropriate IVF?


A. 0.9 normal saline
B. Ringer lactate
C. 5% dextrose

- See Q 20

30. Patient in ICU with pancreatitis has generalized edema with hypotension need
large volume of fluids, likely cause?
A. Third space loss
B. Vascular redistribution
C. Major intravascular loss

- symptom pancreatitis :Fever, tachycardia, possible hypotension secondary to fluid


sequestration (third spacing). mont

31. Patient with 10 days watery diarrhea brought dehydrated, RR 17 what is the
expected acid base balance?
A. Metabolic acidosis
B. Compensated metabolic acidosis
C. Compensated metabolic alkalosis
D. Metabolic alkalosis

- See Q 21

32. pt with pyloric hypertrophy , vomiting , Calculate K requirement in pyloric


stenosis... CL 88 mmole .. What is the expected K level ?
A. k 6-7
B. k 5 -6
C. k normal
D. k low (1.5-3.5)
١٢ ‫صفحة‬
Surgery Made Easy

33. NGT replace with wich type of fluids ?


A. NS
B. RL

34. Pt post APR with epidural catheter ,, he is sleepy with respiratory acidosis ( pco2
52) what is the next step ?
A. Intubation
B. IV naloxone
C. Remove the catheter

- first intubation then reverse narcotic by naloxone


- Causes of hypoventilation include respiratory center depression (a variety of causes
including narcotics), chronic obstructive pulmonary disease, pulmonary disease,
inadequate mechanical ventilation, and poor ventilation secondary to pain.
- Treatment of acute respiratory acidosis is directed at the underlying cause. Measures
to ensure adequate ventilation are also initiated. This may entail patient-initiated
volume expansion using noninvasive bilevel positive airway pressure or may require
endotracheal intubation to increase minute ventilation.
- Improve minute ventilation—remove airway obstruction, pulmonary toilet,
bronchodilators, avoid respiratory depressants, reverse opioid narcotics and
continuous positive airway pressure/bilevel positive airway pressure

35. Pt CU patient with blood sugar 600 and sodium 123... normal blood sugar is 100
corrected sodium is?
A. 131
B. 135
C. 137
D. 139

- Corrected Sodium = Measured Na + (((Serum glucose - 100)/100) x 1.6)

36. Post op hyponatremia , treatment ?


A. Restrict fluid intake

- Water restriction is first-line treatment for hyponatremia, then diuresis.


- Correct Na slowly to avoid central pontine myelinosis (no more than 1 mEq/h)

37. Duodenal obstruction or atresia acide – base balance abnormality?


A. Hypokalemic hypochloremic metabolic alkalosis

38. Pt with Hx of pancreatitis with Hyponatremia. what is the cause ?


A. pseudohyponatremia

- Hyperglycemia (eg DKA) can cause pseudohyponatremia – for each 100


increment of glucose over normal, add 2 points to the Na value
- Hyperlipidemia (eg from acute pancreatitis) can also cause pseudohyponatremia


١٣ ‫صفحة‬
Surgery Made Easy

39. ICU patient post laparotomy on nasogastric suction who is vitally stable with
following PH : 7.5 PCO2: 50 PO2: 80 S.Na: 131 S.K: 3.0 U.NA : 1 U.K: 5 U.cl: 5,
Dx?
A. Compensated metabolic alkalosis
B. Combined metabolic & respiratory alkalosis
C. Respiratory acidosis with metabolic compensation
D. Metabolic alkalosis with respiratory compensation

40. s\p thyroidectomy with hypo Ca signs... initial management will be?
A. Oral ca with vitamin D
B. vitamin D
C. IV Ca gluconate

- Intravenous management—appropriate in acute hypocalcemia


- This is not required in asymptomatic patient; 200–300 mg elemental Ca2+ is
required to eliminate attack of tetany.
- One gram calcium gluconate contains 2.2 mmol Ca2+.
- One gram CaCl2 contains 6.5 mmol Ca2+
- Oral management—appropriate in chronic hypocalcemia
- Calcium carbonate
- Phosphate-binding antacids improve GI absorption of Ca2+.
- Vitamin D

41. Patient presented with severe abdominal pain underwent CT with contrast. while
she is lying on CT table suddenly collapsed... the most likely cause is?
A. Peripheral vasoconstriction
B. Fluids redistribution

42. Critically ill ICU patient with … ? on amphotericin B you noticed persistently low
k despite boluses of 80meq KCL, the next step ?
A. Increase KCL replacement
B. Level of Mg

- Hypocalcemia may occur in conjunction with Mg2+ depletion, which simultaneously


impairs PTH secretion and function. Washington

43. Pt with crohn’s has diarrhea for 7 days. The most likely metabolic disorder is?
2018
A. Metabolic acidosis
B. Compensated metabolic acidosis
C. Metabolic alkalosis
D. Compensated metabolic alkalosis

- See Q21

44. Pt with severe head injury developed hyponatremia of 129. Next step?
A. Restrictions of fluid
B. Vasopressin
١٤ ‫صفحة‬
Surgery Made Easy

C. Hypertonic saline

- Hypertonic saline (7.5%) has been used as a treatment modality in patients with
closed head injuries. It has been shown to increase cerebral perfusion and decrease
intracranial pressure, thus decreasing brain edema. Schwartz

45. Patient with vomiting for many days has flattened T wave on ECG what is the
labs expexted? 2018
A. High urine potassium
B. Hyperkalemia
C. Hypochloremia

- Dx hypokalemia electrocardiographic changes: low voltage, flattened T waves, ST


segment depression, prolonged QT interval, and prominent U waves.

46. Patient with chronic gastric ulcer has pylori stenosis? What is the type of the IVF
for recucitation? 2018
A. NS with K
B. LR
C. 1/5 NS
D. D5 and 1/2 NS

- Hypochloremic and hypokalemic metabolic alkalosis can occur from isolated loss of
gastric contents in infants with pyloric stenosis or adults with duodenal ulcer disease.
Schwartz

47. Pt with gastric outlet obstruction What you will have?


A. Hypochloremic
B. Hyperkalemia

- Hypochloremic and hypokalemic metabolic alkalosis.

١٥ ‫صفحة‬
Surgery Made Easy

Shock
1. A critical care pt. with cardiac index 1.8, PVR 3000. Pulmonary wedge pressure
28, what is the most likely Dx?
A. Septic shock
B. Neurogenic shock
C. Cardiogenic shock

2. Pt with coronary artery disease, brought to hospital unresponsive, admitted in


ICU, he has increase CO (9) and CI, decrease SVR (600)?
A. septic shock
B. Cardiogenic shock
C. Hemorrhagic shock

3. Case scenario pt in ICU post operative CVP low , PWCP high , hypotensive , what
is the cause?
A. Septic shock
B. Cardiogenic shock
C. Hemorrhagic shock
D. Neurogenic shock

4. Case scenario of septic shock with it physiological change ?


A. Decrease SVR
B. Increase cardiac output
C. Systemic vasoconstriction
D. Hypoglycemia

- Distributive ("warm") shock:


- "Early sign septic shock"= Elevated CO + Decrease SVR = Low BP
- Cold shock:
- "Late sign septic shock"= Decrease CO + increase SVR = double Low BP

5. Pt. has been injured with deep mid thigh laceration, BP105/70 P 104 anxious,
how much the blood loss?
A. 5-10
B. 20-25
C. 25-30
D. 30-40 


- Shock in a trauma patient or postoperative patient should be presumed to be due to


hemorrhage until proven otherwise. The clinical signs of shock may be evidenced by
agitation, cool clammy extremities, tachycardia, weak or absent peripheral pulses,
and hypotension. Such apparent clinical shock results from at least 25% to 30% loss
of the blood volume. Schwartz

- grade II

- See table in Schwartz classification of hemorrhage.

١٦ ‫صفحة‬
Surgery Made Easy

6. 60 yrs old male underwent sigmoid resection for diverticular disease. Six hours
after the surgery, nurse informs you that the pt. is sweating, cold, and his urine
output in the last 4 hours has been 80 ml. temp: 36.8 HR: 120 RR 30 BP: 96/70
CVP: 3 cm what is the most likely reason for this shock?
A. Septic
B. Neurogenic
C. Hemorrhagic
D. Cardiogenic 


7. Pt with acute hemorrhage which one of the following finding will be found?
A. Decrease cardiac output
B. Vasodilatation of peripheral vessels
C. Increase respiratory rate
D. Alkalosis

- The most common cause of shock in the surgical or trauma patient is loss of
circulating volume from hemorrhage.
- see images

8. Compensatory mechanism in acute Hemorrhagic shock state include which of


the following ?
A. splanchnic vasodilation
B. tachypnea
C. bradycardia
D. decrease cardiac contractility

- Hemorrhage results in diminished venous return to the heart and decreased cardiac
output. This is compensated by increased cardiac heart rate and contractility, as well
as venous and arterial vasoconstriction. Stimulation of sympathetic fibers innervating
the heart leads to activation of -adrenergic receptors that increase heart rate and β 1
contractility in this attempt to increase cardiac output.
- Clinical signs of shock (hypotension, tachycardia, weak pulses, and/or cool, clammy
skin) occur after at least 25% blood loss

9. Which of the following is true in acute shock status?


A. low aldosterone
B. low cortisol
C. hypoglycemia
D. increase cardiac contractility

- See Q8 explanation .

10. a young victim of RTA, conscious and responsive with a chest contusion, he has
shallow breathing he also can’t move his lower limb and cant flex his elbow but
he’s able to shrug his shoulder and his is hypotensive, the cause of his
hypotensive state mostly related to?
A. high spinal injury
B. tension pneumothorax
C. Hemothorax
١٧ ‫صفحة‬
Surgery Made Easy

D. hemorrhagic shock

11. Post injury which of the following is decreasing?


A. Insulin
B. Aldosteron
C. Cortisol

- See table

12. Pt has trauma to lt thigh , he is pale , has active bleeding , best indicator to his
status is ?
A. Lactate level (< 2.5 )

- Several studies have demonstrated that the initial serum lactate and serial lactate
levels are reliable predictors of morbidity and mortality. Schwartz

13. A critical care patient has the following pulmonary artery catheter values,
cardiac index 5.0, systemic vascular resistance 500, and wedge pressure of 7,
this is most consistent with?
A. Septic shock
B. Hypovolemic shock
C. Cardiogenic shock
D. Neurogenic shock 


14. Post major abdominal surgery blood loss 600 cc, intra op she was hypotensive
received a lot of IVF + 2 liter PRBCs till she maintained her vitals after OR ,
shifted to ICU, UOP 1200/hr there, although she is well hydrated, creatinine &
urea levels increasing continuously for the next 5 days..hematocrit is normal ..
What is the most likely diagnosis?
A. sepsis
B. progressive bleeding
C. High output renal failure

15. Pt. with Acute appendicitis and hypotensive and tachycardia what the patient has
most likely?
A. Peripheral Vasodilatation
B. Decrease cardiac output

- Dx early septic shock = increase cardic output and vasodilation .

١٨ ‫صفحة‬
Surgery Made Easy

Hematology
1. Von willbrand disease best management before surgery?
A. Cryocieptate
B. FFP

- Types of VWD :
• Type I – reduced quantity of vWF "MC"
• Tx: recombinant VIII:vWF, DDAVP, cryoprecipitate
• Type II – defect in vWF molecule itself, vWF does not work well
• Tx: recombinant VIII:vWF, cryoprecipitate, DDAVP
• Type III – complete vWF deficiency (rare)
• Tx: recombinant VIII:vWF; cryoprecipitate(highest concentration of
vWF:VIII), DDAVP will not work for type III

2. Patient you prepare him for elective surgery on warfarin you should stop it ?
A. Before surgery 3 days
B. Before surgery 5 days
C. Continue warfarin
D. Before surgery 7 days

- If it has been determined that warfarin discontinuation is appropriate, we typically


discontinue warfarin five days before elective surgery (ie, last dose of warfarin is
given on day minus 6) and, when possible, check the PT/INR on the day before
surgery . If the INR is >1.5, we administer low dose oral vitamin K(eg, 1 to 2 mg) to
hasten normalization of the PT/INR and recheck the following day. We proceed with
surgery when the INR is ≤1.4. An INR in the normal range is especially important in
patients undergoing surgery associated with a high bleeding risk (eg, intracranial,
spinal, urologic) or if neuraxial anesthesia is to be used. uptodate
Puzzles
- Stop warfarin 4–5 days prior to surgery; if the patient has been treated for DVT,
pulmonary embolism (PE) or arterial embolism, start heparin
• Check INR: should be < 1.5
• Stop heparin 6 hours prior to surgery, then resume it 6 hrs after surgery without
bolusing
• Elective surgery should not be done within 1 month from the diagnosis of DVT/PE
- Okay to stop warfarin for patients with atrial fibrillation (A. fibrillation or A. fib.) without
starting heparin. Postoperatively, administer heparin SC until patient can eat; then
resume warfarin

3. Middle age women admitted for routine lap cholecystectomy gives no history of
increase bleeding tendency and have never investigated for bleeding disorder ,
what is the most appropriate preoperative evaluation for this pt ?
A. No screen test
B. Check for platelet count
C. Check for bleeding time
D. Check for clotting time and PTT

١٩ ‫صفحة‬
Surgery Made Easy

- If the history and physical examination do not suggest the presence of a bleeding
disorder, no additional laboratory testing is required
- If the patient or family history or physical examination suggests the presence of a
bleeding disorder, appropriate screening tests should be performed, including
prothrombin time (PT), activated partial thromboplastin time (aPTT), and platelet
count. uptodate

4. Post cholecystitis skin rash, ecchymosis, Hypotensive 80/40, PTT high, Low
fibrinogen, Which the first thing in management?
A. FFP.
B. Blood transfusion
C. ε-Aminocaproic acid

- most common postoperative bleeding is thrombocytopenia and DIC


- labs : prolonged PTT and PT both
- but DIC will be low fibrinogen and TTP normal fibrinogen
- Tx The most important facets of treatment are relieving the patient’s causative primary
medical or surgical problem and maintaining adequate perfusion. If there is active
bleeding, hemostatic factors should be replaced with FFP, which is usually sufficient to
correct the hypofibrinogenemia, although cryoprecipitate, fibrinogen concentrates, or
platelet concentrates may also be needed.

5. PT. post blood transfusion 10 units. What is most likely to be present ?


A. high potassium
B. high calcium
C. high platelet
D. thrombombocytopenia

- Dilutional thrombocytopenia and dilution of coagulation factors occurs with massive


transfusion
- Hypocalcemia – can cause poor clotting; occurs with massive transfusion; Ca is
required for the clotting cascade; hypocalcemia can also cause hypotension
- increase iron
- increase potassium

6. Pt in OR, they started blood transfusion and the pt become hypotensive what to
do?
A. Stop transfusion

- Dx Hemolytic reactions can be classified as either acute of delayed. Acute hemolytic


reactions occur with the administration of ABO-incompatible blood and can be fatal in
up to 6% of cases. Schwartz
- If the patient is awake, the most common symptoms of acute transfusion reactions
are pain at the site of transfusion, facial flushing, and back and chest pain.
Associated symptoms include fever, respiratory distress, hypotension, and
tachycardia. In anesthetized patients, diffuse bleeding and hypotension are the
hallmarks.Schwartz
- If an immediate hemolytic transfusion reaction is suspected, the transfusion should
be stopped immediately, and a sample of the recipient’s blood drawn and sent along
with the suspected unit to the blood bank for comparison with the pretransfusion
٢٠ ‫صفحة‬
Surgery Made Easy

samples. Urine output should be monitored and adequate hydration maintained to


prevent precipitation of hemoglobin within the tubules. Delayed hemolytic transfusion
reactions do not usually require specific intervention. Schwartz

7. They started blood transfusion on pt, he start to develop hypotension and the
urine dark what is the Dx?
A. Acute hemolytic reaction
B. Non hemolytic reaction
C. Allergic hemolytic reaction

- Acute hemolytic reaction occurs when recipient plasma contains antibodies to donor
RBCs because of ABO incompatibility (pRBCs, whole blood).
- Characterized by immediate fever, chills, dyspnea, back pain, bleeding, and shock
initially, with renal failure occurring later in course. Anesthetized patients may
experience only hypotension or bleeding.
- Treat with volume expansion, diuresis, and urine alkalinization after immediately
stopping transfusion. Reaction can occur after transfusion of only 10 mL; severity
increases with amount transfused.
- Hemoglobinemia (pink plasma) and hemoglobinuria ( red urine ) will occur within
minutes. Decreased haptoglobin indicates hemolysis. Direct agglutination test (Coombs
test) will be positive as long as residual incompatible RBCs persist in circulation.


8. 60 yrs old pt. required blood transfusion post operatively in the ICU. After
starting the transfusion his Bp drops to 70/40, with sinus tachycardia. His urine
is very dark. which of following is the most frequent cause of this condition?
A. Primary bacterial contamination of donor blood
B. Clerical errors in matching donor blood to recipient
C. Inadequate screening of donor blood for leucocytes
D. Recipient antibody formation to antigen on donor blood cells

- See Q 7 explanation.
- Others:
- Febrile nonhemolytic reaction occurs when recipient plasma contains antibodies to
leukocytes in donor unit (pRBCs, whole blood).
- Transfusion-related acute lung injury occurs when donor antibodies attack
recipient leukocytes, leading to immune complex deposition in pulmonary capillary
beds (any transfusion).

9. pt. post blood transfusion 8 units. What is most likely to be present ?


A. high potassium
B. high calcium
C. high platelet
D. alkalosis

- Come with transfusion high potassium and low calcium.

10. Boy with small leg laceration what is the most responsible of small vessel
spontaneous bleeding?
A. PLT plug
B. Fibrin
C. Prostacycline

٢١ ‫صفحة‬
Surgery Made Easy

- Prostacycline It inhibits platelet activation and is also an effective vasodilator.


- Thromboxane A2 it is a type of thromboxane that is produced by activated platelets
and has prothrombotic properties

11. 72 year old lady treated as acute cholecystitis, complain RHC mass with
hematemesis, HR 120, BP 80/50 T 40, Plt 60, high c- profile (PT) , fibrinogen 1.2.
What is your management?
A. Blood transfusion
B. FFP
C. Heparin
D. Amino carbonic acid

12. Young child with wound in the forearm stopped while going to hospital what is
the initial step for hemostasis?
A. Vasoconstriction
B. Pletlets aggregation
C. Myofibroblast
D. Prostacycline

- Three initial responses to vascular injury: vascular vasoconstriction, platelet


adhesion, and thrombin generation. Fiser

13. Fever post blood transfusion developed riggor and chills respond to
acetaminophen?
A. febrile Non hemolytic reaction
B. Allergic reaction

- Febrile nonhemolytic reaction occurs when recipient plasma contains antibodies to


leukocytes in donor unit (pRBCs, whole blood).
- The incidence of febrile reactions can be greatly reduced by the use of leukocyte-
reduced blood products. Pretreatment with acetaminophen reduces the severity of
the reaction. Schwartz
- Allergic reaction to donor components (any transfusion)
- It is characterized by itching, urticaria, wheezing, and angioedema.
- Treat with antihistamines, acetaminophen, and continue transfusion. Premedicate for
future transfusions and use washed, leukocyte-reduced products.
- Patients with history of immunoglobulin A (IgA) deficiency without history of
transfusion may have an anaphylactic reaction.


14. Pt start on blood transfusion then has back pain and bleeding from cannula ,,
what is the best way for Dx?
A. Direct antiglobulin (Coombs) test

- Dx hemolytic anemias. In anesthetized patients, transfusion reactions may present


as diffuse bleeding or hypotension
- Back pain, chills, tachycardia, fever, hemoglobinuria
- Hemoglobinemia (pink plasma) and hemoglobinuria will occur within minutes.
Decreased haptoglobin indicates hemolysis. Direct agglutination test (Coombs test)
will be positive as long as residual incompatible RBCs persist in circulation.
- Haptoglobin < 50 mg/dL (binds Hgb, then gets degraded), free hemoglobin > 5g/
dL, increase in unconjugated bilirubin. fiser

٢٢ ‫صفحة‬
Surgery Made Easy

15. Pt post transfusion arrested ?


A. Clerical error
B. hypothermia

- Hypothermia aggravates coagulopathy and provokes myocardial irritability.

16. post appendectomy, day 3 post op developed lower RT leg DVT started on
heparin, And he was on imipenem for treating the infection , day 7 post op the pt
developed low platelet and ecchymosis what is next? 2018
A. Hold heparin
B. Change abx
C. Transfusion of platelet.

- Dx Heparin-Induced Thrombocytopenia. Treat with immediate cessation of heparin


and initiation of alternate method of anticoagulation to prevent further thrombosis
(argatroban or lepirudin). Be cautious of heparin-bonded catheters or other sources of
heparin. mont

٢٣ ‫صفحة‬
Surgery Made Easy

Surgical infection and antibiotics


1. A 24 years old male HIV positive presented to ER with acute onset pain +
redness in his scrotum, penis and perineum, upon examination you feel
crepitus over erythematous area, with foul smelling gray discharge what is the
most appropriate management for this patient?
A. Pencilin G infusion
B. Surgical debridement
C. antiretroviral drugs
D. hydrocortisone

- Fournier gangrene 1s a soft tissue infection usually involving the scrotum. penis,
and perinewn.
- Treatment of necrotizing fasciitis consists of early aggressive surgical exploration
with debridement of necrotic tissue, broad spectrum antibiotic therapy, and
hemodynamic support as needed. Patients with Fournier's gangrene may ultimately
require cystostomy, colostomy, or orchiectomy.

2. Mechanism of action for meropenem?


A. inhibit bacterial cell wall synthesis
B. inhibit RNA

- Inhibits bacterial wall synthesis like other β-lactam antibiotics.


- Inhibitors of cell wall synthesis – penicillins, cephalosporins, carbapenems
(meropenem, imipenem), monobactams, vancomycin
- others:
- Produces oxygen radicals that breakup DNA – metronidazole (Flagyl)
- Inhibitors of the 30s ribosome and protein synthesis – tetracycline,
aminoglycosides (tobramycin, gentamicin), linezolid
- Inhibitors of the 50s ribosome and protein synthesis – erythromycin, clindamycin,
Synercid. Fiser

3. Patient post femoral popliteal bypass 8 months ago. Had infection that exposed
his graft. What is the most likely causative organism?
A. B hemolytic streptococcus.
B. staph aureus.
C. staph epidermis
D. pseudomonas.

- Graft infection rate – 1% (staph epidermidis#1; staph aureus, E. coli). Fiser


- Staphylococcus aureus – coagulase-positive = Most common organism overall in
surgical site infections
- Staphylococcus epidermidis – coagulase-negative

٢٤ ‫صفحة‬
Surgery Made Easy

4. Old pt dementic with co-morbidities he has infection below the umbilicus his son
told that the pt was scratching his skin before the redness appears, what is the
most causative organism?
A. B hemolytic streptococcus.
B. staph aureus.
C. staph epidermidis
D. pseudomonas.

6. Pt with acute cholecystitis what it the most common organism?


A. E coli

- Most common organisms in cholecystitis – E. coli (#1), Klebsiella, Enterococcus

7. PT received penicillin injection and then become tachycardia 150, with wheezing.
RR. 30. What is the initial management?
A. IV epinephrin
B. IV fluid
C. Intubation
D. Beta blocker

- Dx Anaphylactic shock. Mild anaphylaxis can be treated with 0.01 mg/kg (up to 0.5
mg) of 1 : 1000 (1 mg/mL, or 0.1%) intramuscular (midanterolateral thigh)
epinephrine and an oral or parenteral antihistamine. More severe cases are also
treated with steroids and may require oxygen, endotracheal intubation, IV
epinephrine infusion, bronchodilators, IV fluids, or vasopressors. These patients are
observed for approximately 24 hours in a monitored environment for any recurrence
of severe symptoms.

8. Pt. with Ventilator associated pneumonia, what is the best ABx?


A. Vancomyocin
B. Ceftraxion
C. Meropenum

- Empiric therapy
- Early and mild → cephalosporins (2nd, 3rd) or Fluoroquinolone for penicillin allergy
- Moderate with risk factors → above + vancomycin
- Severe or late → Fluoroquinolone with antipseudomonal 3rd-generation
cephalosporin or meropenem + vancomycin
- Duration of therapy Change antibiotic (down grade) according to culture result, then
continue for 1-2 weeks in total. puzzle

9. What is the empiric ABx for necrotizing fasciitis?


A. Tigacyclin
B. Meropenum
C. Ertapenem
D. Clindamycin 


٢٥ ‫صفحة‬
Surgery Made Easy

- Management of patients with suspected necrotizing infections should begin with


proper patient triage to an ICU for initial evaluation, resuscitation, and treatment. If
the diagnosis is clear, operative exploration and debridement should not be delayed.
Broad-spectrum IV antibiotics should be started as soon as possible, with
vancomycin (for MRSA) in addition to clindamycin or linezolid (to inhibit toxin
synthesis) and gram-negative rod coverage (in the form of a third-generation
cephalosporin or a quinolone). Surgery is the definitive treatment. Schwartz
- Triple therapy: e.g. Zosyn ®, Vancomycin, Clindamycin. surgery recall

10.A farmer bare foot, stepped on a nail or glass, he developed cellulites, with streak
lesion, what is the most likely organism ?
A. Staph aurous
B. Staph epidermidis
C. B hemolytic Strep pyogenes

- Cellulitis is characterized by a nonpurulent diffuse spreading of inflammation


characterized by erythema, warmth, pain, swelling, and induration. Skin breakdown is
a frequent cause, but often no inciting factor is identified. Group A β-hemolytic
Streptococcus is the most common offending pathogen and causes a more diffuse
spread of infection. S. aureus is the second most common offending pathogen and
will cause a more localized cellulitis. Schwartz
- Group A β-hemolytic Streptococcus = Streptococcus pyogenes.

11. Which of the following procedures require prophylactic antibiotic?


A. Appendectomy for perforated appendicitis
B. cholecystectomy of acalculus cholecystitis
C. Appendectomy for gangrenous appendicitis
D. Elective resection of sigmoid cancer

- Agents are selected according to their activity against microbes likely to be present at
3 the surgical site, based on knowledge of host microflora. For example, patients
undergoing elective colorectal surgery should receive antimicrobial prophylaxis
directed against skin flora, gram negative aerobes, and anaerobic bacteria. Schwartz

12. 30 yrs old pt developed high fever, diarrhea, vomiting and redness of the skin of
the entire body, and hypotension. 48 hours post right inguinal hernia repair. The
incision appeared unremarkable. He developed diffuse desquamation the
following day. What is the most likely causative organism?
A. Clostridium perfignes
B. Clostridium difficile
C. B hemolytic strep
D. Staph aureus

- Staphylococcal toxic shock syndrome (TSS) fever, vomiting, rash, desquamation,


shock, end-organ failure.
- Compare with Streptococcus pyogenes "B hemolytic" TSS (a toxic shock–like
syndrome associated with painful skin infection).
- MC infection post surgical procedure is S. aureus.

٢٦ ‫صفحة‬
Surgery Made Easy

13. Hand dorsal side swelling and erythema with axillary lymph node enlargement,
what IV antibiotics?
A. Cefazolin
B. Penicillin G

- Dx cellulitis. Penicillin G Mostly used for gram ⊕ organisms (S pneumoniae, S


pyogenes, Actinomyces).

14. post sigmoid surgery ,fever ,pelvic collection organism?


A. bacteroid

- Anaerobes (anaerobic bacteria)
- Most common organisms in the GI tract
- More common than aerobic bacteria in the colon (1,000:1)
- Need low-oxygen environment (lack superoxide dismutase and catalase, making
them vulnerable to oxygen radicals)
- Bacteroides fragilis – most common anaerobe in the colon
- Escherichia coli – most common aerobic bacteria in the colon

15. Pt diabetic with soft tissue infection, best antibiotic ?


A. Titracycline
B. Tazocin
C. Penicillin G

- Dx cellulitis.

16. A 21-year-old male is brought to the trauma bay after sustaining a superficial
stab wound to the left shoulder. He reports that he recelved his full series of
shots as a child and received his last tetanus booster shot when he was 15. What
should he receive for his tetanus prophylaxis?
A. Amoxicillin 500-mg PO TID
B. Nothing
C. Tetanus immune globulin (TIG) 250-units 1M
D. Tetanus toxoid (dT) 0.5-mL 1M

- TETANUS
- consider Non–tetanus-prone wounds.
- Non–tetanus-prone wounds – give tetanus toxoid only if patient has received < 3
doses or tetanus status is unknown, or > 10 years since booster
- Tetanus-prone wounds (> 6 hours old; obvious contamination and devitalized
tissue; crush, burn, frostbite, or missile injuries) – always give tetanus toxoid unless
patient has had ≥ 3 doses and it has been < 5 years since last booster
- Tetanus immune globulin (given intramuscular near wound site) – give only
withtetanus-prone wounds in patients who have not been immunized or if
immunization status is unknown.Fiser

17. Pt with aortobifomral bypass 18 months later presented with tenderness and
there is collection and graft is seen . Most likely organism?
A. Staph epidermidis
٢٧ ‫صفحة‬
Surgery Made Easy

B. Staph aur.
C. Klebsiella

- See Q3

18. Pt mentally ill diabetic with skin infection after scratching the non-clostridium
infection? 2018
A. Staph epidermis
B. Staph auras
C. Staph viridans
D. Group A β-hemolytic

- Read Q 10

٢٨ ‫صفحة‬
Surgery Made Easy

Trauma and Critical care


1. Patient with escape from burn what is the diagnostic sign of Inhalation injury?
A. Carbonaceous sputum
B. Hoarseness
C. Tachypnea
D. Singte nasal sputum

- Physical findings include burns to the face, singed nasal vibrissae, soot in the
oropharynx, nasal passages, proximal airways, and carbonaceous sputum

2. Case scenario of Trauma pt what is the hormone last one finish his?
A. Insulin
B. Cortisol
C. Aldosterone
D. ADH

- insulin will decrease and other chose will increase responding to trauma

3. Pt post MVA with blunt chest trauma , normal x ray, chest tenderness and
hypoxia not improve with O2 pt in ICU intubated ... CXR unilateral basal infiltrate
with decrease air entry on the Rt side and decrease breath sound ... what is the
cause of his deterioration?
A. Lung contusion
B. ARDS
C. Pneumothorax
D. Pneumonia

- Pulmonary contusion is another common consequence of blunt chest trauma.


Pulmonary contusions generally develop over the first 24 hours and resolve in about
one week. Irregular, nonlobular opacification of the pulmonary parenchyma on chest
radiograph is the diagnostic hallmark. About one-third of the time the contusion is not
evident on initial radiographs
- Common complications include pneumonia and acute respiratory distress syndrome
(ARDS)
- ARDS usually come bilateral
- pneumonia augmented by any chest wall injury and mechanical ventilation

4. A 22 years old man with GSW to left flank, CT showed large pulsatile central
retroperitoneal hematoma extended above the level of celiac artery, she is still
hypotensive despite IVF &blood transfusion approach?
A. Left thoracotomy
B. Pelvic embolization
C. Infra diaphragmatic control of aorta

5. Case scenario for male pt victim of gunshot to the thigh pt was pale ,
unconscious , BP 90/60 pulse 130 Spo2 85 % What is the next ?
A. Orotrachal intubation

٢٩ ‫صفحة‬
Surgery Made Easy

B. Transfusion for blood


C. Infusion for R/L
D. Shift the patient for OR

- ABCD

6. 25 year old women involved in RTA, underwent exploration, she received 20


units PRBCs. On the recovery area you noticed oozing from cannula site, the
most likely cause is?
A. Thrompcytopenia
B. Hypocalcemia
C. Cardiac temponad
D. VWD

- This term refers to a single transfusion of greater than 2500 or 5000 mL over a 24-h
period. A number of problems may accompany the use of massive transfusion,
including thrombocytopenia, impaired platelet function, deficiency in factors V, VIII,
and XI, and the increased acid load of stored blood products. With large transfusions,
a heater may be used to warm the blood, since hypothermia may result in decreased
cardiac output and an acidosis. schwartz

7. A 20 years old man, sustains a gunshot wound to the abdomen, his blood
pressure 110\70, HR 100 at surgery, he is found to have through & through injury
to splenic flexure , involving some of the bowel circumference, minimal fecal
contamination, no devasculrization, which of the following is the best option?
A. Primary repair
B. Resection of the involving part with anastomosis
C. Resection of the involving part with anastomosis & stoma
D. Extended left hemicolectomy

8. 28 years old female involved in GSW through & through to the right lobe of the
liver, she is unstable vitally, during exploration you found active bleeding that
cannot be controlled next step?
A. hepatic artery ligation
B. Perihepatic packing
C. right lobe resection

- In the 15% of patients for whom emergent laparotomy is mandated, the primary goal
is to arrest hemorrhage. Initial control of hemorrhage is best accomplished using
perihepatic packing and manual compression. With extensive injuries and major
hemorrhage a Pringle maneuver should be done immediately. Intermittent release of
the Pringle is helpful to attenuate hepatic cellular loss. In either case, the edges of the
liver laceration should be opposed for local pressure control of bleeding. Hemorrhage
from most major hepatic injuries can be controlled with effective perihepatic packing.

9. 29 YO man present with gunshot wound in the RUQ. On physical examination,


the patient has a tender abdomen. At the surgery found to have 500 ml
hemoperitoneum with through & through injury to the right lobe of the liver that
is no longer actively bleeding. Further management would consist of ?

٣٠ ‫صفحة‬
Surgery Made Easy

A. Closing the injury with liver suture


B. Packing injury with omentum
C. Application of fibrin sealant
D. No further management
E. Drainage with penrose drain

10.scenario about trauma pt in ICU, what is the most indicator for organs perfusion?
A. Urine output
B. lactic acid

- Urine output is a quantitative, reliable indicator of organ perfusion. Adequate urine


output is 0.5 mL/kg per hour in an adult, 1 mL/kg per hour in a child, and 2 mL/kg per
hour in an infant <1 year of age. Because measurement of this resuscitation related
variable is time dependent, it is generally more useful in the OR and intensive care
unit (ICU) setting, than in initial evaluation in the trauma bay. Schwartz
- Clinically, lactate levels may not be useful in directing resuscitation, although lactate
levels may be higher in nonsurvivors of serious injury. Schwartz

11.19 yrs old male pt. victim of RTA sustained blunt chest trauma. Physical
examination revealed absent breathing sound on the right side with hyper
resonant percussion note. BP: 80/40 HR: { 130 RR: 28 Temp: 36.7 which of the
following physiological effect is excepted ?
A. Increased venous return
B. Increased cardiac output
C. Increased intrathoracic pressure
D. increased ventilation

- pneumothorax.

12. Trauma patient with facial injury in respiratory distress and hematemesis. Vitals
unstable. What is your next step?
A. Cricothyrodotomy
B. Tracheostomy.

- Patients in whom attempts at intubation have failed or who are precluded from
intubation due to extensive facial injuries require operative establishment of an
airway. Cricothyroidotomy is performed through a generous vertical incision, with
sharp division of the subcutaneous tissues.
- In patients under the age of 11, cricothyroidotomy is relatively contraindicated due to
the risk of subglottic stenosis, and tracheostomy should be performed.
- Emergent tracheostomy is indicated in patients with laryngotracheal separation or
laryngeal fractures, in whom cricothyroidotomy may cause further damage or result in
complete loss of the airway. Schwartz

13.MVA sustained chest trauma with bilateral ribs fractures and chest tenderness,
x_ray showed bilateral lung infiltrations, he is desaturated. What is the underlying
cause of his desaturation?
A. Lung contusion
B. Atelectasis

٣١ ‫صفحة‬
Surgery Made Easy

C. ARDS

- Bilateral ribs ..

14.post trauma pt. Studies showed transected pancreatic duct at the neck of
pancreas,, what is the management?
A. Whipple
B. Put drain and close.
C. Distal pancreatectomy.

- Distal pancreatectomy has been useful for transections of the body of the pancreas,
but a Roux-en-Y anastomosis to both ends of the pancreas has been a preferred and
satisfactory method of management of the completely transected pancreas in the
region of the neck. Pancreaticoduodenectomy has only been performed for combined
pancreaticoduodenal injuries.
- Pancreas injury grades :
- Grades I and II—goal is hemostasis and adequate drainage.
- Grade III—distal pancreatectomy
- Grade IV—if duct injury is indeterminate on local exploration, consider
intraoperative pancreatic ductography either via needle into gallbladder or
duodenotomy and ampulla cannulation.
- (1) Stable patient—oversew salvageable portion of proximal pancreas stump,
Roux-en-Y anastomosis of distal pancreas to jejunal limb.
- (2) Unstable patient—get hemostasis, drain widely, get out. Postoperative
endoscopic retrograde cholangiopancreatography to define injury and possibly
place duct stent.
- Grade V—trauma Whipple; high morbidity and mortality rates

15.Pediatric 13 years old, post MVA, vitally stable, spleen laceration . CT showed
peri splenic fluid with blush of contrast, Your management?
A. Splenectomy
B. Partial splenectomy
C. Non-operative management.

- In pediatric patients, however, nonoperative management has become the preferred


means of splenic salvage. The identification of contrast extravasation as a risk factor
for failure of nonoperative management led to liberal use of angioembolization.
Schwartz
- Clinically stable children with isolated liver and spleen injuries, grades I through IV,
should receive nonoperative management provided by a pediatric surgeon. Children
with grade V or higher (liver) injury usually warrant surgical intervention although
children who are clinically stable can be managed non-operatively even with Grade V
injuries. uptodate

16.Patient involved in MVA with frontal head trauma, CT showed high density
crescent shaped material, what is the most likely diagnosis?
A. Epidural with right mid line shift. ( extradural )
B. Subdural with no mid line shift.
C. Subdural with Rt midline shift

٣٢ ‫صفحة‬
Surgery Made Easy

- Subdural hematoma On head CT scan, the clot is bright or mixed-density, crescent-


shaped (lunate), may have a less distinct border, and does not cross the midline due
to the presence of the falx. Schwartz
- Epidural push brain in CT so cross midline

17.Facial fracture with dropped mouth angle, which facial nerve branch?
A. Buccal
B. Marginal
C. Zygomatic
D. cervical

- marginal mandibular of facial nerve .

18.Patient with gunshot to the right chest, on examination revealed (low harsh
sound in inspiration), Your management?
A. Chest tube insertion.
B. RT thoracotomy
C. sterile dressing

19.Pt sustained closed fracture of tibia and fibula, c/o sever pain in the same limb
(suspicion of compartment syndrome) which is the best?
A. Physical examination
B. Angiogram
C. US

- While the diagnosis is based on clinical exam, pressures can be measured with
needles placed into the compartment, which is necessary in unconscious patients
who will not show these exam findings. When compartment syndrome is suspected,
emergent fasciotomy must be performed in which the overlying tight. Schwartz

20.Extraperitonel Urinary bladder injury?


A. foly’s catheter.
B. laparoscopic repair
C. Supra-pubic repair

- The majority of extraperitoneal bladder injuries can be treated nonoperatively, and


probably do not need Urology involvement. The bladder catheter is left in place 10-14
days (we do 10 days), and a repeat cystogram is obtained. If there is no leak, the
catheter can be removed. If there is still some leakage, Urology consultation should
then be obtained. Schwartz
- Intraperitoneal bladder injuries should be explored immediately and repaired.
Schwartz

21. Post intubation end tidal CO2= 0 what is the cause?


A. esophagus intubation
B. High intrathoracic pressure

22. LAR procedure c/o abdominal pain US showed 8*8 com collection?
٣٣ ‫صفحة‬
Surgery Made Easy

A. Drainage
B. Re exploration with taking down the anastomosis
C. Hartmann 


23. RTA pt, with head injury she was conscious, and she become suddenly
unconscious again (lucid interval) what is the Dx?
A. Extradural hematoma
B. Subdural
C. Subarchnodi
D. Intracerebular 


- Epidural hematoma " Extradural " – caused by injury to middle meningeal artery


- Has lens shape on head CT and pushes brain away
- Patients classically lose consciousness, have a lucid interval, and then lose
consciousness again.
- Tx: operate for significant neurologic degeneration or mass effect (shift > 0.5 cm)

24. Trauma stab wound to the right side of the sternum with fainting heart sounds
and pulses paradoxes and absent air entry on left side? 2018
A. Chest decompression
B. Paracentesis
C. Ehco fast
D. CXR

- Dx Tension pneumothorax is a clinical diagnosis and an immediate threat to life.


- Diagnose by tracheal deviation, distended neck veins, diminished lung sounds, and
hemodynamic instability.
- Perform immediate needle decompression as a temporizing measure prior to emergent
chest tube placement. Fiser

25. Deep stab wound in the arm, patient can't flex his thumb. What is the never
injured?
A. Median
B. Radial
C. Ulnar
D. Musculocatnous 


- Lack of ability to abduct and oppose the thumb due to paralysis of the thenar muscles.
This is called "ape-hand deformity".

26. Pt with stab wound below the coricoed cartilage, he has subcutaneous
emphysema, and vitally stable, HR: 90, BP: 120/90, what is next ?
A. Exploration
B. Ct angio for the neck
C. Chest x ray
D. Intubation

27. 22 yrs old woman is brought to ER following an alte..... she has 3 cm stab wound
to the left sided of the neck above the cricoids cartilage there is a large
٣٤ ‫صفحة‬
Surgery Made Easy

hematoma. And stridor develops what is the most appropriate initial


management?
A. Bronchoscopy
B. Close observation
C. Endotracheal intubation
D. CT angiography of the neck 


28. 30 yrs old male was brought to the ER after a car accident, he was alert, with
right chest wall contusion. Fraction crepitation over several right ribs. And
absent breath sound on the right side. Bp: 80/50 HR: 130 RR: 30, What is the
most likely diagnosis?
A. Right lung contusion
B. Tension pneumothorax
C. Simple pneumothorax
D. Lung contusion 


- Pneumothorax Lung collapses secondary to parenchymal injury from penetrating


trauma or blunt trauma (sharp fragments from rib fractures).
- Small pneumothoraces less than 1 cm may be managed conservatively if there is no
respiratory compromise.
- Tension pneumothorax is a clinical diagnosis and an immediate threat to life.
Diagnose by tracheal deviation, distended neck veins, diminished lung sounds, and
hemodynamic instability. mont

29. Trauma Patient, had liver contusion treated conservative, 2 weeks later came
with colicky abdominal pain and with melena, upper GI showed no ulcers but
there was retained blood in duodenum on examination vitally stable and no
tenderness but was mildly jaundiced his Hb was 10, What is next?
A. Tagged RBC
B. Angio
C. Serial Hb level

- Angioembolization and endoscopic retrograde cholangiopancreatography (ERCP) are


useful adjuncts that can improve the success rate of nonoperative management. The
indication for angiography to control hepatic hemorrhage is transfusion of 4 units of
RBCs in 6 hours or 6 units of RBCs in 24 hours without hemodynamic instability.
Schwartz

30. Post RTA, CT abd was done and it showed significant air around the spleen, with
no evidence of extravasations. What is the best management?
A. Splenectomy
B. Partial splenectomy
C. Observation

31. Pt with swallowed cleaning agent ( alkali solution ) presented with stridor?
A. EGD
B. NGT lavage
C. Drink neutralizing agent
D. intubation

٣٥ ‫صفحة‬
Surgery Made Easy

- Dx caustic injury. Signs of impending respiratory compromise may include a hoarse


voice, wheezing, or stridor; subjective dyspnea is a particularly concerning symptom
and should trigger prompt elective endotracheal intubation. Schwartz
- If respiratory distress is present or there are signs of severe oropharyngeal or glottic
edema and/or necrosis, the patient should be intubated for airway protection prior to
continuing the endoscopy. uptodate
- No NG tube. Do not induce vomiting. Nothing to drink. Fiser

32. Pt with stab wound to the abdomen, intra op finding of 2 cm transverse colon
injury treatment ?
A. Segmental resection
B. Primary repair
C. Diverting colostomy

- Right and transverse colon injuries Tx: 1) primary repair or 2) resection and


anastomosis (for destructive injuries [ie > 50% circumference or associated with
- significant colon devascularization]); all are essentially treated like small bowel
injuries
- No diversion needed for right and transverse colon injuries. Fiser


33. Stab wound to the neck zone 2 with blood in the NGT , what is the next step?
A. Neck exploration

34. Patient 16 days post-operative complain of dyspnea... With big clotted


hemothorax, cultured taken and patient on antibiotics,,he has dysnea , started on
antibiotic, what is the management?
A. Chest tube
B. Rib excision and open drainage
C. Decortication
D. Long term Abx

- The most common complication after thoracic injury is development of an empyema.


Management is based on CT diagnostic criteria. 94 Percutaneous drainage is indicated
for a single loculation without appreciable rind. While fibrinolytics are often used for
empyema there is a paucity of data to support their use. Early decortication via video-
assisted thoracic surgery should be done promptly in patients with multiple loculations
or a pleural rind of >1 cm. 95 Antibiotic treatment is based on definitive culture results,
but presumptive antibiotics should cover MRSA in the SICU. Schwartz
- Empyema may also result from retained hemothorax.
- If TPA fails, then VATS versus open thoracotomy for drainage and decortication is
necessary. mont

35. Patient open eye for pain, localize( withdrawal ???)pain with inappropriate
speech what is GCS ? 2018
A. 8
B. 9
C. 10
D. 11

٣٦ ‫صفحة‬
Surgery Made Easy

- read about GCS table

36. Patient brought to ER unstable with pelvic fracture, FAST showed free fluid in
Morison pouch , your action?
A. Pelvic stabilization, then laparotomy
B. Angioembolization, pelvic stabilization then laparotomy
C. CT scan abd/pelvis/chest
D. Laparotomy then Pelvic stabilization

- See Figure 7-69. Management algorithm for patients with pelvic fractures with
hemodynamic instability P213 . Schwartz

37. Patient brought as RTA, distended jugular vein and absent air in Lt side of chest,
tachypnic, this is associated with ?
A. muffled heart sound
B. Arterio venous mismatch
C. Increase central venous pressure

38. pt with massive transfusion his urinary bladder pressuser 35 next?


A. decompressive laparotomy

- Dx Abdominal compartment syndrome


- Sx’s – hypotension, dis tended abdomen, low urine output, increased airway
pressures, prolonged transport time
- Occurs after massive fluid resuscitation, trauma, or abdominal surgery
- A pressure greater than 20 mmHg constitutes intraabdominal hypertension, but the
diagnosis of ACS requires intra-abdominal pressure greater than 25 to 30 mmHg,
with at least one of the following: compromised respiratory mechanics and ventilation,
oliguria or anuria, or increasing intracranial pressures.
- Patients with intra-abdominal hypertension should be monitored closely with repeated
examinations and measurements of bladder pressure, so that any further
deterioration is detected and operative management can be initiated. Left untreated,
ACS may lead to multiple system end-organ dysfunction or failure and has a high
mortality.
- Bladder pressure > 25–30 suggests compartment syndrome
- IVC compression is the final common pathway for decreased cardiac output
- Caused by swollen abdominal contents
- Low cardiac output causes visceral and renal malperfusion (↓ urine output)
- Upward displacement of diaphragm affects ventilation
- Tx: decompressive laparotomy. Fiser and Schwartz 


39. Post RTA CT showed sever spleen injury with moderate free fluid ( he didn’t
mention the grade ), Vitally he is stable, what is determined his plan of
treatment ?
A. Serial HB level
B. hemodynamic stability
C. CT grading
D. Free fluid amount
٣٧ ‫صفحة‬
Surgery Made Easy

- Indications for early intervention include initiation of blood transfusion within the first
12 hours and hemodynamic instability. Schwartz

40. ARDS patient start on PEEP as initial treatment then he become hypotensive,
what is the Dx ?
A. Pneumothorax

- Excessive PEEP complications – ↓ right atrial filling (main reason for ↓ed


CO), ↓BP, ↓ renal blood flow (↑ed renin), ↓ urine output, ↑ wedge pressure,
and ↑pulmonary vascular resistance. Fiser
- Barotrauma (injury to airway pneumothorax), decreased CO from decreased preload.
recall

41. Chest blunt trauma with sings of aortic dissection, depressed left main bronchus
and wide mediastinum in chest x-ray , Most accurate method to diagnose?
A. CT angio
B. transesophageal echocardiogram (TEE)
C. aortography

- Once the diagnosis of dissection is considered, the thoracic aorta should be imaged
with CT, MRA, or echocardiography. The accuracy of these noninvasive imaging tests
has all but eliminated the need for diagnostic aortography in most patients with
suspected aortic dissection. Schwartz
- see Figure 22-20. Algorithm used to facilitate decisions regarding treatment of acute
aortic dissection. Shwartz P810

42. Trauma laceration , irrigation with ?


A. NS
B. RL

- Irrigation to visualize all areas of the wound and remove foreign material is best
accomplished with normal saline (without additives).Schwartz

43. Post RTA, bladder injury, taken to the OR, no significant injuries, found to have
adhesion from previous abdominal surgery you noticed 3 holes in 6 cm distance
at ileum what is your action?
A. Primary repair and drain
B. Resection... 


- Resect complex injuries (grades III–V).


- Usually stapled, functional side-to-side anastomosis for the sake of time
- May need to create ostomy if delayed diagnosis, unstable patient
- Preservation of ileocecal valve if possible if significant length of bowel is being
resecte
- Less than 200-cm jejunum and ileum at risk for short bowel syndrome


٣٨ ‫صفحة‬
Surgery Made Easy

44. A 28 year old female with large Pelvic hematoma managed conservatively... you
noticed a drop of HB from 12 to 10 over 3 days...what is your next step?
A. FFP BID for 2 days
B. Close monitoring HB and VS
C. Exploration

45. Young male s\p trauma with pnemthorax and femur fracture, chest tube had been
inserted and immobilizations of femur was performed, during preparation for
transportation to another hospital he suddenly developed hypotension and
decrease o2 saturation next step?
A. Intubate
B. IV fluid
C. Recheck the patency of the tube and location
D. Proceed for transfer
E. Re check the femur support 


46. A 55 years old man is brought to ER after a high speed MVA. He is


heamodynamically stable with cross hematuria , CT cystography showed air in
bladder and contrast extravasations to paracolic gutter management is?
A. Folys catheter
B. OR for primary repair
C. Bilateral nephrostomy tubes 


- BLADDER TRAUMA:
- Hematuria best indicator of bladder trauma
- Blood at the meatus or scrotal/sacral hematoma – suspect bladder or urethral injury
- > 95% associated with pelvic fractures (blunt trauma)
- Signs and symptoms – meatal blood, sacral or scrotal hematoma
- Dx: cystogram
- Extraperitoneal bladder rupture – cystogram shows starbursts Tx: Foley 7–14 days

Intraperitoneal bladder rupture – more likely in kids, cystogram shows leak Tx:
operation and repair of defect, followed by Foley drainage

47. A 25 years old man presented with gunshot wound to the buttocks, abdominal
examination in unremarkable and he is heamodynamically stable, CT abdomen &
pelvis unremarkable. Proctoscopy reveals blood & stool in distal rectal vault but
no injuries identified which of the following is the best management option?
A. Proximal diverting colostomy for distal extraperitoneal rectal injury
B. Primary closure of the proximal extraperitoneal rectal injury, diverting 

colostomy distal rectal irrigation
C. Presacral drainage and antibiotics
D. APR 


48. Pt fall down from a bridage 6 m and hypotensive bradycardia and warm on
examination unremarkable peripherals What is the type of shock? 2018
A. Neurogenic
B. Septic
C. Hemorhhegic.

٣٩ ‫صفحة‬
Surgery Made Easy

- Warm skin in Septic and neurogenic shock depend on examination

49. Patient post RTA in ICU, intubated, with failure of multiple attempts to extubate ,
when to consider tracheostomy? 2018
A. 5 days
B. 10th days
C. 15th days
D. 20 days

- Endotracheal tube or tracheostomy tube; and need for reintubation within a specified
time period: either within 24-72h or up to 7 days. PMC2760915

50. Patient with pneumothorax on 2 ICT but not improve with continues air leak what
is next? 2018
A. Fibrotic bronchoscopy
B. CT
C. Reposition the tubes 


- Patients with persistent pneumothorax, large air leaks after tube thoracostomy, or
difficulty ventilating should undergo fiber-optic bronchoscopy to exclude a
tracheobronchial injury or presence of a foreign body. Schwartz

51. Pt has been stapped on the right middle thigh present to ER after 10 hours with
massive bleeding, now despite the blood and fluid resuscitation still un stable.
Patient is diabetic, most likely cause? 2018
A. Brain hypoxia
B. Air embolism
C. Hyperglycemia 


52. Pt after CT with contrast developed urticaria and wheezing, Vitals accepted ?
A. Prednisone
B. Epinephrine.
C. antihistamine

- pt developed anaphylactic shock.

53. Pt with oliguria post AAA repair What is the minimal IVP pressure for
decompression? 2018
A. 25
B. 45
C. 15
D. 35

- see Q 38

٤٠ ‫صفحة‬
Surgery Made Easy

Burn
1. 70 kg pt, with 40% burn calculate the fluid for 24 hours?
A. 12.6 


- So Parkland Formula (4*70*40) + Urine output (.5*40*70) = 12.6


- The Parkland Formula is as follows:
- Fluid for first 24 hours (ml) = 4 * Patient's weight in kg * %BSA
- Afterwards, the first half of this amount is delivered in the first 8 hours, and the
remaining half is delivered in the remaining 16 hours.
- Urine output best measure of resuscitation (0.5 cc/kg/h in adults, 1 cc/kg/h in
children, 2 cc/kg/hr in infants < 6 months). Fiser


2. Young boy post inhalation injury, what is best predictor of mortality ?


A. hypoxia
B. Airway obstruction
C. Infection

- The combination of burns, inhalation injury, and pneumonia increases mortality by up


to 60% over burns alone. Schwartz
- Pneumonia – most common infection in patients with > 30% BSA burns; also most
common cause of death after > 30% BSA burns; inhalational injury #1 RF for
pneumonia in burn patients. Fiser

3. Patient with p/H/O burn with lateral neck chronic nodular lesion and induration
what is management?
A. Lesion excision and full thickness graft
B. Lesion excision and partial thickness graft
C. biopsy from the edge

- (incisional biopsy / punch biopsy ) if large lesion or at narrow area ... If small lesion
excisional biopsy with adequate margin

4. Pt came with electrical burn given iv fluid but persistent dark urine, next?
A. Give more fluids
B. HCO3

- Dx myoglobinuria due to renal injury.


- Most of the destruction from electrical burns is internal because the route of least
electrical resistance follows nerves, blood vessels, and fascia; injury is usually worse
than external burns at entrance and exit sites would indicate; cardiac dysrhythmias,
myoglobinuria, acidosis, and renal failure are common
- ;lj Muscle damage results in release of hemochromogens (myoglobin), which are
filtered in the glomeruli and may result in obstructive nephropathy. Therefore,
vigorous hydration and infusion of intravenous sodium bicarbonate (5% continuous
infusion) and mannitol (25 g every 6 hours for adults) are indicated to solubilize the
hemochromogens and to maintain urine output if significant amounts are found in the
serum. These patients also require additional intravenous volumes above predicted
٤١ ‫صفحة‬
Surgery Made Easy

amounts based on the wound area because most of the wound is deep and cannot
be assessed by standard physical examination. In this situation, urine output should
be maintained at 2 mL/kg/hr. Sabiston
- To avoid renal injury, think “HAM”: Hydration with IV fuids, Alkalization of urine
with IV bicarbonate Mannitol diuresis

5. young male with 30% flame burn in the body, Most appropriate dressing?
A. partial thickness skin graft
B. Aqacell silver
C. Wet to dry 


6. According to the parkland formula, the amount of fluids that should be


administered in the first 8 hours to a 70 kg man with 50% TBSA burn?
A. 875ml\hRL
B. 437.5 ml\h RL
C. 625 ml\h NS
D. 732 ml\h NS 


٤٢ ‫صفحة‬
Surgery Made Easy

Postoperative and Surgical


complication
1. Case scenario for pt post LAR 10 the day develop fever and abdominal pain CT
abdomen revealed 8x8 cm pelvic collection at site of anastomosis what is the
next?
A. percutaneous drainage
B. Re exploration and anastomosis repair
C. Drainage with stoma
D. Conservative treatment with Abs

- For management of major anastomotic bleeding, the initial management should be


conservative with supportive care, including blood transfusions and correction of any
underlying coagulopathy. Endoscopic and/or surgical intervention should be reserved
for unstable patients or those with persistent bleeding despite conservative measures.
uptodate
- For patients who present with localized peritonitis and low-grade sepsis, a diagnostic
imaging workup is initiated. We perform a computed tomography (CT) scan with oral,
intravenous, and rectal contrast. Alternatively, a water-soluble contrast enema may be
performed, if available in your institution. If a leak is present, the majority will be
localized.
- If a free intraperitoneal leak is demonstrated, the patient should be taken to the
operating room for surgical management.
- If the patient is stable with small, contained abscesses (<3 cm), we recommend
conservative management with broad-spectrum antibiotics and bowel rest.
- For larger abscesses (>3 cm), multiloculated collections, or multiple collections, an
attempt at percutaneous drainage should be made. In those cases where image-
guided drainage is not technically feasible or where the patient's clinical condition
deteriorates despite drainage, surgical intervention in the form of an exploratory
laparotomy should be undertaken as described in the following paragraph.

2. Patient during difficult lap cholecystectomy received two units of PRBCs and
post-operative found to have renal impairment Most common cause of post op
renal failure in a patient with normal pre op renal function is?
A. Sepsis
B. Hypotension intra operative
C. Drug toxicity
D. Operative injury to the renal system

- Acute renal failure occurs in up to 30 percent of patients who have undergone cardiac
surgery and it appears to be associated with increased mortality. The best preventive
strategy is to optimize renal perfusion (ie, avoid hypotension and hypovolemia) and to
avoid potentially nephrotoxic agents (eg, aminoglycoside antibiotics, angiotensin
converting enzyme inhibitors, and radiologic contrast agents) in the immediate
postoperative period. There is no convincing evidence of benefit from early and/
or aggressive dialysis. uptodate
٤٣ ‫صفحة‬
Surgery Made Easy

3. Pt with sign and symptoms of PE, what is the most important investigation?
A. spiral CT

- The pulmonary angiogram remains the gold standard for diagnosing PE, but spiral CT
angiogram has become an alternative method because of its relative ease of use and
reasonable rates of diagnostic accuracy. Schwartz

- The pulmonary angiogram remains the gold standard for diagnosing PE, but spiral CT
angiogram has become an alternative method because of its relative ease of use and
reasonable rates of diagnostic accuracy. Schwartz

4. Post knee replacement develop abd pain + nausea + vomiting Glucose low,
Hyponatremia Hyperkalemia Normal urine output Vital stable?
A. Septic shock
B. Intra abd bleeding
C. PE.
D. adrenal insufficiency

- Acute adrenal insufficiency – hypotension (refractory to fluids and pressors), fever,


lethargy, abdominal pain, nausea and vomiting, ↓ glucose, ↑ K, ↓ Na
- Tx: dexamethasone, fluids, and give cosyntropin test (dexamethasone
does not interfere with test). Fiser


5. Pt. post op day 2 he was complaining of SOB and RUQ mild pain, on
examination pt looks ill, and having mild epigastria tenderness, with absent of
the bowel sound. Vitals: HR: 120, O2%: 90 in room air, temp: 37.8?
A. Postoperative Ileus
B. PE
C. Atelectasis
D. Wound infection 


- FEVER:
- MC fever source within 48 hours             Atelectasis
- MC fever source 48 hours – 5 days        Urinary tract infection
- MC fever source after 5 days                      Wound infection
- Fiser

6. Post op day 1, pt. developed fever, what is next ?


A. CXR
B. Chest physiotherapy

- Atelectasis Most common cause of fever in first 48 hours after operation


- Tx: incentive spirometer, pain control, ambulation
th
7. Post op pt, develop fever on the 6 day post op what is the cause?
A. Wound infection
B. Atelectasis
C. UTI

٤٤ ‫صفحة‬
Surgery Made Easy

- Intraoperative fever may be secondary to malignant hyperthermia, a transfusion


reaction, or a pre-existing infection.
- Fever in the first 24 hours usually occurs as a result of atelectasis. A high fever
(>39°C) is commonly the result of a streptococcal or clostridial wound infection,
aspiration pneumonitis, or a pre-existing infection. However, fever in this time period
can also be seen in trauma or burn patients as a part of the SIRS response in the
absence of infection.
- Fever that occurs more than 72 hours after surgery has a broad differential
diagnosis, including but not limited to the following: wound infection (including fascial
or muscle infections), pneumonia, gastroenteritis, infectious colitis (including C.
difficile), abscesses, peritonitis, UTI, infected prosthetic materials or catheters, DVT,
thrombophlebitis, drug allergy, or devastating neurologic injury. In
immunocompromised hosts, viral and fungal infections should also be considered.
Transfusion reactions can be confused for infection due to the presence of fever;
although, the treatment is vastly different and will not be discussed here.


8. Most important predictor of morbidity?


A. Pco2 >47
B. Po2 low
C. Age
D. FEV1

- Pulmonary function studies are routinely performed when any resection greater
than a wedge resection will be performed. Of all the measurements available, the
two most valuable are forced expiratory volume in 1 second (FEV 1 ) and carbon
monoxide diffusion capacity (Dlco).
- The respiratory function is best assessed with the forced expiratory volume in 1
second. Schwartz

9. 27 yrs old, male pt. post complicated appendectomy after OR shifted directly to
ICU, which one of the following is the most likely to present?
A. Increase aldosterone
B. Decrease insulin
C. Decrease cortisol
D. Decrease epinephrine

- Dx shock. aldosterone is produced in response to stimulation by angiotensin II.


Aldosterone is a mineralocorticoid that modulates renal function by increasing
recovery of sodium and excretion of potassium. One of the problems in shock is that
the release of all of these hormones is not infinite; they can be exhausted.
- The other answer increase
- see images table


٤٥ ‫صفحة‬
Surgery Made Easy

Nutrition/TPN
1. patient with small bowl obstruction post-operative laparotomy pt develop high
output enterocutaneous fistula what is appropriate management ?
A. Central line
B. jejunostomy
C. Gastrostomy
D. NGT

- There is no evidence to support withholding enteric feedings for patients after bowel
resection or for those with low-output enterocutaneous fistulas of <500 mL/d. In fact,
a recent systematic review of studies of early enteral feeding (within 24 hours of
gastrointestinal surgery) showed no effect on anastomotic leak and a reduction in
mortality. Early enteral feeding is also associated with reduced incidence of fistula
formation in patients with open abdomen. Enteral feeding should also be offered to
patients with shortbowel syndrome or clinical malabsorption, but necessary calories,
essential minerals, and vitamins should be supplemented using parenteral modalities.
- indiction of parenteral nutrition in Patients with enteroenteric, enterocolic,
enterovesical, or high-output enterocutaneous fistulas (>500 mL/d)

2. A 37-year-old woman with Crohn disease who has been managed on TPN for 6
months complains of nonhealing ulcers on her lower extremities. On physical
exam, she has bilateral pretibial erythematous areas with blisters, pustules, and
superficial ulcerations. Which of the following elemental deficiencies would be
most consistent with this clinical picture?
A. Iron
B. Copper
C. Chromium
D. Zinc
E. Selenium

- Acrodermatitis enteropathica is a recessively inherited partial defect in intestinal zinc


absorption. Affected infants develop an erythematous and vesiculobullous dermatitis,
alopecia, ophthalmic disorders, diarrhea, and severe growth retardation. The
syndrome responds to oral supplementation with pharmacologic doses of zinc.

3. After massive bowel resection for IBD, a young male whom, He is now
dependant on TPN 5 days per week. He is having 3L of stool output per day.
There is no evidence of enteric infection. How to improve his condition?
A. oral bile salt
B. Erythro mycin
C. motilin

- Antimotility agents, such as loperamide hydrochloride or diphenoxylate, may be


administered to delay small-intestinal transit. Octreotide can be administered to
reduce the volume of GI secretions, although in animal models, its use is associated
with an inhibition of intestinal adaptation.

٤٦ ‫صفحة‬
Surgery Made Easy

4. Most common complication with subclavian line?


A. obstruction
B. Thrombosis
C. infection
D. Sepsis

- One of the more common and serious complications associated with long-term
parenteral feeding is sepsis secondary to contamination of the central venous
catheter. Schwartz
- The rate of catheter infection is highest for those placed in the femoral vein, lower for
those in the jugular vein, and lowest for those in the subclavian vein. schwartz

6. 54 years old female admitted electively with gastric outlet obstruction, she lost
10 kg in 3 months HB 10 ,PLT 105 , ALB 25 LFT minimal elevation preoperative
management ?
A. IV albumin
B. Transfusion PRBC
C. Platelet Transfusion
D. Start TPN

- The principal indications for parenteral nutrition are malnutrition, sepsis, or surgical or
traumatic injury in seriously ill patients for whom use of the gastrointestinal tract for
feedings is not possible.

7. Pt post trauma and brain injury was unconscious for long time best way to feed?
A. NGT
B. Nasojujenal
C. Jejunostomy
D. Gastrostomy (PEG tube )


- Percutaneous Endoscopic Gastrostomy for Patients who are unable to move food


from their mouth to their stomach are the ones who commonly need PEG tube
placement. This includes those with neurologic disorders such as stroke, cerebral
palsy, brain injury, amyotrophic lateral sclerosis, and impaired swallowing. medscape
- Jejunostomy SUITABLE for Long term—functional GI tract but poor gastric emptying,
reflux, aspiration risk, gastroparesis or gastric dysfunction
- Gastrostomy SUITABLE for Long term—good gastric emptying; avoid if significant
reflux or aspiration problem Sabiston
- Nasogastric SUITABLE for Short term—functional GI tract
- Nasoduodenal, nasojejunal SUITABLE for Short term—functional GI tract but poor
gastric emptying, reflux, aspiration risk; begin feed only when volume resuscitated
and hemodynamically stable

8. Pt on TPN with alopecia and perioral lesion what will cause that?
A. Zinc
B. Ph
C. Mg 


٤٧ ‫صفحة‬
Surgery Made Easy

- Acrodermatitis Enteropathica is characterized by impaired wound healing as well as


erythematous pustular dermatitis involving the extremities and the areas around the
bodily orifices. Schwartz
- Zinc = Poor wound healing, perioral rash, hair loss, dysgeusia

9. 65 yrs old average built male is admitted for surgery of gastric antrum carcinoma
which is causing gastric outlet obstruction. He has lost 10 Kg. in last 3 months.
Hb: 110.2 g/L plt: 105*10 Prothrombin time: 13 sec INR: 1.1 PTT: 45 sec albumin:
29g, which of the following will be an appropriate pre op preparation ?
A. Blood transfusion
B. IV albumin transfusion
C. TPN
D. platelet concentrate transfusion

10. 50 yrs old man is on TPN, for short bowel syndrome, TPN stopped for
radiological investigation during the procedure patient becomes drowsy and
then unconscious. what is the most likely cause of his condition?
A. Hyponatremia
B. Hypomegnesimia
C. Hypoglycemia
D. Hypophophotimia 


- If TPN is suddenly discontinued for any reason, IV administration of any 5% dextrose


solution at the same infusion rate will prevent hypoglycemia.
- When discontinuing TPN, decrease current TPN bag to 42 mL/h for 1 hour before
cessation of TPN (if patient has counterregulatory issues, decrease TPN over 2–3
hours before cessation). mont

11. Patient on long term TPN, develop scaly dermatitis ?


A. Essential fatty acid
B. Zinc

- During prolonged parenteral nutrition with fat-free solutions, essential fatty acid
deficiency may become clinically apparent and manifests as dry, scaly dermatitis and
loss of hair. The syndrome may be prevented by periodic infusion of a fat emulsion at
a rate equivalent to 10% to 15% of total calories.
- Essential trace minerals may be required after prolonged TPN and may be supplied
by direct addition of commercial preparations. The most frequent presentation of
trace mineral deficiencies is the eczematoid rash developing both diffusely and at
intertriginous areas in zinc deficient patients. Schwartz

12. Pt NPO for 10 days, what is his source of energy?


A. Fat and protein
B. Fat and glucagon
C. ketoacids

- In the healthy adult, principal sources of fuel during short-term fasting (<5 days) are
derived from muscle protein and body fat, with fat being the most abundant source of
energy. Schwartz

٤٨ ‫صفحة‬
Surgery Made Easy

- After an overnight fast, liver glycogen is rapidly depleted as glucagon responds to


falling serum glucose levels. Carbohydrate stores are exhausted after 24 hours. For
the first few days during starvation, caloric needs are met by fat and protein
degradation. Most of the protein is from breakdown of skeletal and visceral muscle,
which is converted to glucose via hepatic gluconeogenesis. The brain preferentially
uses this endogenously produced glucose, with the remainder consumed by red
blood cells and leukocytes. Within approximately 10 days of starvation, the brain
adapts and uses fat in the form of ketoacids as its fuel source. Use of ketoacids has a
protein-sparing effect. Washington

13. patient in ICU with long term TPN , pneumonia and pancreatitis presented with
generalize muscle weakens : K normal , phosphate low , Na

normal vitally stable a febrile the most likely cause is?
A. refeeding syndrome
B. rhubdomyolysis
C. sepsis

- Dx sepsis : high fever meaning there is source so


- One of the earliest signs of systemic sepsis from Central line–associated
bloodstream infections (CLA-BSI) may be the sudden development of glucose
intolerance (with or without temperature increase) in a patient who previously has
been maintained on parenteral alimentation without difficulty. When this occurs, or
if high fever (>38.5°C [101.3°F]) develops without obvious cause, a diligent search
for a potential septic focus is indicated. Other causes of fever should also be
investigated. If fever persists, the infusion catheter should be removed and
submitted for culture. If the catheter is the cause of the fever, removal of the
infectious source is usually followed by rapid defervescence. Some centers are
now replacing catheters considered at low risk for infection over a guidewire.
However, if blood cultures are positive and the catheter tip is also positive, then the
catheter should be removed and placed in a new site. Should evidence of infection
persist over 24 to 48 hours without a definable source, the catheter should be
replaced into the opposite subclavian vein or into one of the internal jugular veins
and the infusion restarted. Schwartz
- Refeeding syndrome Occurs when feeding after prolonged starvation/
malnutrition, ETOH abuse often present, Symptoms usually occur on day
4 following re-feeding, Results in decreased K, Mg, and PO4; causes cardiac
dysfunction, profound weakness, encephalopathy, CHF, failure to wean from the
ventilator, Prevent this by starting to re-feed at a low rate (10–15 kcal/kg/day).fiser

14. 75 yr female is admitted in ICU after Hartmann’ procedure For obstructing


sigmoid CA, she developed fever, septic work up is negative except for positive
blood culture which showed staphylococcus aureus. What is the next
appropriate Treatment?
A. AmphotericinB
B. Change A bx
C. Change IV line over guide wire
D. Remove central line & send tip for culture

٤٩ ‫صفحة‬
Surgery Made Easy

- Some centers are now replacing catheters considered at low risk for infection over a
guidewire. However, if blood cultures are positive and the catheter tip is also positive,
then the catheter should be removed and placed in a new site.
- If the catheter is the cause of the fever, removal of the infectious source is usually
followed by rapid defervescence.
- Should evidence of infection persist over 24 to 48 hours without a definable source,
the catheter should be replaced into the opposite subclavian vein or into one of the
internal jugular veins and the infusion restarted. Schwartz

15. An ICU patient with necrotizing pancreatitis, on TPN via central line, he is febrile
& blood culture is positive for fungal infection...the next step ?
A. Remove and replace with guide wire
B.
C. Remove and send the tip for C/S

- See explanation Q 14

16. Burn pt> 60%, in ICU and TPN feeding...the Best nutritional indication?
A. Urine UOP
B. Increased weight
C. Decreased catabolism
D. Negative nitrogen balance 


- These nutrients can be given in quantities considerably greater than the basic caloric
and nitrogen requirements, and this method has proved to be highly successful in
achieving growth and development, positive nitrogen balance, and weight gain in a
variety of clinical situations. Schwartz
-
17. A 50 years old man is in the ICU with necrotizing pancreatitis on TPN. he is
struggling to wean from ventilator you suspect over feeding syndrome and elect
to perform indirect calorimetry, which of the following values for respiratory
quotient, would be consistent with an over feed syndrome?
A. 0.65
B. 1.3
C. 0.7 


- RESPIRATORY QUOTIENT (RQ)


- Ratio of CO2 produced to O2 consumed – is a measurement of energy expenditure
- RQ > 1 = lipogenesis (overfeeding), Tx: ↓ carbohydrates and caloric intake
- High carbohydrate intake can lead to CO2 buildup and ventilator problems
- CO2 is produced when excess carbohydrates are converted to fats
- RQ < 0.7 = ketosis and fat oxidation (starving). Tx: ↑ carbohydrates and caloric intake
- Pure fat utilization – RQ = 0.7
- Pure protein utilization – RQ = 0.8
- Pure carbohydrate utilization – RQ = 1.0
- Balanced nu trition – RQ = 0.825
- Fiser.

٥٠ ‫صفحة‬
Surgery Made Easy

18. Patient post illioectomy and has fistula his wt is 70 kg what is the calories in take
should be?
A. 1400
B. 1800
C. 2200
D. 2000

- 70X30 = 2100 kcl/d


- It has been demonstrated that the provision of 30 kcal/kg per day will adequately
meet energy requirements in most postsurgical patients, with a low risk of
overfeeding. After trauma or sepsis, energy substrate demands are increased,
necessitating greater nonprotein calories beyond calculated energy expenditure
(Table 2-10). Schwartz P51

19. Pt with central line for IVF with difficult peripheral line has bacteraemia blood
culture came staph. Epidermis, what is next? 2018
A. Change abx under guid wire
B. Removal of the line
C. Change over guide wire

- see Q14

٥١ ‫صفحة‬
Surgery Made Easy

Wound healing
1. Both result in hair loss and poor wound healing
A. Vit A
B. Vit C

- The vitamins most closely involved with wound healing are vitamin C and vitamin A.
- Vitamin C = Scurvy = swollen gums, bruising, petechiae, hemarthrosis, anemia, poor
wound healing, perifollicular and subperiosteal hemorrhages, “corkscrew” hair.
- Vitamin A = Night blindness (nyctalopia); dry, scaly skin (xerosis cutis); corneal
degeneration (keratomalacia); Bitot spots (foamy appearance) on conjunctiva

2. 17 yrs old girl, she has contaminated deep laceration wound, last tetanus dose
was taken 12 yrs back, what is next?
A. Nothing
B. Tetanus only
C. Toxoid only
D. tetanus and ABx

- Tetanus immunization should be considered for anyone with a traumatic open wound.
It should definitely be given when a wound is high risk for a Clostridium tetani
infection, including wounds that are deep, contaminated, and with devitalized tissue.
mont
- Patients should receive tetanus toxoid for penetrating injuries if more than 5 years
have passed since the last vaccination. schwartz
- All injured patients undergoing an operation should receive preoperative antibiotics.
The type of antibiotic is determined by the anticipated source of contamination in the
abdomen or other operative region; additional doses should be administered during
the procedure based on blood loss and the half-life of the antibiotic. Extended
postoperative antibiotic therapy is administered only for contaminated open fractures.
Tetanus prophylaxis. Schwartz

3. 45 yrs old known diabetic with chronic renal failure develops wound infection
post appendectomy, he is using prednisolone for bronchial asthma.

which of the following is the major cause of umpired wound healing?
A. Anemia
B. DM
C. Local tissue infection
D. Steroid Use 


- Steroids inhibit wound macrophages, fibroplasia, angiogenesis, and wound


contraction.
- They increase the risk for infection.
- Vitamin A supplementation, 25,000 international units (IU) oral daily preoperatively
and 4 days postoperatively, aids in promoting epithelialization and collagen synthesis.
- other factors ( Oxygenation, Infection, Nutrition, Smoking, Age, Foreign Bodies,
Chemotherapy, DM ). mont

٥٢ ‫صفحة‬
Surgery Made Easy

- Large doses or chronic usage of glucocorticoids reduce collagen synthesis and


wound strength. 64 The major effect of steroids is to inhibit the inflammatory phase of
wound healing (angiogenesis, neutrophil and macrophage migration, and fibroblast
proliferation) and the release of lysosomal enzymes. Schwartz 


4. 53 yrs old female pt. underwent open cholecystectomy for symptomatic


gallstone with primary closure of the surgical wound. What is the time period
required for complete Re-epithilaztion of such wound?
A. 1 day
B. 2 days
C. 3 days 


- Re-epithelialization is complete in less than 48 hours in the case of approximated


incised wounds, but may take substantially longer in the case of larger wounds,
where there is a significant epidermal/dermal defect. Schwartz

5. 27 yrs old female post appendectomy developed wound infection and was
treated by opening the wound and regular dressing and debridement. which of
the following has the most significant effected on wound healing?
A. Vit A
B. Vit B6
C. Vit D
D. Vit E 


- The vitamins most closely involved with wound healing are vitamin C and vitamin A.
Schwartz

6. Patient with toe ulcer did not heal for 5 years biopsy showed
pseudoepitheliomatous hyperplasia what will you do?
A. Tight dressing
B. Ray amputation
C. Surgical debridement
D. Repeat biopsy

- PEH is a benign condition and can be managed with surgical excision with adequate
margin and antibiotics, hence it has to be differentiated from other mimickers of SCC
both benign and malignant.

7. Collagen tensile strength?


A. After 24 hours
B. After 48 hours
C. 4- 6 days
D. One month

- Tensile strength – most important factor in healing closed incisions (primary


intention). Fiser
- Re-epithelialization is complete in less than 48 hours in the case of approximated
incised wounds, but may take substantially longer in the case of larger wounds,
where there is a significant epidermal/dermal defect. schwartz

٥٣ ‫صفحة‬
Surgery Made Easy

8. Mechanism of steroid in defective wound healing?


A. induce fibroblast
B. induce PMNs
C. inhibit macrophages
D. decrease wound tension

- Large doses or chronic usage of glucocorticoids reduce collagen synthesis and


wound strength. 64 The major effect of steroids is to inhibit the inflammatory phase of
wound healing (angiogenesis, neutrophil and macrophage migration, and fibroblast
proliferation) and the release of lysosomal enzymes. Schwartz 


9. A patient with keloid scar post surgery, the most effective way to decrease its
incident is ?
A. Pre op intralesional steroid injection
B. Post op local steroid
C. Decrease tension

- The three strategies that reduce adverse scarring immediately after wound closure
are tension relief, hydration/occlusion, and use of taping/pressure garments. Wounds
with greater tension (perpendicular to Langer’s lines), with excessive tension on
closure, and in certain anatomic locations (deltoid and sternal) are at a higher risk of
adverse scarring. sabiston

٥٤ ‫صفحة‬
Surgery Made Easy

Pre-operative assessment,
Anesthesia and pain
management
1. Patient for operation which one of the following effect the cardiac, delayed the
surgery?
A. MI 4 month ago
B. Sinus tachycardia
C. HTN with LT ventricular hypertrophy
D. Premature ventricular beat

- Preexisting hypertension is the most common medical reason for postponing surgery.
Hypertension is well known to be a risk factor for cardiovascular catastrophe, a risk
that logically extends into the perioperative period
- Preexisting hypertension can induce a variety of cardiovascular responses that
potentially increase the risk of surgery, including diastolic dysfunction from left
ventricular hypertrophy, systolic dysfunction leading to congestive heart failure, renal
impairment, and cerebrovascular and coronary occlusive disease. The level of risk is
dependent upon the severity of hypertension. uptodate

2. Which of the following test will predict mostly the post op pulmonary
complications?
A. ABG
B. Pulmonary function test
C. Thread mill test
D. CXR

- Pulmonary function tests will indicate the type and severity of the disease, as well as
response to the treatment.belly
- ABG can be considered in patients with a history of lung disease or smoking to
provide a baseline for comparison with postoperative studies, but is not reliable to
accurately predict postoperative pulmonary complications. wanshtion

3. 32 year old male underwent exploratory laparotomy for adhesive small bowel
obstruction and after 30 min surgeon noticed progressive bowel dilatation With of
the following Most likely cause is?
A. Use of Nitrous Oxide
B. Accumulation intraluminal fluid
C. Esophageal intubation
D. Over dose of muscle relaxant .

- Diffuses into any air-filled cavity to displace nitrogen. Thus, administration is avoided
in patients with possible pre-existing bowel distention, increased middle ear pressure,

٥٥ ‫صفحة‬
Surgery Made Easy

pneumothorax, pneumoperitoneum, pneumocephalus, intraocular gas, or venous air


embolism.

4. Renal patient S/P MI 3 months, safest anesthesia?


A. local
B. epidural
C. spinal
D. general

5. Succinylcholine and fentanyl, pt became hyperthermic, what is the cause (due to


the use of which agent?
A. Depolarizing agent
B. Non-depolarizing agent

- Depolarizing agents – only one is succinylcholine


- Succinylcholine many side effects = Malignant hyperthermia, Hyperkalemia, Open-
angle glaucoma.

- Do not use in patients with severe burns, neurologic injury, neuromuscular disorders,
spinal cord injury, massive trauma, or acute renal failure (all have up-regulation of
acetylcholine receptors which can release significant amounts of K) and children

6. Air embolus, management ?
A. left lateral decubitus, trendelenburg

- Air embolus :
- MC occurs with air sucking through a central line or central line site

- CO2 embolus can occur with laparoscopic procedures

- Sx’s: sudden drop in ETCO2, hypotension, tachycardia, mill wheel murmur (airlock
prevents venous return)

- Tx: stop CO2 insufflation if laparoscopic procedure

- Trendelenburg (head down) and left lateral decubitus position (keeps air in right
ventricle)

- Hyperventilate with 100% oxygen (helps reabsorb air embolus faster)

- Aspirate central line if present (try to remove air)

- Pressors and inotropes

- Prolonged CPR


7. 10 years Hx of DVT, pt booked for abdominal surgery , what pre op to be given?


A. LMWH+IPCD
B. IPCD (pneumatic compression)
C. Heparin
D. LMWH

- Approach to Prophylaxis:
- Determine the Patient’s Risk Factors
- Low risk—age less than 40 years; ambulatory or minor surgery
- Moderate risk—age greater than 40 years; abdominal, pelvic, or thoracic surgery
- High risk—age greater than 40 years; prior DVT or PE, malignancy, hip and other
orthopedic surgeries, immobility, hypercoagulable states
- Prophylaxis of Choice
٥٦ ‫صفحة‬
Surgery Made Easy

- Encourage early ambulation in all patients, and get physical/occupational therapy


involved early, if indicated.


- CHEST guidelines for prevention of VTE in nonorthopedic surgical patients:


- Low risk for VTE (∼1.5%)—suggest mechanical prophylaxis, preferably with
intermittent pneumatic compression (IPC).
- Moderate risk for VTE (∼3%)—suggest LMWH, low-dose unfractionated heparin,
or mechanical prophylaxis with IPC over no prophylaxis.
- High risk for VTE (∼6%)—recommend pharmacologic prophylaxis with LMWH or
low-dose unfractionated heparin over no prophylaxis. In these patients, also
recommend adding mechanical prophylaxis with IPC to pharmacologic
prophylaxis.
- Prophylaxis should be started before the initiation of anesthesia; low-dose SC
unfractionated heparin is usually given in a dose of 5000 units 2 hours preoperatively
and then every 8–12 hours postoperatively.
- High-risk patients should be watched closely for clinical signs and symptoms of DVT.
Duplex scanning is the least invasive method for screening. mont

8. Patient diabetic on oral hypoglycemic going for OR?


A. Continues oral hypoglycemic
B. start him on short acting insulin
C. start him on long acting insulin

- Patients who take oral hypoglycemic agents (sulfonylureas, such as chlorpropamide


and glyburide) typically withhold their normal dose the day of surgery. Patients can
resume their oral agent when diet is resumed. An exception is metformin. If the
patient has altered renal function, this agent needs to be discontinued until renal
function normalizes or stabilizes to avoid potential lactic acidosis. 1 Coverage for
hyperglycemia is with a short acting insulin preparation based on blood glucose
monitoring. Sabiston

9. 10 yrs old body presented with (15-20%) burn he will go for OR for escharotomy.
what is best anesthetic agent to use?
A. Propafol
B. Ketamin 


- Ketamine good for children. Side effects: hallucinations and Contraindicated in


patients with head injury. Fiser
- Succinylcholine don’t give in children's and burn pt.

10. Pt has hx of CVA. and he's going to for OR, when is the most risk to develop CVA
during anesthesia phases?
A. Induction
B. Recovery
C. Reverse
D. Maintenance

- Most catastrophic events happen in induction phase. Schwartz

11. What is the best to know if the tube is in the trachea or esophagus?
٥٧ ‫صفحة‬
Surgery Made Easy

A. ET CO2 "capnography"
B. PCO2
C. Xray
D. Breath sound 


- Correct endotracheal placement is verified with direct laryngoscopy, capnography,


audible bilateral breath sounds, and finally a chest film. The GlideScope ® , a video
laryngoscope that uses fiber optics to visualize the vocal cords, is being employed
more frequently. 9 Advantages of orotracheal intubation include the direct
visualization of the vocal cords, ability to use larger-diameter endotracheal tubes, and
applicability to apneic patients. Schwartz
- Capnography allows the confirmation of endotracheal intubation and continuous
assessment of ventilation, integrity of the airway, operation of the ventilator, and
cardiopulmonary function. Schwartz

12. During lap chole, pt become suddenly bradycardia. What is the most likely the
cause?
A. Fast stretching of the peritoneum
B. Anesthetic agent
C. Insufflations of cold gas

- The pressure effects of the pneumoperitoneum on cardiovascular physiology also


have been studied. In the hypovolemic individual, excessive pressure on the inferior
vena cava and a reverse Trendelenburg position with loss of lower extremity muscle
tone may cause decreased venous return and decreased cardiac output. This is not
seen in the normovolemic patient. The most common arrhythmia created by
laparoscopy is bradycardia.
- A rapid stretch of the peritoneal membrane often causes a vagovagal response with
bradycardia and, occasionally, hypotension. 19 The appropriate management of this
event is desufflation of the abdomen, administration of vagolytic agents (e.g.,
atropine), and adequate volume replacement. Schwartz

13. While a surgeon performing Lap chole. For a 24 yrs old female. She started to be
hypotensive with non specific ECG changes. Her o2% dropped to 88%

which of the following is the most appropriate management?
A. Full hepirinaztion
B. Immediate deflation of the abd.
C. IV ABx
D. Ceasing mechanical ventilation 


- See Q12 Explanation above

14. 30 yrs old pt. sustained 45% burn underwent general anesthesia for surgical
debridement. During the procedure he was found to be hyperthermia then he had
sudden cardiac arrest which of the following drugs may have lead to this
problem?
A. Atracrium
B. Propafol
C. Citoflirne
٥٨ ‫صفحة‬
Surgery Made Easy

D. Succinylcholine

- Succinylcholine don’t give in children's and burn pt.


- Succinylcholine – fast, short acting; causes fasciculations, ↑ ICP; degraded by
plasma pseudocholinesterases (can not be reversed); many side effects →
- A. Malignant hyperthermia Caused by a defect in calcium metabolism
Tx: dantrolene (10 mg/kg) inhibits Ca release and decouples excitation complex;
cooling blankets, HCO3, glucose, supportive care
- B. Hyperkalemia – depolarization releases K Do not use in patients with severe
burns, neurologic injury, neuromuscular disorders, spinal cord injury, massive trauma,
or acute renal failure (all have up-regulation of acetylcholine receptors which can
release significant amounts of K). Fiser

15. 76 yrs old man with history of coronary artery disease and asymptomatic
reducible inguinal hernia request an elective hernia repair. Which of the following
will lead to the delay of surgery?
A. Jugular venous distention
B. Coronary artery bypass surgery 3 months earlier
C. History of smoking
D. HTN

- See cardic risk table 


16. Which one of the following could increase the Postop. Cardiac complication?
A. Smoking
B. Left ventricle hypertrophy
C. MI prior 3 months

- See images " table of goldman risk assessment for non cardiac surgery"

17. Patient with controlled renal dialysis, his ASA?


A. II
B. III
C. IV

- ASA III= poorly controlled DM or HTN, COPD, morbid obesity (BMI ≥40), active
hepatitis, alcohol dependence or abuse, implanted pacemaker, moderate reduction of
ejection fraction, ESRD undergoing regularly scheduled dialysis, premature infant
PCA < 60 weeks, history (>3 months) of MI, CVA, TIA, or CAD/stents.
- https://www.asahq.org/resources/clinical-information/asa-physical-status-
classification-system

18. 75 year old man with no significant past medical history and normal lab test
values is scheduled to undergo an elective mesh plug inguinal hernia repair. In
the pre-operative holding area, his ECG monitor demonstrated an irregularly
irregular rhythm without T waves. His heart rate varies between 70 and 85 bpm.
What is the most appropriate management of this patient?
A. Postpone the OR
B. Thread mell test

٥٩ ‫صفحة‬
Surgery Made Easy

C. Echo, if normal proceed to OR


D. digitalis

- Atrial fibrillation: irregularly irregular ventricular rate with the absence of discernible
P waves
- Hemodynamically unstable patients require immediate synchronized direct current
cardioversion.
- In stable patients, rhythm control can often be achieved pharmacologically with beta-
blockers, calcium channel blockers, digoxin, or antiarrhythmics, such as amiodarone.
- If rhythm control is unattainable, rate control is the next goal. Although current
American Heart Association guidelines suggest similar outcomes for rate and rhythm
control for patients with new onset atrial fibrillation, surgical patients frequently have
an inciting event (operation, acute volume overload) and potential contraindications
for anticoagulation that may make rate control more desirable.
- New-onset atrial fibrillation that persists beyond 48 hours may require
anticoagulation to prevent sequelae of embolization. mont

19. Name of CO2 monitor ?


A. Capnogram.

20. During central line insertion, your patient became hypotensive and chest
auscultation reveled machinery murmur, CXR unremarkable, No widening in
mediastinum what is the cause?
A. Air embolism
B. Vasovagal
C. hemomediastenum
D. pneuomothorax

- see Q6 above

21. A 33 years old female is schedule for lap Chole. pre op evaluation showed TSH..
T4.. (hypothyroid) the next most appropriate action is?
A. Proceed for surgery after starting thyroxin
B. Proceed for surgery & start thyroxin post op
C. Postpone surgery until euthyroid status is achieved

- Patients with newly diagnosed hypothyroidism generally do not require preoperative


treatment, although they may be subject to increased sensitivity to medications,
including anesthetic agents and narcotics. Severe hypothyroidism (high TSH level)
can be associated with myocardial dysfunction, coagulation abnormality, and
electrolyte imbalance, notably hypoglycemia. Severe hypothyroidism needs to be
corrected before elective operations. Hypothyroidism should also be considered in a
severely ill patient who is not recovering from surgery in a normal fashion. sabistone

22. A 37-year-old woman with an otherwise unremarkable medical history


undergoes an elective laparoscopic cholecystectomy for symptomatic
cholelithiasis. Her preoperative blood pressure is 135/82 mm Hg and her pulse is
75 beats per minute. In the operating room...After confirming the position of the
Veress needle, the abdomen insufflated with carbon dioxide. Suddenly, the
patient's pulse drops to 42 beats per minute. And her end-tidal carbon dioxide
٦٠ ‫صفحة‬
Surgery Made Easy

level remains unchanged. Anesthesia asked you to stop, The most appropriate
next step is?
A. Administration of a 500 mL bolus of normal saline
B. Release of abdominal insufflations

23. pt for Elective surgery, and blood glucose was uncontrolled, whats grade of
ASA ?
A. 1
B. 2
C. 3
D. 4

- Grade 3 = Substantive functional limitations; One or more moderate to severe diseases.


Examples include (but not limited to): poorly controlled DM or HTN, COPD, morbid
obesity (BMI ≥40) .
- https://www.asahq.org/resources/clinical-information/asa-physical-status-classification-
system

24. Patient planned for PNS excision Pre op assessment only the uvula base and
soft palate seen What is the class in mallampati? 2018
A. 1
B. 2
C. 3
D. 4

- Mallampati classification:
- class 1: soft palate, fauces, uvula, pillars
- class 2: soft palate, fauces, position of uvula
- class 3: soft palate, base of uvula
- class 4: hard palate

25. Patient with A picture of SBO , hypotensive, The best Anasthesia ?


A. Ketamin
B. Nitric oxide
C. Propofol

- Ketamine is useful in acutely hypovolemic patients to maintain blood pressure via


sympathetic stimulation but is a direct myocardial depressant in patients who are
catecholamine depleted. Ketamine is a bronchodilator, making it useful for asthmatic
patients, and rarely is associated with allergic reactions. Schwartz
- Nitrous oxide cause Diffuses into closed spaces (avoid in patients with small bowel
obstruction or pneumothorax)
- Propofol cause Side effects: hypotension, respiratory depression, metabolic acidosis
(avoid prolonged use in children) Fiser

26. Patient will go for small left big toe lesion excision the best local anasthesia?
2018
٦١ ‫صفحة‬
Surgery Made Easy

A. Lidocaine
B. Lidocaine with epinephrine
C. Bubivacine

- The agents most commonly used are lidocaine and bupivacaine. Lidocaine has the
advantage of rapid onset, whereas bupivacaine has the advantage of long duration
(average 68 hours). 8 Although bupivacaine can produce irreversible heart block in
high doses, this is rarely an issue with the amounts typically used in the hand. For
pediatric patients, the tolerated dose is 2.5 mg/kg. This can be easily remembered by
noting that when using 0.25% bupivacaine, 1 mL/kg is acceptable dosing.
- A commonly held axiom is that epinephrine is unacceptable to be used in the hand.
Several recent large series have dispelled this myth. Schwartz 


٦٢ ‫صفحة‬
Surgery Made Easy

Plastics, Skin, and Soft Tissues


1. Pt has forearm melanoma with palpable lymph nodes largest one is 3x1 cm in
size, you will do dissection up to which level of lymph nodes?
A. level I
B. level 1&2
C. level 1&2&3
D. excision of affected lymph node

- Axillary node melanoma with no other primary – Tx: complete axillary node dissection
(remove Level I, II, and III nodes – unlike breast CA); primary lesion may have
regressed or the melanoma primary is unpigmented. Fiser

2. Pt c/o abdominal distention, mild pain for months, CT showed retroperitoneal


mass with no mesenteric or aortic LNs, what is the most likely diagnosis?
A. liposarcoma
B. lymphoma

- Fifteen percent of adult soft tissue sarcomas occur in the retroperitoneum. Most
retroperitoneal tumors are malignant, and about one third are soft tissue sarcomas.
The most common sarcomas occurring in the retroperitoneum are liposarcomas,
malignant fibrous histiocytomas, and leiomyosarcomas. Most retroperitoneal
sarcomas are liposarcomas or leiomyosarcomas. In contrast to extremity sarcomas,
local recurrence and intraabdominal spread are frequent patterns of relapse for
retroperitoneal tumors. Schwartz
- Will be lymphoma if there is fever. weight loss and metastatic LNs.

3. Pt with right thigh high grade sarcoma what's is definitive treatment ? ( 1mm
positive margin )?
A. Function Sparing excision with 1 cm safety margin
B. Wide local excision with radiotherapy
C. Chemotherapy only.
D. Chemotherapy followed by excision. 


4. 30 yrs old man, sustained a second degree flame burn involving the anterior part
of the trunk, after he properly resituated with intravenous fluids and given
adequate analgesia which of the following would be the best dressing?
A. Aquacel silver
B. Heavy padded gauze
C. Aerosol plastic spray.
D. Partial thickness skin graft 


- Silver-impregnated dressings such as Acticoat (Smith & Nephew, London, United


Kingdom), Aquacel Ag (Convatec, Princeton, NJ), and Mepilex Ag (Mölnlycke Health
Care US, LLC, Norcross, GA) are increasingly being used for donor sites, skin grafts,
and partial-thickness burns. These may be more comfortable for the patient, reduce
the number of dressing changes, and shorten hospital length of stay, but they do limit
serial wound examinations. Biologic membranes such as Biobrane (DowHickham,
٦٣ ‫صفحة‬
Surgery Made Easy

Sugarland, TX) provide a prolonged barrier under which wounds may heal. Because
of the occlusive nature of these dressings, these are typically used only on fresh
superficial partial-thickness burns that are clearly not contaminated. Schwartz

5. Female, immigrant with Hx of burn in the thigh 12 years ago, she is c/o itchiness
on/off over the old burn with non- healing, elevated edge ulcer, next step ?
A. Biopsy from ulcer edge
B. Excision

- Dx cutaneous SCC, There is an increased risk of cutaneous SCC in chronically


inflamed skin resulting from scars, burns, chronic ulcers, sinus tracts, or inflammatory
dermatoses such as lichen sclerosus et atrophicus.
- Biopsy is necessary to confirm the diagnosis of SCC. uptodate

6. Patient with H/O burn 30 y back, showed anterior leg mid shaft ulcer about 12
cm, Dx?
A. Squamous cell carcinoma
B. Kaposi sarcoma

- See explanation in Q 5

7. A 45 y male with 2x2.5 cm painless swelling in medial thigh, you suspect ?


liposarcoma, what is your biopsy?
A. FNA
B. longitudinal incisional biopsy
C. transvers incisional biopsy
D. transvers excisional biopsy

- Contemporary guidelines recommend incisional biopsy when core needle biopsy


cannot produce adequate tissue for diagnosis or when findings on core needle biopsy
are nondiagnostic.
- The biopsy incision should be oriented longitudinally along the extremity to allow a
subsequent wide local excision that encompasses the biopsy site, scar, and tumor en
bloc.

8. Post lipoma excision showed sarcoma what is next?


A. Re excision with Negative margin

- If an unexpected positive margin is found on pathologic examination of the resection


specimen, re-excision should be performed. In patients with a positive margin,
particularly in patients with macroscopic residual disease, local control is unlikely
even with the addition of postoperative radiation therapy

9. Bed sore with full thickness skin and central slough, stage?
A. III
B. II
C. I

٦٤ ‫صفحة‬
Surgery Made Easy

- Table 45-15 National pressure ulcer advisory panel staging system in Schwartz
P1880
- CLASSIFICATIoN
- Stage I = Intact skin with nonblanchable redness
- Stage II = Partial-thickness loss of dermis; may present as blister
- Stage III= Full-thickness loss of dermis with visible subcutaneous fat (no deeper
structures exposed)
- Stage IV = Full-thickness loss of dermis with exposed bone, tendon, or muscle
- Unstageable = Full-thickness loss of dermis with ulcer base obscured by eschar
- Stage I and II ulcers are treated conservatively with dressing changes and basic
pressure ulcer prevention strategies as already discussed. Patients with stage III or
IV ulcers should be evaluated for surgery. Schwartz

10. Pt post RTA for 3 days has facial bone #mandibular #with facial nerve injury
mouth incompetence and tongue deviation (facial palsy)treatment is?
A. conservative
B. nerve repair
C. bone fixation
D. Nerve graft

- Figure 45-40. Facial reanimation treatment algorithm. Schwartz P1867

11. The most significant prognostic factor for soft tissue sarcoma is?
A. site
B. size
C. grade
D. type of cells

- Tumor grade is the most important prognostic factor (undifferentiated worse). Fiser


٦٥ ‫صفحة‬
Surgery Made Easy

Surgical oncology
1. Most resistance phase to radiation in cell cycle?
A. S phase
B. M phase
C. G1 phase
D. G2 phase

- The extent of DNA damage from indirectly ionizing radiation is dependent on the
phase of the cell cycle. The most radiation-sensitive phases are G 2 and M, whereas
G 1 and late S phases are less sensitive. Thus irradiation of a population of tumor
cells results in killing of a greater proportion of cells in G 2 and M phases. However,
delivery of radiation in divided doses, a concept referred to as fractionation, allows
the surviving G 1 and S phase cells to progress to more sensitive phases, a process
referred to as reassortment. In contrast to DNA damage after indirectly ionizing
radiation, that after exposure to directly ionizing radiation is less dependent on the
cell-cycle phase. Schwartz

٦٦ ‫صفحة‬
Surgery Made Easy

The breast
1. young female complains of unilateral intermittent spontaneous bloody nipple
discharge?
A. Intraductal papilloma

- Intraductal papillom Most common cause of bloody nipple discharge, Are usually
small, nonpalpable, and close to the nipple, These lesions
are not premalignant → get contrast ductogram to find papilloma, then needle
localization, Tx: subareolar resection of the involved duct and papilloma

2. Lactating pt has painful lump on US hypodence irregular edges?


A. Galactocele
B. Fibroadenoma
C. Fibrocystic Change

- Galactocele:
- Occurs after cessation of lactation secondary to an obstructed lactiferous duct
- Round, well-circumscribed, mobile, tender subareolar mass with milky yellow or
greenish yellow nipple discharge
- Treatment—needle aspiration; excision indicated if cyst cannot be aspirated or cyst
becomes infected


6. 30 yrs old women, 2 from her relative have breast ca, they was diagnosed in
younger age, and she have positive BRCA. what is the best modality for screening?
A. Bilateral mammogram
B. Bilateral US
C. PET mammogram
D. MRI 


- The use of MRI for breast cancer screening is recommended by the ACS for women
with a 20% to 25% or greater lifetime risk using risk assessment tools based mainly
on family history, BRCA mutation carriers, those individuals who have a family
member with a BRCA mutation who have not been tested themselves, individuals
who received radiation to the chest between the ages of 10 to 30 years, and those
individuals with a history of Li-Fraumeni syndrome, Cowden syndrome, or
Bannayan-Riley-Ruvalcaba syndrome or those who have a first-degree relative with
one of these syndromes. MRI is an extremely sensitive screening tool that is not
limited by the density of the breast tissue as mammography is, however, its
specificity is moderate leading to more false-positive events and the increased need
for biopsy. Schwartz

7. Post mastectomy and ALND, can’t elevate her arm and there was widening of the
scapula, the injured nerve is?
A. Long thoracic
B. Thoracodorstal
C. intercostobrachial 


8. Painful fibroadenoma (there was u/s pic) 1 x 2 cm management?


٦٧ ‫صفحة‬
Surgery Made Easy

A. Reassurance
B. FNA
C. Excisional biopsy

- Round, well-circumscribed, rubbery, mobile, nontender mass. Usually solitary but


may be multiple and bilateral

- Ultrasound and biopsy to confirm diagnosis.

9. Steps for assessment of breast?


A. Physical examination
B. investigation
C. pathology

- First History, physical examination then investigation

10. Female without nipple bloody discharge with dilated duct


A. Duct ectasia

- Dx Periductal mastitis (mammary duct ectasia or plasma cell mastitis


- Symptoms: noncyclical mastodynia, erythema, nipple retraction, creamy discharge
from nipple; can have sterile or infected subareolar abscess
- Risk factors – smoking, nipple piercings
- Biopsy – dilated mammary ducts, inspissated secretions, marked periductal
inflammation
- Tx: if typical creamy discharge is present that is not bloody and not associated with
nipple retraction, give antibiotics, reassure, and continue breastfeeding; if not or if it
recurs, need to rule out inflammatory CA (incisional biopsy including the skin).Fiser


11. A 45-year-old female presents with a mobile mass in the breast that is 12 cm in
diameter and increasing in size. Examination reveals a mass with a firm and
rubbery consistency. Mammogram shows a rounded dense mass. Biopsy reveals
a non-epithelial lesion with highly cellular stroma, polychromasia and high
mitotic figures. What is the most appropriate further management?
A. MRM
B. Wedge resection
C. Mastectomy
D. Wide local excision

12. 45 years old female, presents with palpable breast mass over the right upper
outer quadrant for 3 months & has no change in size, physical examination, the
mass feels hard , 2 cm other than that palpable mass, the reminder of the
physical examination unremarkable, no axillary lymph nodes, Mammography is
normal, further management consists of?
A. Follow up after 3 months

B. Core needle biopsy

C. Excision of the mass 


٦٨ ‫صفحة‬
Surgery Made Easy

Thyroid, Parathyroid and Adrenal


glad and other glands
1. 14 yrs old, post partial thyrodiectmoy, histopathology came as 1 cm papillary ca.
what is next?
A. Completion thyrodiectmoy

B. Follow up

- Surgery for papillary and follicular thyroid CA → start with lobectomy

- Indications for total thyroidectomy:

- Tumor > 1 cm
- Extra-thyroidal disease (beyond thyroid capsule, clinically positive nodes,
metastases)

- Multi-centric or bilateral lesions

- Previous XRT

- Indications for MRND:

- Extra-thyroidal disease

- Indications for post-op 131I (6 weeks after surgery, want TSH high for maximum
uptake)

- Tumor > 1 cm
- Extra-thyroidal disease

- Need total thyroidectomy for 131I to be effective


2. Pt for total thyroidectomy, after the dissection of the Rt lobe the surgeon noticed
that the RLN was cut, what is the next appropriate thing to do?
A. Intra op repair and don’t proceed to the other side.
B. Intra op repair and continue to the other side.
C. Nerve simulation
D. Stop the procedure.

3. Solitary thyroid nodule Pt present with neck swelling U/S neck showed Solitary
thyroid nodule 1cm What's your plan?
A. follow up
B. lobectomy
C. Total Thyroidectomy
D. FNA

- 1. Low TSH suggests toxic nodule; obtain radionuclide scan and treat with ablation or
surgery. “Hot” nodules are rarely malignant.
- 2. Normal TSH requires FNA to rule out malignancy.
- 3. FNA results: 70% benign (colloid nodule, thyroiditis), 15% nondiagnostic, 10%
suspicious/indeterminate, 5% malignant Hernias, abdominal wall and soft tissue
tumors

4. 23 yrs old Pt, known to have hypertension with 4.5 cm mass on the lt adrenal, CT
was normal, other blood and urine test were normal what is next?

٦٩ ‫صفحة‬
Surgery Made Easy

A. Repeat CT after 3 months


B. CT guided biopsy
C. repeat invx after 3 months
D. Excision

- Dx Incidentaloma Adrenal lesion found during routine imaging; excludes tumors


discovered for evaluation of hormone hypersecretion or staging of known cancers
- Perform surgical resection for functioning tumors or malignancy.
- Lesions less than 4 cm are followed with yearly CT scan.
- Nonfunctional lesions of 4–6 cm can be managed conservatively or operatively
depending on patient risk factors. mont

5. 24 yrs old female presented with palpitation, weight loss, and hyperactivity.
Physical examination revealed diffuse enlarged thyroid gland mass with
exophthalmos. Bp: 140/80 HR: 110 RR: 22 temp: 36.7 T4: 300 TSH: 0.001

what is the next step in management?
A. Betaa blocker
B. Anti thyroid medication
C. Radioactive iodine ablation
D. Near total thyroidectomy 


- THYROID STORM
- Symptoms: ↑ HR, fever, numbness, irritability, vomiting, diarrhea, high-output cardiac
failure (most common cause of death)
- Tx: β-blockers (first drug to give to reduce peripheral T 4 to T 3 conversion and
decrease the hyperthyroid symptoms), Lugol’s solution (KI, most effective but takes
while to work), cooling blankets, oxygen, glucose

6. 56y old male complains of generalized body ache and depression has following
lab result : with high ca low phosphate normal PTH and TSH dx?
A. Primary hyper-parathyrodism
B. Secondary hyper-parathyrodism
C. Sarcoidosis
D. thyrotoxicosis

- Hypercalcemia also is found in approximately 10% of patients with sarcoidosis


secondary to increased 25-hydroxy vitamin D 1-hydroxylase activity in lymphoid
tissue and pulmonary macrophages, which is not subject to inhibitory feedback
control by serum calcium. Schwartz
- Primary hyperparathyroidism = ↑ PTH , ↑ Ca, ↓ po4
- Secondary hyperparathyroidism = ↑ PTH , ↓ Ca, ↓ po4
- 3rd hyperparathyroidism post transplant = ↑ PTH , normal Ca, normal po4

7. 25 yr old male with incidentally finding of supra renal 3 cm mass in CT scan for
low back pain and high ca++.... Next ?
A. 24 hr urine catecholamine
B. 24 hr urine metanephrine
C. Ct with contrast
D. Dexamethasone suppression test
٧٠ ‫صفحة‬
Surgery Made Easy

E. aldosterone

8. Patient post APR of sigmoid cancer on TPN and bad oral hygiene developed
parotid gland abscess (sialadenitis) , high WBC , he is febrile and by
examination, you found pus coming from Stensen duct, pt already on antibiotics,
what you will do?
A. I and D
B. Antibiotics
C. dealy for flactuation
D. Parotid massage to increase saliva flow and good hygiene

- Sialoadenitis – acute inflammation of a salivary gland related to a stone in the duct;


most calculi near orifice
- 80% of the time affects the submandibular or sublingual glands
- Recurrent sialoadenitis is due to ascending infection from the oral cavity
- Tx: incise duct and remove stone
- Gland excision may eventually be necessary for recurrent disease


9. Post gastrectomy, intubated , has parotid gland infection, organizm?


A. Staph aureus
B. strept B hemolyticus
C. Bacteriotides

- PAROTITIS is Infection of the parotid gland


- most common causative organism Staphylococcus
- associated risk actors are Age older than 65 years, malnutrition, poor oral hygiene,
presence o NG tube, NPO, dehydration
- most common time o occurrence Usually 2 weeks postoperative
- signs are Hot, red, tender parotid gland and increased WBCs
- treatment are Antibiotics, operative drainage as necessary. recall

10. Polyuria , polydipsia , weakness, high Ca, low ph, normal k DX?
A. hyperparathyroidism
B. mylome
C. vit D deficiency

- Dx Primary hyperparathyroidism = ↑ PTH , ↑ Ca, ↓ po4


- Can get hyperchloremic metabolic acidosis
- Stones—nephrolithiasis in 20%–25%; nephrocalcinosis—less than 5%
- Chronic hypercalcemia impairs concentration, leading to polyuria, polydipsia, and
nocturia.


11. 45 yr male male with RT neck swelling 4.5cm FNA show papillary lesion (FNA
show follicular neoplasm with negative lymph node , Bethesda IV, The treatment
is?
A. Total thyroidectomy
B. RT lobectomy
C. subtotal thyroidectomy
٧١ ‫صفحة‬
Surgery Made Easy

D. Chemo & XRT

12. 48 years old female with thyroid problem, presented with central thyroid mass,
pathology revealed colloid, CXR picture showed mass in the mediastinum , the
next step?
A. CT neck
B. US neck
C. FNA

13. A 65 years old female with a history of hashimoto thyroiditis presented with
sudden rapid enlarging neck mass with signs of inflammation in short period,
most likely represent?
A. Lymphoma
B. MTC
C. papillary CA
D. Sub acute thyroiditis 


14. Patient with hypertension , neck swelling & blood test showed increase Ca level ,
so this patient has?
A. MEN II a
B. MEN I
C. MEN II b
D. MENIII

٧٢ ‫صفحة‬
Surgery Made Easy

Hernia
1. Post umbilical hernia repair D#1, with small swelling over the wound, negative
cough impulse , no redness no erythema or discharge?
A. hematoma
B. Seroma
C. abscess
D. recurrent hernia

- Seromas are loculated fluid collections that most commonly develop within 1 week of
synthetic mesh repairs. Large hernia sac remnants may fill with physiologic fluid and
mimic seromas. Patients often mistake seromas for early recurrence. Treatment
consists of reassurance and warm compression to accelerate resolution. To avoid
secondary infection, seromas should not be aspirated unless they cause discomfort or
they restrict activity for a prolonged time. schwartz
- Immediate complications include bleeding (which may be due to accidental damage
to the inferior epigastric or iliac vessels) and urinary retention which may require
catheterisation. Occasional overenthusiastic infusion of local anaesthetic may lead to
femoral nerve blockade, the patient being unable to move a leg. This usually resolves
over 12 hours but is alarming. bailey and loves
- Over the next week, seroma formation and wound infection may occur. Seroma is due
to an excessive inflammatory response to sutures or mesh and cannot be prevented.
In most cases the fluid resolves spontaneously but may require aspiration. After
laparoscopic surgery, a seroma may be misdiagnosed as an early recurrence. Wound
infection is not uncommon. bailey and loves

2. During repair of femoral hernia through an inguinal approach you try to reduce
the bowel but are unsuccessful. The most appropriate next step is:
A. Pull on the bowel until it gives
B. Resect the bowel through a femoral exploration
C. Divide cooper ligament
D. Divide the lacunir ligament

- A Cooper ligament repair (McVay) using the inguinal canal approach allows reduction
of the hernia sac with visualization from above the inguinal ligament and closure of
the femoral space. Occasionally, it may be necessary to divide the inguinal ligament
to reduce the hernia. The repair can be performed with or without mesh.

4. Pt Present to ER Post inguinal hernia repair 10 days back testicular swelling,


Cause?
A. testicular artery thrombi
B. pampiniform venous plexus thrombosis
C. internal ring narrowing
D. external ring narrowing

- Ischemic orchitis is likely caused by injury to the pampiniform plexus and not to the
testicular artery. It usually manifests within 1 week of inguinal hernia repair as an
enlarged, indurated, and painful testis, and it is almost certainly self-limited. It occurs
٧٣ ‫صفحة‬
Surgery Made Easy

in <1% of primary hernia repairs; however, this figure is larger for recurrent inguinal
hernia repairs. Schwartz

5. male pt. S/P gastric bypass 10 years back with central abdominal pain and
vomiting, according to the pt the pain relieve with leg flexion, cause?
A. internal hernia
B. Obturator hernia
C. femoral hernia

- Howship-Romberg sign is pain along medial thigh exacerbated by abduction,


extension, and medial rotation of the thigh

6. Pt with symptoms and signs of Strangulated hernia?


A. urgent OR

7. Post inguinal hernia repair 10 days back c/o swelling, redness & tenderness?
A. Seroma
B. Hematoma
C. Recurrent hernia

- see Q 1

8. Pt booked for elective inguinal hernia repair, what ABX should be given pre op ?
A. No need.
B. 1 dose cefazloin.
C. 2 dose cefazolin.
D. 3 dose cefazolin.

- The risk for surgical site infection is estimated to be 1% to 2% after open inguinal
hernia repair and slightly less with laparoscopic repairs. These are clean operations,
and the risk for infection is primarily influenced by associated patient diseases. Most
would agree that there is no need to use routine antimicrobial prophylaxis for hernia
repair. Prospective randomized clinical trials have not supported the routine use of
perioperative antimicrobial prophylaxis for inguinal hernia repair for patients at low
risk for infection. Patients who have significant underlying disease, as reflected by an
American Society of Anesthesiology score of 3 or more, receive perioperative
antimicrobial prophylaxis with cefazolin, 1 to 2 g, given intravenously 30 to 60
minutes before the incision. Clindamycin, 600 mg intravenously, can be used for
patients allergic to penicillin. Only a single dose of antibiotic is necessary. Sabiston

9. While you are trying to reduce bowel with inguinal hernia there is minimal
perforation with minimal with minimal spillage what will use for irrigation of the
wound?
A. Normal saline
B. NS with 0.01% metro
C. NS with 0.01% drug

٧٤ ‫صفحة‬
Surgery Made Easy

10. While performing a laparoscopic inguinal hernia repair the surgeon finds and
artery in the extrpertonial connective tissue. Running vertically just medial to the
bowel as the bowel passes through the abdominal wall. Which artery is this?
A. Inferior epigastric artery
B. Deep circumflex iliac artery
C. Superior epigastric artery
D. Femoral artery

- image.

11. DM Pt post ventral hernia repair one week later develop wound infection, and
after some time pt, presented with small swelling in the previous scar with
positive bowel sound. what is the reason for this:
A. DM
B. Wound infection
C. Inadequate dissection

- see comment in Q1

12. Post lap inguinal hernia repair, with lateral thigh numbness what is the nerve
injured? 2018
A. ilioinguinal nerve
B. genitofemoral nerve
C. Later Femoral cutaneous nerve

- Chronic pain syndromes include local nerve entrapment, meralgia paresthetica, and
osteitis pubis. At greatest risk of entrapment are the ilioinguinal and iliohypogastric
nerves in anterior repairs and the genitofemoral and lateral femoral cutaneous nerves
in laparoscopic repairs. Schwartz
- Lateral femoral cutaneous (L2-L3) = Sensory—anterior and lateral thigh.
- Genitofemoral nerve (L1-L2) = Sensory—scrotum/labia majora, medial thigh
- Iliohypogastric (T12-L1) = Sensory—suprapubic region
- ilioinguinal nerve = pubic symphysis and the superomedial aspect of the femoral
triangle receive a sensory supply from this nerve

13. 52 yrs old obese and diabetic women underwent ventral insicinal hernia mesh
repair 6 months back. Presents with pain, swelling and discharge at site of
surgery. Examination reveals erythema, pus discharge and partially exposed
mesh. which of the following is the appropriate management?
A. Mesh removal
B. Wound debridment
C. Incision and draginge
D. Broad spectrum antibiotics 


14. 50 yrs old woman presents to the ER with a 2 days history of a tender medial
thigh mass. The pain increases with internal rotation of the thigh

which of the following is the most likely diagnosis?
A. Inguinal hernia
B. Femoral hernia

٧٥ ‫صفحة‬
Surgery Made Easy

C. Obturator hernia
D. Sigelian hernia 


- See Q5

15. 56 yrs old woman with liver failure and ascites has an enlarged umbilical hernia
the ascites is refractory to diuretic therapy. What is the most appropriate
management of his hernia?
A. Open repair with waterproof mesh
B. High volume paracentesis immediately before repair
C. Deferring hernia repair until correction of the ascites with TIPS
D. Repair of the hernia and use of abd binder after operation

- Umbilical hernia in patients with ascites Maximize diuresis +/– Transhepatic


intrajugular portcaval shunt (TIPS) When to operate? Incarceration, strangulation,
bowel obstruction, or overlying skin break In these conditions, you should drain
ascites then close the defect (water-tight) primarly to avoid ascites fluid from leaking;
then consider applying absorbable mesh (biological). puzzles
- Patients with refractory ascites may be candidates for transjugular intrahepatic
portocaval shunt (TIPS) or eventual liver transplantation. Umbilical hernia repair
should be deferred until after the ascites is controlled. Schwartz

16. Post laparoscopic inguinal hernia repair a 29 years old male developed
paresthesia over lateral thigh, to which side was the tachers placed and caused
this symptoms:
A. below and lateral iliopubic tract
B. below and medial iliopubic tract
C. above and lateral iliopubic tract
D. above and medial iliopubic tract 


- staples were not placed below the level of the iliopubic tract to avoid neuralgia involving
the lateral cutaneous nerve of the thigh or the femoral branch of the genitofemoral nerve
and they were not placed in the area of the triangle of Doom. 

17. 30 years old male S\P inguinal hernia repair presented with paresthesia over the
lateral scrotum and medial thigh the most likely causative nerve is?
A. Iliohypogastric Nerve
B. Ilioinguinal Nerve
C. Genitofemoral Nerve
D. Lateral Femoral Cutaneous Nerve

- See Q12 above


- Genitofemoral nerve (L1-L2) = Sensory—scrotum/labia majora, medial thigh


18.Pt with back pain and nausea, With lumber swelling CT showed hernia above
iliac crest What is the cause of the hernia?
A. Obesity
B. Chronic cough
C. Traumatic hernia

٧٦ ‫صفحة‬
Surgery Made Easy

D. Primary hernia 


- Dx Petit hernia.
- lumbar hernia 1- Petit hernia (inferior lumbar) triangle 2- Grynfeltt–Lesshaft (superior
lumbar) triangle.

٧٧ ‫صفحة‬
Surgery Made Easy

Head and Neck


1. Pt with neck mass in lat side biopsy show squamous cell ca. No Hx of
malignancy. Best tool to looking for primary?
A. CT
B. MRI
C. PET scan
D. Panendoscopy

- Epidermoid CA (SCCA variant) found in cervical node without known primary →


- 1st – panendoscopy to look for primary; get random biopsies
- 2nd – CT scan
- 3rd – still cannot find primary → ipsilateral MRND, ipsilateral tonsillectomy (most
common location for occult head/neck tumor), bilateral XRT (nodal region and
potential primary sites). FISER


2. Post lymph node dissection presented with paresthesia peri-auricular, and


paresthesia around lip, what is the never injured ?
A. Temporal of facial nerve

B. Mandibular branch of Trigeminal

C. Spinal accessory

D. Greater auricular

3. Post neck lymph node excision. Pt. presented with drop of mouth angel which
facial nerve branch has been injured?
A. Buccal branch

B. Zygomatic branch

C. Marginal Mandibular

D. Cervical

4. Post cervical lymph node excisional biopsy in posterior neck triangle pt cannot
elevate his shoulder, (affecting shoulder shrug), what N. injured?
A. spinal accessory nerve ---- trapezius muscle
B. supraclavcular nerve

- "Shoulder syndrome" describing the clinical syndrome of pain and shoulder


dysfunction that is associated with SAN injury.
- SAN that innervates the upper trapezius muscle 

5. A young male fall from height presented with paresthesia over the index finger,
decease jerk reflex of triceps... CT showed cervical disc prolapsed in which
level?
A. C4-5

B. C5-6

C. C6-7

D. C8-T1

- Common and testable exam findings:


- C5 radiculopathy
٧٨ ‫صفحة‬
Surgery Made Easy

- deltoid and biceps weakness 


- diminished biceps reflex 
- C6 radiculopathy 
- Brachioradialis and wrist extension weakness
- Diminished brachioradialis reflex
- Paresthesias in thumb 
- C7 radiculopathy
- Triceps and wrist flexion weakness 
- Diminished triceps reflex
- Paresthesia in the index,middle, ring 
- C8 radiculopathy 
- Wakness to distal phalanx flexion of middle and index finger (difficulty with fine motor
function)
- Presthesias in little finger

٧٩ ‫صفحة‬
Surgery Made Easy

Upper GIT and Lower GIT


Oesophagus
1. Case scenario about pt has history of dysphasia upper endoscopy done show
lesion biopsy taken the result revealed high grade dysplasia what is the next ?
A. Esophagectomy
B. Follow up after 6 month
C. Nissine funduplcation

- High grade dysplasia (HGD) Barrett’s esophagus (considered carcinoma in situ)


- Needs to be confirmed by 2 experienced pathologists
- Options:
- Esophagectomy or Endoscopic surveillance (3-month intervals; 4 quadrant Bx’s at
1-cm intervals for entire length of HGD and Bx of any suspicious areas)
- Some patients with HGD never develop CA (basis for option 2)
- Cell types other than Barrett’s (eg squamous cell CIS) – follow above as well
- Uncomplicated Barrett’s (eg non-HGD) can be treated like GERD (ie PPI or Nissen)
– Tx will decrease esophagitis and further metaplasia
- Need annual follow-up EGD surveillance for lifetime, even after Nissen
- Barrett’s CA risk is not reversed with PPI or fundoplication Fiser
- Biopsy specimens should be reviewed by a pathologist with expertise in the field. If
low-grade dysplasia is confirmed, biopsy specimens should be repeated after 12
weeks of high-dose acid suppression therapy. If high-grade dysplasia or intramucosal
cancer is evident on more than one biopsy specimen, then treatment is escalated.
Treatment options include endoscopic mucosal resection, endoscopic ablation of the
BE, or esophageal resection. Schwartz 


2. Case scenario about pt has history of dysphasia upper endoscopy done confirm
the pt has achalasia + mass notice what is next ?
A. Heller Myotomy
B. Balloon Dilation
C. Esophagectomy
D. Botox injection

3. 40 yo male, MF Had sudden onset of GI bleeding for the first time O/E normal v/
S , LFT normal , Endoscopy showed paraesophageal bleeding What is the dc?
A. Schistosome
B. Alcoholic cirrhosis
C. Wilson dz
D. Bud chairri sx

- Heavy infestations are more likely to produce hepatic disease. Eventually, severe
periportal fibrosis in a characteristic pipestem pattern (Symmers pipestem fibrosis)
may occur. Although hepatocellular function is spared, periportal fibrosis can lead to
portal hypertension with the usual potential sequelae, including splenomegaly,
ascites, esophageal variceal bleeding, and development of portosystemic collaterals.
٨٠ ‫صفحة‬
Surgery Made Easy

Through these collaterals (or directly from the inferior vena cava in the case of
bladder wall schistosomiasis), eggs can reach the pulmonary circulation. The
resulting pulmonary granulomatosis and fibrosis can lead to pulmonary
hypertension and frank cor pulmonale with a high mortality rate. Medscape

4. Case scenario about pt with history of GERD he underwent for upper endoscopy
found lesion The treatment of choice for Barrett’s esophagus with severe
dysplasia is?
A. Follow-up endoscopy and biopsy
B. Esophagectomy
C. Nissen fundoplication
D. Proton pump inhibitors

- Elective esophagectomy in patients with Barrett’s disease with high-grade dysplasia


- Some suggest this is the preferred mode of resection for middle and lower-third
esophageal tumors but can be considered in all patients with esophageal cancer.
mont

5. Pt Jaundiced with vomiting of fresh blood, EGD showed bleeding esophageal


varices that failed to be controlled endoscopically, what is the next step in
management?
A. TIPS
B. Sclerotherapy
C. Sengstaken tube
D. Variceal banding

- Primary management of esophageal varices is with pharmacologic therapy and


endoscopic banding A-Pharmacologic therapy: vasopressin infusion, beta-blockers B-
Endoscopy (banding, sclerotherapy) Success rate at controlling bleeding: 90%
- If the bleeding cannot be controlled endoscopically, balloon tamponade is indicated:
A- Sengstaken-Blakemore tube B-Four-port Minnesota tube . Mont


6. Pt post EGD and four biopsies was taken from esophageal incisors, after 6 hours
when the pt, started oral feeding he was having sternum pain and sternum
tenderness, what is the best test to confirm the diagnosis?
A. Chest x ray
B. Esophagram
C. CT
D. Endoscopy 


- Perforation of the esophagus constitutes a true emergency. It most commonly occurs


following diagnostic or therapeutic procedures. Schwartz
- Cervical esophagus near cricopharyngeus muscle most common site. Fiser
- Barium esophagram is the standard for diagnosis.
- The key to management and patient survival is early recognition with timely diagnosis
and therapy.

7. 52 YO man with alcoholism & known cirrhosis comes into the emergency with
acute hematemesis. Bleeding esophageal varices are found during upper GI

٨١ ‫صفحة‬
Surgery Made Easy

endoscopy. Which of the following is the most likely to be effective treatment for
him?
A. Balloon tamponade of the esophagus
B. Banding
C. Epinephrine
D. Endoscopy sclerotherapy

- Primary management of esophageal varices is with pharmacologic therapy and


endoscopic banding A-Pharmacologic therapy: vasopressin infusion, beta-blockers B-
Endoscopy (banding, sclerotherapy) Success rate at controlling bleeding: 90%
- If the bleeding cannot be controlled endoscopically, balloon tamponade is indicated:
A- Sengstaken-Blakemore tube B-Four-port Minnesota tube . Mont

8. A 35 y Patient has weight loss 4 kg in 6 month, with difficulty of swallowing


liquids and manometry showed increase pressure in middle third of esophagus?
part 14
A. Achalasia
B. Esophagus cancer
C. Diffuse Esophagus spasm
D. plummer venson syndrome

- Dx Diffuse Esophageal Spasm.


- chest pain, dysphagia; aggravated by reflux, cold liquids, and periods of emotional
stress. Associated with other gastrointestinal (GI) complaints
- Diagnosis:
- Esophagram (“corkscrew esophagus”)
- Manometry – frequent strong non-peristaltic unorganized contractions, LES
relaxes normally.
- Tx: calcium channel blocker, trazodone; Heller myotomy if those fail (myotomy
of upper and lower esophagus; right thoracotomy)
- Surgery usually less effective for diffuse esophageal spasm than for achalasia

9. Patient comes in with recurrent aspiration, weight loss and progressive


dysphagia. Started 5 months ago, Barium swallow showed dilated esophagus
and manometry showed incomplete relaxation of the LES, Diagnosis?
A. Esophageal Web
B. Stricture
C. Esophageal Cancer
D. Achalasia
E. Nutcracker esophagus

- Dx Achalasia
- Aperistalsis and incomplete relaxation of the LES.
- Etiology can be idiopathic or infectious (i.e., Trypanosoma cruzi) degeneration of
Auerbach plexus; degeneration can lead to hypertension of LES, failure to relax, and
progressive loss of peristalsis.
- Symptoms include dysphagia, regurgitation, weight loss, retrosternal chest pain,
nocturnal coughing, recurrent pulmonary infections. Progressing dysphagia beginning

٨٢ ‫صفحة‬
Surgery Made Easy

with liquids, then solids. Patient should eat carefully at meals and consume copious
amounts of water.
- Diagnosis: Barium swallow demonstrates “bird’s beak” narrowing of distal
esophagus with proximal dilation; esophageal manometry is the gold standard for
diagnosis—it will show aperistalsis and incomplete relaxation of the LES. LES resting
pressure is often hypertensive, but it can be normotensive.
- Tx: balloon dilatation of LES → effective in 80%; nitrates, calcium channel blocker
- If medical Tx and dilation fail → Heller myotomy (left
thoracotomy, myotomy of lower esophagus only; also need partial Nissen
fundoplication)
- Can get esophageal CA late (squamous cell most common)


Stomach

1. 57 year old male patient presented to complaining from abdominal pain ,EGD
showing gastric sub mucosal swelling with, of 10 *5 cm sized mass, most likely
diagnosis?
A. GIST "gastrointestinal stromal tumors "
B. Pancreatic ectopic tissue
C. Dieulfoy malformation
D. Submucosal lymphatic proliferation

- Most common benign gastric neoplasm, although can be malignant


- Symptoms: usually asymptomatic, but obstruction and bleeding can occur
- Hypoechoic on ultrasound; smooth edges
- Dx: biopsy – are C-KIT–positive
- Considered malignant if > 5 cm or > 5 mitoses/50 HPF (high-powered field)
- Tx: resection with 1-cm margins; no nodal dissection
- Chemotherapy with imatinib (Gleevec; tyrosine kinase inhibitor) if malignant. Fiser
- Endoscopy may be useful to further characterize the lesion if a gastric mass is
identified. Both GISTs and leiomyomas may appear as a submucosal mass with
smooth margins, with a normal overlying mucosa, and bulging into the gastric lumen.
uptodate

2. 3 cm sessile gastric polyp in the greater curvature?


A. wedge resection
B. total gastrectomy
C. partial gastrectomy.
D. Endoscopic resection

- In patients with sessile polyps, endoscopic mucosal resection may be needed to


provide an accurate histological assessment and achieve complete resection

3. Patient presented with gastric mass invading mucosa, submucoa and extending
to serosa ,, no distant mets . With positive 2 large lymph nodes ( largest one ) 3
cm. What is the stage?
A. T1<N2<M0

٨٣ ‫صفحة‬
Surgery Made Easy

B. T2<N2<M0
C. T3<N1<M0
D. T4<N2<M0

- American Joint Committee on Cancer TNM Classification


1. T = Primary tumor
a. Tis—carcinoma in situ—no invasion of lamina propria
b. T1a—Tumor invades lamina propria or muscularis mucosa.
c. T1b—Tumor invades submucosa.
d. T2—Tumor invades muscularis propria or subserosa.
e. T3—penetrates serosa
f. T4—Tumor invades adjacent structures.
2. N = Regional lymph nodes involved
a. N0—No regional lymph nodes involved.
b. N1—metastasis in 1–2 regional lymph nodes
c. N2—metastasis in 3–6 regional lymph nodes
d. N3a—metastasis in 7–15 regional lymph nodes
e. N3b—more than 16 regional nodes involved
3. M = Distant metastasis
a. M0—no distant metastasis
b. M1—distant metastasis


4. Pt epileptic presented with upper GI bleeding and after resuscitation and NGT
insertion ... bleeding stop, most likely the diagnosis is?
A. Watermelon gastric
B. Dieulafoy's lesion
C. phytobezores
D. Mallory Weiss syndrome

- This is a linear mucosal tear at the gastroesophageal junction secondary to violent


retching or vomiting
- Bleeding often stops spontaneously
- CAUSES : MC Alcohol use, Hiatal hernia, old Age
- Precipitating factors include vomiting, straining or lifting, coughing, seizures, blunt
abdominal injury, nasogastric tube placement, and gastroscopy. uptodate

5. Isolated grade 1 Gastric Varices with splenic artery thrombosis?


A. splenectomy

- If the patient has splenic vein thrombosis and left-sided (sinistral) or segmental portal
hypertension, splenectomy is quite effective in controlling bleeding from isolated
gastric varices. Schwartz

6. Post sleeve c/o vomiting and diarrhea after meal?


A. dumping syndrome

- Occurs from rapid entering of carbohydrates into the small bowel


- 2 phases:

٨٤ ‫صفحة‬
Surgery Made Easy

- Hyperosmotic load causes fluid shift into bowel (hypotension, diarrhea, dizziness)


- Hypoglycemia from reactive ↑ in insulin and ↓ in glucose (2nd phase rarely occurs)

7. old patient, had abdominal pain with nausea and vomiting. Upon exploration,
found to have 3 cm Perforated 1st duodenal ulcer. (Didn’t mention bleeding)
What is the management?
A. Omental patch.
B. Omental patch with vagotomy.
C. gastrodeudostomy
D. PPI.

- Surgical options—laparoscopic comparable with open approach


- Unstable patient or significant peritoneal contamination—omental patch
- Stable patient (duodenal ulcer)—patch with PPI therapy, highly selective vagotomy
(HSV), or vagotomy and drainage
- Stable patient (gastric ulcer)—removal of ulcer by excision and primary repair of
stomach; antrectomy is ideal, or patch with PPI and H. pyloritherapy


8. Female patient known to have PUD. On upper endoscopy, there was ulcer in the
stomach 3*4 non-healing. Biopsy showed (chronic non-specific inflammation )
what is the management?
A. H2 Pylori Eradication with PPI
B. repeat endoscopy biopsy
C. partial Gastrectomy

- algorithm :

9. 50 yrs old female post laparoscopic Roux-en-Y bypass done seven months back,
presented to the ER complainin of vomiting and abdominal colic. Abdomin
examination revealed tachycardia and tender epigastric area. Plain abdominal X
ray showed mild abdominal distention. 

what is the most likely diagnosis?
A. Marginal ulcer.
B. Anastomotic stenosis
C. Internal hernia
D. Vitamin deficiency

- Patients who develop a bowel obstruction after laparoscopic gastric bypass require
surgical and not conservative therapy due to the high incidence of internal hernias
and the potential for bowel infarction.
- Antecolic position of the Roux limb is associated with a lower incidence of internal
hernias leading to obstruction in most series with short-term follow-up. However,
reports with longer follow-up suggest later internal hernia incidence may increase with
an antecolic approach. Schwartz

10. Pt. underwent gastric bypass or gastrectomy post day 2 pt developed leak, What
is the most likely cause?
A. Technically
٨٥ ‫صفحة‬
Surgery Made Easy

B. Blood supply compromise


C. Infection
D. Thermal injury

- Leak after Roux-en-Y gastric bypass :

- Ischemia – most common cause of leak

- Signs of leak – ↑ RR, ↑ HR, fever, elevated WBCs; often do not have abdominal pain

- Tx: early leak (not contained) → emergent re-op; late leak (weeks out from surgery, likely


contained) → percutaneous drain, antibiotics. Fiser 


- Gastric leaks can be due to mechanical or ischemic causes. According to Baker et stapler
misfiring, or direct tissular injury are categorized as “mechanical-tissular” causes and usually
appear within 2 d of surgery (early), compared to the “ischemic causes” that usually appear
on day 5-6 post operatively (post op) (intermediate). PMC4194572

11. Post gastric bypass came with abdominal pain and distension , the most likely
cause is ?
A. Stenosis

- not complete Q

- read about bypass complication

12. Patient with gastric ca. post billroth II complain abdominal discomfort relieved
after bilious vomiting, what is diagnosis?
A. Afferent loop syndrome

B. Efferent loop syndrome

C. Dumping syndrome

D. alkaline reflux gastritis

E. Internal hernia

- Afferent-loop obstruction
- With Billroth II or Roux-en-Y; caused by mechanical obstruction of afferent limb
- Symptoms: RUQ pain; nonbilious vomiting, pain relieved with bilious emesis
- Risk factors – long afferent limb with Billroth II or Roux-en-Y
- Dx: CT scan – shows dilated afferent limb
- Tx: balloon dilation may be possible
- Surgical option: re-anastomosis with shorter (40-cm) afferent limb to relieve
obstruction


13. Post bypass gastric has muscular pain , peripheral numbness (parasthesia) ,
what is the cause ?
A. Vit B12 deficiency
B. Folate
C. Vit B1 deficiency
D. iron

- Postoperative nutritional complications after LRYGB include a 66% incidence of iron


deficiency, a 5% incidence of iron deficiency anemia, a 50% incidence of vitamin B 12
deficiency, 100 and an at least 15% incidence of vitamin D deficiency, which usually is
present preoperatively. Schwartz

- B12 deficiency"cobalamin" = Megaloblastic anemia, peripheral neuropathy

- Iron deficiency = Microcytic anemia

٨٦ ‫صفحة‬
Surgery Made Easy

14. Long scenario, An UGIE showed an ulcerated bleeding lemon sized mass, most
likely diagnosis?
A. GIST
B. Pancreatic ectopic tissue
C. Doulfoy malformation
D. Submucosal lymphatic proliferation

- GIST:
- Most common benign gastric neoplasm, although can be malignant
- Symptoms: usually asymptomatic, but obstruction and bleeding can occur
- Hypoechoic on ultrasound; smooth edges
- Dx: biopsy – are C-KIT–positive
- Considered malignant if > 5 cm or > 5 mitoses/50 HPF (high-powered field)
- Tx: resection with 1-cm margins; no nodal dissection
- Chemotherapy with imatinib (Gleevec; tyrosine kinase inhibitor) if malignant


15. A 68 years old woman presented with a history of weight loss & nausea, but
denies vomiting or early satiety work up ultimately leads to biopsy proven low
grade B cell mucosa associated lymphoid tissue lymphoma confined to the
stomach, what is the most appropriate treatment?
A. Antibiotics
B. Chemoradiotherapy
C. PPIs
D. Subtotal gastrectomy followed by radiation therapy

- Dx gastric lymphoma grade two. If low-grade lymphoma persists after H. pylori


eradication, radiation should be considered for disease clinically confined to the
stomach (stage I), while chemotherapy with or without radiation is used for more
advanced lesions
- See Figure 26-58. Algorithm for the treatment of gastric lymphoma Schwartz P1085


16. pt with chronic gastric ulcer . about 2 cm histopathology show GIST, TTT?
A. Wedge resection
B. Distal gasteroectomy
C. Total gasterectomy

- According to the tumor size and mitotic count, A the risk of aggressive behavior was
classified into four groups. Very low risk was defined as <2 cm and <5/50 HPF (high-
power field) and low risk was defined as 2 to 5cm and <5/50 cm. Intermediate risk
was defined as <5 cm and 6 to 10/50 HPF or 5 to 10cm and >5/50 HPF. And high risk
was defined as >5 cm and >5/50HPF, >10cm with any mitotic rate or >10/50/ HPF
with any size. It was known that the risk of recurrence differs by the primary site of
the tumor. Recently, new classification based on tumor location, size, and mitotic rate
has been used to evaluate the risk of recurrence and metastasis.
- Wedge resection with clear margins is adequate surgical treatment. True invasion of
adjacent structures by the primary tumor is evidence of malignancy. If safe, en bloc
resection of involved surrounding organs is appropriate to remove all tumor when the
primary is large and invasive.
- Figure 26-59. Algorithm for the treatment of gastrointestinal stromal tumor. Schwartz
P1986

٨٧ ‫صفحة‬
Surgery Made Easy

17. Patient post gastrectomy one day ago has fever Increase HR tachypnea, Next
step ? 2018
A. OR
B. Gastrografin
C. CT

- Dx leak. Tachycardia, tachypnea, fever, and oliguria are the most common
symptoms. Schwartz
- CT scan with oral contrast is the best diagnostic test to evaluate for leak or
obstruction after RYGBP.
- Upper gastrointestinal series with Gastrografin are routinely performed by most
bariatric surgeons before further diet progression in order to detect any subclinical
leaks. washington
- Gastrografin used to diagnosis small intestinal obstruction

Small intestine

1. Case scenario of abdominal pain with absent peritoneal sings Dx mesenteric


venous thrombosis treatment ?
A. Tissue plasminogen
B. Anticoagulation with heparin
C. Thrombectomy
D. Laparotomy

- The treatment of established mesenteric venous thrombosis (acute and subacute) is


predominantly conservative, consisting of systemic anticoagulation to minimize
extension of thrombus, bowel rest, and careful, serial observation for any signs of
clinical deterioration

2. Treatment of intussusception in 50 YO female is?


A. Lap reduction
B. Lap resection
C. Colonoscopy
D. Barium enema

- Adult intussusceptions are far less common and usually have a distinct pathologic
lead point, which can be malignant in up to one half of cases.
- Surgical reductions without resection have been successfully reported in these
patients. Schwartz

3. pt with negative ASCA and positive pANCA ?


A. UC
B. Crown's disease

- ANCA positivity is seen in 60 to 80 percent of patients with ulcerative colitis and the
related disorder, primary sclerosing cholangitis

٨٨ ‫صفحة‬
Surgery Made Easy

- ASCA+ Present in 60% of patients with CD compared with 5% of patients with UC .


Less than 5% in normal population.

4. Young Patient c/o of central abdominal pain, bloating, post prandial vomiting
and weight loss, the pain is relieved by knee-chest position?
A. SMA syndrome

- Superior mesenteric artery syndrome, characterized by compression of the third


portion of the duodenum by the superior mesenteric artery as it crosses over this
portion of the duodenum. This condition should be considered in young asthenic
individuals who have chronic symptoms suggestive of proximal small bowel
obstruction. Schwartz
- Symptoms may be relieved when patient is lying prone, in the left lateral decubitus, or
in a knee-chest position. These positions remove tension from the mesentery and
superior mesenteric artery opening the space between the superior mesenteric artery
and aorta. uptodate

5. Old man, known to have AF, presented with severe central abd pain what is the
most likely dx?
A. Mesenteric occlusion
B. Perforated viscous
C. Acute pancreatitis

6. Post sleeve c/o vomiting and diarrhea after meal?


A. dumping syndrome .

7. 34 yrs old man presented to ER with history of frequent episode of vomiting


fresh blood. Upper gastrointestinal endoscopy showed actively bleeding in the
st
posterior wall of the 1 part of duodenum which was controlled by heat probe.

Which of the following arteries is the most likely source of this bleeding ?
A. Splenic artery
B. Gastrodudenal
C. Superior mesenteric
D. Right gastric 


- Dx duodenum ulcer. Usually in 1st part of the duodenum; usually anterior


- Anterior ulcers perforate
- Posterior ulcers bleed from gastroduodenal artery. Fiser 


8. Pt with crohn’s disease has enterocutaneous fistula, this will result in?
A. Decreased total body weight
B. Decreased lean body weight

9. Pt during laparotomy, the finding was pneumatosis intestinalis, what is the


management?
A. Close without intervention
B. Resection

- Depend on why laparotomy ?


٨٩ ‫صفحة‬
Surgery Made Easy

- No treatment is necessary unless one of the very rare complications supervenes,


such as rectal bleeding, cyst-induced volvulus, or tension pneumoperitoneum.
Prognosis in most patients is that of the underlying disease. The important point is to
recognize that pneumatosis intestinalis is a benign cause of pneumoperitoneum.
Treatment should be directed at the underlying cause of the pneumatosis, and
surgical intervention should be predicated on the clinical course of the patient.

10. Young girl post RTA with repeated vomiting and obstipation and dehydrated with
coiled spring sign appearance in CT scan, treatment?
A. NPO ,NGT and observation
B. dudeno-jujenostomy
C. resection

- Dx Paraduodenal hematomas on CT scan (or missed on initial CT scan)


- Can present with high small bowel obstruction (SBO) 12–72 hours after injury
- UGI study will show “stacked coins” or “coiled spring” appearance (make sure
there is no extravasation of contrast)
- Tx: conservative (NGT and TPN) - cures 90% over 2–3 weeks (hematoma is
reabsorbed)


11. Patient post small bowel carcinoid tumor resection present with diarrhea and
liver mets, what is the best diagnostic test for recurrence ?
A. Octreotide scan

- An annual OctreoScan is recommended unless more frequent evaluation is indicated.


An OctreoScan may be performed earlier if evidence of disease progression is seen.

12. parastomal hernia was found Incidentally on CT abdomen done for a patient post
colectomy and iliostomy..the next step will be?
A. Relocation
B. Observation
C. Exploration

- Parastomal hernia is the most common late complication of a colostomy and requires
repair if it is symptomatic

Appendix
1. 47 years old gentleman underwent laparoscopic appendectomy for clinical
appendicitis . Histopathology revealed acute appendicitis and carcinoid tumor in
tip of appendix measuring 1.7 cm , not invading mesoappendix , no enlarged
lymph nodes . the most appropriate next treatment ?
A. Nothing to do ((observation ))
B. right hemicolectomy
C. chemotherapy
D. radiotherapy

٩٠ ‫صفحة‬
Surgery Made Easy

- The majority of carcinoids are located in the tip of the appendix. Malignant potential is
related to size, with tumors <1 cm rarely resulting in extension outside of the
appendix or adjacent to the mass. The mean tumor size for carcinoids is 2.5 cm. 133
Carcinoid tumors usually present with localized disease (64%). Treatment for tumors
≤1 cm is appendectomy. For tumors larger than 1 to 2 cm located at the base,
involving the mesentery, or with lymph node metastases, right hemicolectomy is
indicated. Despite these recommendations, surveillance, epidemiology, and end
results data indicate that proper surgery for carcinoids is not performed at least 28%
of the time. Schwartz

2. Pt with pic of acute appendicitis, after performing appendectomy found to have


wide soft diverticulum neck, 50 cm from ileocecal valve, what is the
management?
A. Dirvituclectomy
B. Close the abdomen

- The management of incidentally found (asymptomatic) Meckel’s diverticula is


controversial. Until recently, most authors recommended against prophylactic
removal of asymptomatic Meckel’s diverticula, given the low lifetime incidence of
complications.

3. 26 yrs old man presented to ER with history of RIF pain since 5 days. Ass. With
fever and anorexia. Physical examination revealed marked tenderness and
guarding in the RIF with exaggerated bowel sound. Abdominal CT showed large
abscess in the RIF pointing to the area of McBurney's point with oral contrast
reaching the splenic flexure. HR: 96 beats/min, BP: 110/70 RR: 17 temp: 38.7
WBC: 18.2 which of the following is the appropriate intervention?
A. Laparoscopic exploration of the abdomen
B. Imaging guided drainage of the abscess
C. Drainage of the abscess through gridiron incision
D. Abdominal exploration through lower midline incision 


- The treatment of appendicitis is surgical and requires removal of the inflamed


appendix, except in cases of appendiceal perforation, in which treatment can vary
(drainage and antibiotic treatment). mont

4. 23 yrs old man, presented to ER, complaining of RIF pain since 7 days with
anorexia and diarrhea. Physical examination revealed tender mass in the Right
lower guarding, CT revealed appendiceal mass in the RIF with deep abscess in
the pelvis 2*2 cm which of the following is the appropriate management?
A. Non surgical management
B. Transrectal abscess drainage
C. Percutaneous abscess drainage
D. Appendectomy with abscess drainage 


5. 18 yrs old female, presented with right sided lower abdominal pain. Which
started in the center of abd. 3 days ago. Physical examination revealed a tender
guarded right lower abdomen. CT scan of the abd. Showed a mass of matted
bowel loops around an enlarged appendix. temp: 37.8 HR: 88 RR 18 BP: 120/80
WBC: 14.1 what is the most appropriate management?
٩١ ‫صفحة‬
Surgery Made Easy

A. Non surgical management


B. Limited right hemi-colectomy
C. Open appendectomy
D. Lap. Appendectomy

6. pt presented with history of bleeding per rectum for 3 months, proctoscopy


confirm second degree pile. What is the next step ?
A. Colonoscopy
B. Hemorrhoidectomy
C. Pile banding

- Rubber Band Ligation. Persistent bleeding from first-, second-, and selected third-
degree hemorrhoids may be treated by rubber band ligation. Schwartz

7. Obese patient with a RIF pain for 5 days associated with n/V WHAT IS NEXT
STEP?
A. CT abdomen
B. US abdomen
C. open surgery
D. Diagnostic Laparoscopy

- Overall, CT scan is more sensitive and specific than ultrasonography in diagnosing


appendicitis. Schwartz

Colon

1. Case scenario about pt with family history of colonic cancer which gene will
lead to colon cancer?
A. APC
B. K-ras
C. P53
D. HMLH1

- APC (adenomatous polyposis coli) tumor suppressor gene: first studied in familial
adenomatous polyposis. Inactivation/mutation is present in 80% of sporadic colorectal
cancers.
- K-ras—proto-oncogene; mutation leads to uncontrolled cell division
- DCC (deleted in colorectal carcinoma)—tumor suppressor gene. This mutation is
present in more than 70% of colorectal cancers.
- p53—gene crucial for initiation of apoptosis. Mutations are present in 75% of
colorectal cancers.

2. Unstable patient, known to have Crohn's with distended?


A. Total colectomy
B. total proctocolectomy

٩٢ ‫صفحة‬
Surgery Made Easy

- Indications for Surgery. Indications for surgery in ulcerative colitis may be emergent
or elective. Emergency surgery is required for patients with massive life-threatening
hemorrhage, toxic megacolon, or fulminant colitis who fail to respond rapidly to
medical therapy.
- Emergent Operation. In a patient with fulminant colitis or toxic megacolon, total
abdominal colectomy with end ileostomy (with or without a mucus fistula), rather than
total proctocolectomy, is recommended. Schwartz

3. 49 years female with Sigmoid adenocarcinoma and 2 family positive, what is for
surveillance?
A. PET-ct for whole body
B. Mammogram
C. MRI brain
D. Trans-vaginal US

- Screening colonoscopy is recommended annually for at-risk patients beginning at


either age 20 to 25 years or 10 years younger than the youngest age at diagnosis in
the family, whichever comes first. 66 Because of the high risk of endometrial
carcinoma, transvaginal ultrasound or endometrial aspiration biopsy is also
recommended annually after age 25 to 35 years. Because there is a 40% risk of
developing a second colon cancer, total colectomy with ileorectal anastomosis is
recommended once adenomas or a colon carcinoma is diagnosed. Annual
proctoscopy is necessary because the risk of developing rectal cancer remains high.
Similarly, prophylactic hysterectomy and bilateral

4. Pt in ICU sigmoidoscopy showed white plaque what is the management?


A. Vancomycin


- Dx Pseudomembranous Colitis " Clostridium difficile Colitis". Symptoms: watery,


green, mucoid diarrhea; pain and cramping
- Can occur up to 3 weeks after antibiotics; increased in postop, elderly, and ICU
patients
- Pseudomembranous colitis is caused by C. difficile, a gram-positive anaerobic
bacillus. C. difficile colitis is extremely common and is the leading cause of
nosocomially acquired diarrhea.
- Key finding – PMN inflammation of mucosa and submucosa, Pseudomembranes,
plaques, and ringlike lesions
- Most common in the distal colon
- Diagnosis of this disease was traditionally made by culturing the organism from the
stool. Detection of one or both toxins (either by cytotoxic assays or by
immunoassays) has proven to be more rapid, sensitive, and specific.
- Management should include immediate cessation of the offending antimicrobial
agent. Patients with mild disease (diarrhea but no fever or abdominal pain) may be
treated as outpatients with a 10-day course of oral metronidazole. Oral vancomycin is
a second-line agent used in patients allergic to metronidazole or in patients with
recurrent disease. More severe diarrhea associated with dehydration and/or fever
and abdominal pain is best treated with bowel rest, intravenous hydration, and oral
metronidazole or vancomycin. Schwartz

٩٣ ‫صفحة‬
Surgery Made Easy

5. Patient on antibiotics and present with diarrhea sigmoidescopy showed yellow


plaque of sigmoid(pseudomembranous colitis), what is the best method of
diagnosis?
A. Stool C/S
B. Colonoscopy
C. PCR
D. Cytotoxic assay

- Cell cytotoxin assay in tissue culture is a highly sensitive and specific test for the
detection of toxin B (rounding effect) and is the gold standard diagnostic test for CDC.
Saibston

6. X-ray abdomen with coffe bean appearance, and scenario of old lady without
peritoneal signs what to do?
A. Endoscopy decompression
B. Sigmoidescopy
C. CT abdomen
D. laparotomy

7. Pt with diverticulitis improved with medical Tx ... came back with second attack 3
months with same pic of non-complicated diverticulitis Rx?
A. Antibiotics
B. Colonoscopy
C. Resection
D. CT scan

- Most patients with uncomplicated diverticulitis will recover without surgery, and 50%
to 70% will have no further episodes. 49 It has long been believed that the risk of
complications increases with recurrent disease. For this reason, elective sigmoid
colectomy has often been recommended after the second episode of diverticulitis,
especially if the patient has required hospitalization. Resection has often been
recommended after the first episode in very young patients and is often
recommended after the first episode of complicated diverticulitis. These general
guidelines have been questioned in recent years, and more recent studies suggest
that the risk of complications and/or need for emergent resection does not increase
with recurrent disease. Schwartz

8. A young male known to have UC present to ER with severe abdominal pain,


distention, bloody diarrhea, stable vitally after resuscitation... Initial
management will be:
A. bowel rest, Steroid, Abx, IVF
B. urgent colectomy
C. decompression with rectal tube 


- Toxic colitis and toxic megacolon:
- Toxic colitis: > 6 bloody stools/d, fever, ↑ HR, drop in Hgb, leukocytosis
- Toxic megacolon: above plus distension, abdominal pain and tenderness
- Initial Tx: NG tube, fluids, steroids, bowel rest, and antibiotics (ciprofloxacin and
Flagyl) will treat 50% adequately; other 50% require surgery

٩٤ ‫صفحة‬
Surgery Made Easy

- Follow clinical response and abdominal radiographs


- Avoid barium enemas, narcotics, anti-diarrheal agents, and anti-cholinergics


9. Pt known to have sigmoidal ca, operated years back, presented with 10 cm


mass in the lower abdomen just below the umbilicus, biopsy showed spindle
cells with... best management will be ?
A. Wide local excision
B. Chemoradiotherapy

- palliative care a better option than surgical resection. PMID: 26781531

10. A 65 years old male presented with 2 days history of abdominal distension,
nausea and obstipation, O\E marked distension with diffuse abdominal
tenderness, no rebound or guarding, AXR Showed uniformal distension of the
colon about colon 6 cm ,CT showed no signs of obstruction& dilated large
bowel, with no transitional zone next step ?
A. Rectal tube
B. Total colectomy with anastomosis
C. Colonoscopy
D. Neostigmine

- OGILVIE’S SYNDROME:
- Pseudoobstruction of colon
- Associated with opiate use; bedridden or older patients; recent surgery, infection, or
trauma
- Get a massively dilated colon, which can perforate
- Treatment—decompression
- Enema
- Colonoscopic decompression if enema unsuccessful or if patient has significant cecal
dilation—40% recurrence
- Intravenous (IV) neostigmine—20% recurrence. Caution in cardiac patients
(bradycardia). Should only be pursued in a monitored setting
- Partial colectomy if perforated, ischemic, or colonoscopy unsuccessful; try to avoid
operation, given that multiple comorbidities are typically present in this patient
population.

11.Patient after endoscopic polypectomy 1.2 cm of adenomatous polyp, when to


do next colonoscopy?
A. 1 year
B. 3 year
C. 5 year
D. 10 year

- Colonoscopy at first detection; then colonoscopy in 3 y If no further polyps,


colonoscopy every 5 y If polyps, colonoscopy every 3 y Annual colonoscopy for >5
adenomas Schwartz table 29-2 P1210

٩٥ ‫صفحة‬
Surgery Made Easy

Rectum

1. An 80-year-old woman comes to K/C of rheumatoid arthritis on NSAID came to


the emergency room with history of per rectum and sigmoid copy reveled the
blood above the sigmoid junction . Which diagnostic test should be performed
next?
A. Angiography
B. Upper endoscopy
C. Colonoscopy
D. Go to OR

- Advantages of colonoscopy compared with other tests for lower GI bleeding include
its potential to precisely localize the site of the bleeding regardless of the etiology or
rate of bleeding. uptodate

2. Smoker patient c/o weight loss, vague abdominal pain, PR has anterior rectal
mass & mucosa intact the most common tumor is?
A. Gastric cancer
B. Esophageal cancer
C. Rectal cancer
D. Pancreatic cancer

3. 20 yrs old male pt. presented with abd pain, no bloody diarrhea, no mucous,
EGD done and it showed: edematous mucosa, and loss of colon haustral what is
the dx?
A. Crohn’s
B. UC
C. Derveticulosis 


- Barium enema has been used to diagnose chronic ulcerative colitis and to determine
the extent of disease. However, In long-standing ulcerative colitis, the colon is
foreshortened and lacks haustral markings (“lead pipe” colon). Because the
inflammation in ulcerative colitis is purely mucosal, strictures are highly uncommon. Any
stricture diagnosed in a patient with ulcerative colitis must be presumed to be malignant
until proven otherwise. Schwartz

Anus
1. Patient presented to ER with anal pain and itching. Examination showed multiple
small anal warts. What is your management?
A. Surgical excision.
B. podophyllin ointment.

- Dx Condylomata acuminata Treatment—excision/cauterization and topical agents


a. Excision/cauterization under anesthesia for extensive involvement. Be mindful of
anal stenosis as a complication with large, treated areas.
b. Podophyllin, cryotherapy, and trichloroacetic acid (TCA) are routine office
treatments. Biopsy should be performed before chemical treatment.

٩٦ ‫صفحة‬
Surgery Made Easy

c. Podofilox, 5-fluorouracil, and imiquimod are home treatments, with variable


frequency of application.
d. A giant condyloma (Buschke-Löwenstein tumor) is often locally invasive, and half
contain SCC. Major morbidity is due to local invasion and recurrence.

2. Pt. presented with multiple auxiliary lesion with multiple fistula, what is the
management?
A. ABx
B. Topical antifungal
C. Excision of skin and subcutaneous tissue

3. Pt with sever anal pain with defecation associated with blood in toilet paper,
what is the condition?
A. Anal fissure

- Dx Anal fissure. Straining bowel movements, constipation


- Caused by a split in the anoderm
- 90% in posterior midline
- Causes pain and bleeding after defecation


4. 30 Years old female presented to ER with sever perianal pain, physical


examination demonstrate indurations but no obvious mass in the
perianal area best management?
A. Stool softeners &sitz baths
B. EUA
C. IV abx
D. MRI pelvis 


5. Pt with Lowe back pain for 3 days o/e there is induration over the cleft,
tenderness and erythema, (case of PNS ) what is the treatment?
A. Excision
B. Control of the infection
C. Hair removal
D. fistulogram 


٩٧ ‫صفحة‬
Surgery Made Easy

Gastrointestinal Bleeding and


acute Abdomen
1. Case scenario for female pt complain for sever central abdominal pain +vomiting
, on examination : stable vital sing , a febrile , there is periumblical tender
swelling tense , no cough impulse , no peritoneal sing what is next?
A. US abdomen
B. CT abdomen
C. Go for OR
D. Upper endoscopy

6. Case scenario for pt complain of LT lower quadrant pain for 4 days, nausea ,
vomiting, OE stable vital , a febrile , abdomen LIF tenderness , WBC 15000
Diagnosis ?
A. Pyelonephritis
B. Diverticulitis
C. Colon CA
D. Inflammatory bowel disease

- The classic presentation in acute pyelonephritis is the triad of fever, costovertebral


angle pain, and nausea and/or vomiting
- Classical triad of Diverticulitis: LLQ pain/ tenderness, fever and leukocytosis so Left
lower quadrant tenderness is the most common physical finding, as most diverticula
occur in the sigmoid colon medscape

7. Middle aged lady with sudden severe lower abdominal pain followed by shock,
the most likely diagnosis is?
A. Rupture ovarian follicle
B. Rupture ectopic pregnancy
C. Acute appendicitis
D. Mesenteric lymphadenitis

8. Young pt k/c of cholelithiasis with generalized abdominal distention and pain x-


ray showed distended small bowel loops with multiple air-fluid levels and
pneumobilia, what is the diagnosis?
A. Vulvolus
B. Intraluminal small bowel obstruction.

9. Patient k/c of gastric or duodenal ulcer, presented with rectal bleeding with (clots)
by sigmoidoscopy, Your initial next step?
A. Colonoscopy
B. NGT insertion

٩٨ ‫صفحة‬
Surgery Made Easy

- A nasogastric tube is inserted as part of the initial management.



The character of the aspirate may help to differentiate between upper and lower GI
source.
- a.Gross blood: likely upper GI source
- b.Bile, but no blood: strongly suggestive of lower GI source
- Clear nonbilious effluent: cannot rule out a duodenal source of bleeding

10.pt with lower GI bleeding, colonoscopy showed Sigmoid angiodysplasia, what


you will do ?
A. Angio-embolization
B. Inject epinephrine
C. Resect segment

- Colonoscopy may occasionally identify a bleeding diverticulum that may then be


treated with epinephrine injection or cautery. schwartz
- Embolization of colonic bleeds carries a higher risk for ischemia with perforation.

11. Patient developed RLQ abdominal pain after lifting heavy object , o/E irreducible
with –ve cough impulse and not changing with contracting the abdominal
muscle , likely diagnosis is ?
A. Rectus sheath hematoma
B. incarcerated inguinal hernia

- Fothergill’s sign is a palpable abdominal mass that remains unchanged with


contraction of the rectus muscles and is classically associated with rectus
hematoma. Schwartz’s

12. 68 yrs old male presented with sudden onset of sever lower abd. Pain and
vomating . The pt. has not passed stool for the last 24 hours. On examination the
patient abdomen is markedly distended and tympanic on percussion. X ray
reveld Omega sign (X-ray showed bent inner tube): Bp: 150/85 HR: 120 RR: 20
temp: 37.4 what is the most appropriate next step in management ?
A. Prokinetic therapy
B. Gastrografin enema
C. Emergency laparotomy
D. Sigmoidescopy 


- Sigmoid volvulus:
- IF sigmoidoscopy and plain films Fail to confirm the diagnosis; “bird’s beak” is
pathognomonic seen on enema contrast study as the contrast comes to a sharp end
- signs of necrotic bowel in colonic volvulus are Free air, pneumatosis (air in bowel
wall).
- signs of strangulation are Discolored or hemorrhagic mucosa on sigmoidoscopy,
bloody fluid in the rectum, rank ulceration or necrosis at the point of the twist,
peritoneal signs, Fever, hypotension, WBCs.
- initial treatment are Nonoperative: I there is no strangulation, sigmoidoscopic
reduction is successful in 85% of cases; enema study will occasionally reduce (5%).
- recurrence after nonoperative reduction of a sigmoid volvulus = 40%!

٩٩ ‫صفحة‬
Surgery Made Easy

- indications for surgery are Emergently if strangulation is suspected or nonoperative


reduction unsuccessful (Hartmann’s procedure); most patients should undergo
resection during same hospitalization of redundant sigmoid afrer successful
nonoperative reduction because o high recurrence rate (40%). recall
- Unless there are obvious signs of gangrene or peritonitis, the initial management of
sigmoid volvulus is resuscitation followed by endoscopic detorsion. Detorsion is
usually most easily accomplished by using a rigid proctoscope. schwartz
- Clinical evidence of gangrene or perforation mandates immediate surgical exploration
without an attempt at endoscopic decompression. Schwartz

13. X-ray abdomen with coffee bean appearance, and scenario of old lady without
peritoneal signs what to do ?
A. Endoscopy decompression
B. Sigmoidescopy
C. CT abdomen
D. laparotomy

- Dx Sigmoid volvulus:
- Abdominal x-ray – bent inner tube sign or coffee bean appearance; Gastrografin
enema may show bird’s beak sign (tapered colon)
- Unless there are obvious signs of gangrene or peritonitis, the initial management of
sigmoid volvulus is resuscitation followed by endoscopic detorsion. Detorsion is
usually most easily accomplished by using a rigid proctoscope. Schwartz

14. Middle aged lady with sudden severe lower abdominal pain followed by shock ?
A. Rupture ovarian follicle
B. Rupture ectopic pregnancy
C. Acute appendicitis
D. mesenteric lymphadenitis 


15. female pt with anemia and RLQ non tender mass ?


A. CT
B. colonoscopy

16. pt with massive lower GI bleeding , NGT clear, transfused 4 units PRBCS , pt is
stable, proctoscope : fresh blood with clots, what is the next :
A. Angio
B. RBC scan
C. CT
D. Colonoscopy

- Anoscopy/sigmoidoscopy/colonoscopy
- Overall diagnostic accuracy is up to 97%.
- Lack of adequate bowel prep in the acute setting may render these tests
inconclusive.

15. An 80-year-old woman comes to the emergency room with a 3-day history of
moderate bleeding per rectum and has a massive bloody bowel movement upon

١٠٠ ‫صفحة‬
Surgery Made Easy

presentation. Resituated well, a nasogastric tube aspirate returns bile and no


blood. Which diagnostic test should be performed next?
A. Angiography
B. Capsule endoscopy
C. Colonoscopy
D. Tagged red blood cell scan

17.stable Pt with epigastric pain for 6 hours and high amylase and WBC CT showed
intrapertonieal air but no contrast extravasation what is next? 2018
A. Laparotomy.
B. Gastrografin study
C. Diagnostic laparoscopy
D. Conservative management

- Patients undergoing nonoperative therapy should be closely monitored for signs


suggestive of peritonitis, the development of which would mandate urgent surgery. As
stated before, the administration of hypertonic water-soluble contrast agents, such as
Gastrografin used in upper gastrointestinal (GI) and small bowel follow-through
examinations, causes a shift of fluid into the intestinal lumen, thereby increasing the
pressure gradient across the site of obstruction. Schwartz

18. pt with sudden epigastric pain started 8H ago labs high amylase and WBC
Abdomen tender all over with sluggish bowel sound, what is next investigation?
2018
A. Chest Erect x ray
B. CT abdomen
C. US

19. Patient with sever Lower GI bleeding, NGT done and showed greenish collection,
colonoscopy not completed because of massive active bleeding with colts in the
colon, what is next?
A. Upper GI endoscopy
B. Tagged RBCS
C. Angiogram

- Figure 28-29. Diagnostic and management algorithm for obscure gastrointestinal (GI)
bleeding. Schwartz P1169

١٠١ ‫صفحة‬
Surgery Made Easy

Hepatobiliary
Liver

1. 71 years old lady K/C of inoperable Cholangiocarcinoma post palliative stenting


present to ER with history of nausea vomiting, RT upper quadrant pain with
fever. Labs WBC 20, LFT: TBili 25, direct 16. CXR elevated Rt hemi diaphragm
what is your diagnosis?
A. liver abscess
B. pyogenic cholangitis
C. chlangiohepatitis

2. Hepatitis C virus with liver mass and high alpha fetoprotein?


A. HCC

- The incidence is rising due, in large part, to the hepatitis C virus (HCV) epidemic.
- Elevated serum α-fetoprotein (AFP) occurs in 75%

3. Pt with RUQ pain CT done and it showed homogenous mass with central
necrosis what is the DX?
A. FNH
B. Hemangioma
C. HCC

- Focal nodular hyperplasia:


- Has central stellate scar (diagnostic)
- No malignancy risk; very unlikely to rupture
- Dx: abdominal CT; has Kupffer cells, so will take up sulfur colloid on liver scan
- Tx: conservative therapy (no resection)


4. Patient with liver mass upon aspiration the content was like "anchovy sauce "
Whats the treatment ? 2018
A. Metronidazole

- This material has been likened to anchovy paste or chocolate sauce. Amebic abscesses
are the most common type of liver abscesses worldwide.
- Metronidazole 750 mg three times a day for 7 to 10 days is the treatment of choice and
is successful in 95% of cases. Defervescence usually occurs in 3 to 5 days, but the time
necessary for the abscess to resolve depends on the initial size at presentation and
varies from 30 to 300 days. Schwartz

5. Pt came back from holiday with RUQ abdominal pain and tensmus on ex
enlarged liver( classic case of liver abscess symptoms and signs ) , next?
A. U/S abdomen
B. Serology for HBV,HCV
C. WBC
D. Blood C/S

١٠٢ ‫صفحة‬
Surgery Made Easy

- Dx Amebic:
- ↑ LFTs; ↑ in right lobe of liver, usually single
- Primary infection occurs in the colon → amebic colitis
- Risk factors – travel to Mexico, ETOH; fecal–oral transmission
- Positive serology for Entamoeba histolytica – 90% have infection
- Symptoms: fever, chills, RUQ pain, ↑ WBCs, jaundice, hepatomegaly
- Reaches liver via portal vein
- Cultures of abscess often sterile → protozoa exist only in peripheral rim
- Can usually diagnose based on CT characteristics
- Tx: Flagyl; aspiration only if refractory; surgery only if free rupture


34 year old female on OCP, presented RUQ pain after trauma, she is unstable
hemodynamic and signs of peritonitis and drop Hb, on operating room you found
rapture mass this is most likely a?
A. Liver adenoma
B. Liver hemangioma
C. FNH

- Hepatic adenoma
- Women, steroid use, OCPs
- 80% are symptomatic; 50% risk of significant bleeding (rupture)
- Can become malignant (5%)
- More common in right lobe
- Symptoms: pain, ↑ LFTs, ↓ BP (from rupture), palpable mass
- Dx: no Kupffer cells in adenomas, thus no uptake on sulfur colloid scan (cold)
- Tx:
- Asymptomatic and < 4 cm – stop OCPs; if regression, no further therapy is needed; if no
regression, patient needs resection of the tumor
- Symptomatic or > 4 cm – tumor resection for bleeding and malignancy risk; embolization
if multiple and unresectable. Fiser


Gallbladder

1. case scenario of symptom and signs of acute cholangitis , ERCP done and pus
coming from ampulla what is the best Ax ?
A. cefazolin
B. clindamycin
C. gentamycin
D. piperacillin tazobactam

- Dx cholangitis, Pip-Taz is the ideal choice (linezolide for VRE) → good bile
bioavailability. puzzles

2- 25 Y Patient admitted with RHC pain febrile WBC 16000, jaundice, ERCP done and
it was not complicated and pus drained from ampulla of vater, 6 hours patient
collapsed and become hypertensive, tachycardia and tachypnic, diagnosis?
A. Septic shock
B. PE
١٠٣ ‫صفحة‬
Surgery Made Easy

C. MI
D. Duodenal injury with peritonitis

- Perforation is one of the most feared complications of endoscopic retrograde


cholangiopancreatography (ERCP).
- The mean time to diagnosis was 24(+/-13) hours
- Four types of perforation complicating endoscopic retrograde
cholangiopancreatography (ERCP) have been recognized.
- Type I: Free bowel wall perforation
- Type II: Retroperitoneal duodenal perforation secondary to periampullary injury
- Type III: Perforation of the pancreatic or bile duct
- Type IV: Retroperitoneal air alone
- Patients with undetected leaks can present hours after the procedure with pain, fever,
and leukocytosis. Pneumomediastinum and subcutaneous emphysema can occur .
Pneumothorax and gas in the portal system have also been rarely described.
uptodate
- sepsis " hypotension"

3. Case scenario for pt with cholecystitis post laparoscopic cholecystectomy pt


bring bile from the sub hepatic drain site ERCP was done revealed there is leak
from cystic duct what is the next?
A. Endoscopic stenting (ERCP)
B. Percutaneous Trans hepatic drain
C. Re exploration
D. Another sub hepatic drain

- Cystic duct leaks can usually be managed with percutaneous drainage of intra-
abdominal fluid collections followed by an endoscopic biliary stenting. Schwartz

4. You are performing a laparoscopic cholecystectomy on a 25-year-old woman


with acute cholecystitis. The gall bladder is edematous and inflamed. And the
tissue planes are difficult to discern. After clipping you decided to convert to
open, and you noticed what you believed was the cystic duct, is the hepatic duct
& 2 clips founded in proximal hepatic duct that divided and one in the distal
hepatic duct. What operative procedure should be performed to repair this
injury?
A. Hepaticojejunostomy
B. hepaticoduodenostomy
C. Primary repair
D. Stent

- Limited injuries to common bile or common hepatic ducts that are recognized at the
time of surgery can be repaired by T-tube drainage of the common bile duct. Primary
repair of the bile duct should be avoided because of the high rates of breakdown or
stricture formation. In cases of more significant damage to the duct, a
hepaticojejunostomy will be required.

١٠٤ ‫صفحة‬
Surgery Made Easy

5. Scenario about pt with biliary pancreatitis, after 3 days of resuscitation and


hydration his labs results normalized, what is the most appropriate next step in
management ?
A. early lap cholecystectomy

- Acute gallstone pancreatitis is best managed by either early cholecystectomy (within


48–72 hours of presentation) or delayed cholecystectomy (after 72 hours but during
the same hospitalization) to allow the pancreatitis to resolve.

6. Pt has RUQ pain radiating to the scapula, nausea and vom- iting, U/S shows small
gall bladder stones & 8 mm polyp, what is the management?
A. Lap chole.
B. observation

- Polypoid lesions of the gallbladder are associated with increased risk of cancer,
particularly in polyps >10 mm. The calcified “porcelain” gallbladder is associated with
>20% incidence of gallbladder carcinoma. Schwartz

7. Pt with symptoms & signs of biliary colic (no cholecystitis or OJ) what is the
cause of the pain in this pt?
A. contraction of bladder
B. CCK release
C. Cystic stone
D. CBD stone

- The pain develops when a stone obstructs the cystic duct, resulting in a progressive
increase of tension in the gallbladder wall. Schwartz

8. Post lap chole. with cystic duct leak?


A. Lap correction
B. Endoscopic stenting
C. Open correction

- Cystic duct leaks can usually be managed with percutaneous drainage of intra-
abdominal fluid collections followed by an endoscopic biliary stenting. schwartz

9. Old man was admitted in the hospital complain of right upper quadrant pain one
day post admission the abdomen was tender, Ultrasound showed dilated
gallbladder with 2 mm gallbladder wall no gallstones in normal CBD no
Pericholecystic fluid?
A. open cholecystectomy
B. laparoscopic cholecystectomy
C. percutaneous cholecystectomy
D. IVF and rest

10. Post lap chole c/o abdominal pain?


A. US

11. DM pt with RUQ pain, with emphysema in GB wall on us what abx?


١٠٥ ‫صفحة‬
Surgery Made Easy

A. Tazocin
B. meropenum 


12. 37 yrs old women presented to emergency department with a history of right
upper quadrant abdominal pain since 48 hours. With nausea and vomiting.
Physical examination revealed marked tenderness and guarding in the right
upper quadrant. Abdominal US demonstrated pericholycystic fluids, think
gallbladder wall with positive sonogrophic Murphy's sign. which of the following
is the recommended treatment?
A. Conservative treatment and interval cholecystectomy
B. Insertion of precautious cholecyststomy tube
C. Emergency laparoscopic cholecystectomy
D. Emergency open cholecystecomty 


- Cholecystectomy is the definitive treatment for acute cholecystitis. In the past, the
timing of cholecystectomy has been a matter of debate. Early cholecystectomy
performed within 2 to 3 days of the illness is preferred over interval or delayed
cholecystectomy that is performed 6 to 10 weeks after initial medical treatment and
recuperation. Several studies have shown that unless the patient is unfit for surgery,
early cholecystectomy should be recommended, as it offers the patient a definitive
solution in one hospital admission, quicker recovery times, and an earlier return to
work. Schwartz

13. Old patient with intestinal obstruction, the x-ray showed picture of gallstone
ileus, gas in biliary tree, management is?
A. ERCP
B. Exploratory laparotomy
C. Conservative management 


- Abdominal x-ray: occasionally reveals radiopaque gallstone in the bowel; 40% of


patients show air in the biliary tract"Pneumobilia", small bowel distention, and air
Fluid levels secondary to ileus
- Bouveret syndrome—gallstone impaction in pylorus or duodenum resulting in
symptoms of gastric outlet obstruction
- Treatment involves laparotomy with removal of the stone via a
small enterotomy proximal to the point of obstruction.
- Resection is only necessary if perforation or ischemia
- The cholecystenteric fistula is left alone because many close spontaneously, and
recurrence rate is only 5%. mont

14. PT has RUQ pain, no fever, no jaundice -vr murphy sign on chest x-ray has RT
diaphragm elevated, he has all labs normal, what is most likely the diagnosis?
A. Mucocele
B. Empyema

15. Pt post Lap chole. for I wk developed fever , jaundice , RUQ pain what is the next
step ?
A. Ct scan abdomen
B. ERCP
١٠٦ ‫صفحة‬
Surgery Made Easy

C. U/S
D. MRI

16. 45 years old male came with hx of 1 day RUQ pain, dark urine , jaundice ,
vomiting and fever ,what is the most likely organism ?
A. E.coli
B. Stept Staph
C. Bacteroid

- Most common organisms in cholecystitis – E. coli (#1), Klebsiella, Enterococcus

17. Pt with perforated Gallbladder and spillage septic .. MC organism ?


A. KEIBSILLA
B. Pseudomonas
C. c. diff
D. Yersinia

18. A 42 years old female diabetic presented to ER with nausea, vomiting &
abdominal pain, T.38c and WBC 17000, RUQ US is negative for gall stone but
suggestive of air within the lumen of the gall bladder. The most appropriate
initial abx would be?
A. Ampicillin
B. Piperacillin- tazobactam
C. Meropenem
D. Clindamycin 


19. You perform an uncomplicated laparoscopic cholecystectomy on 50 years old


women for symptomatic cholelithiasis, one week later, the surgical pathologist
calls you say that the specimen contains adenocarcinoma of the gall bladder
wall limited to muscular layer & cystic duct margin is negative, what should you
recommend as the treatment plan?
A. No further treatment is needed
B. Wide local excision of liver around the gallbladder fossa
C. Extended right hepatectomy
D. Hepaticojejunostomy 


20. pt with lap cholecystectomy show completely transacted CBD?


A. Primary repair
B. Stent
C. HepaticoDuodenostomy
D. choledochojejunostomy

- Intraoperative CBD injury – if < 50% the circumference of the common bile duct,
can probably perform primary repair; in all other cases, will likely need
hepaticojejunostomy (or choledochojejunostomy); do not try to attach to duodenum –
won’t reach. Fiser
- Major bile duct injuries such as transection of the common hepatic or common bile
duct are best managed at the time of injury. In many of these major injuries, the bile
duct has not only been transected, but a variable length of the duct removed. This
١٠٧ ‫صفحة‬
Surgery Made Easy

injury usually requires a biliary enteric anastomosis with a jejunal loop. Either an end-
to-side Roux-en-Y choledochojejunostomy or, more commonly, a Roux-en-Y
hepaticojejunostomy should be performed. Schwartz

21. Patient k/c of gallstones has Biliary pancreatitis. 3 days after management his
clinical and laps improve. Next step? 2018
A. proceed with cholecystectomy
B. ERCP

- An ultrasonogram of the biliary tree in patients with pancreatitis is essential. If


gallstones are present and the pancreatitis is severe, an ERC with sphincterotomy
and stone extraction may abort the episode of pancreatitis. Once the pancreatitis has
subsided, the gallbladder should be removed during the same admission. When
gallstones are present and the pancreatitis is mild and self-limited, the stone has
probably passed. For these patients, a cholecystectomy and an intraoperative
cholangiogram or a preoperative ERC is indicated. Schwartz

Pancreas

1. 70 years old gentleman, presenting with vague abdominal pain for the last 3
months associated with yellow discoloration of the sclera and the skin and dark
urine. Examination showed yellow sclera and skin. CT showed pancreatic head
mass with biliary dilatation and metastatic lesion in liver segment 2. Patient
underwent endoscopic stenting. What is the most appropriate management?
A. Conventional pancreaticoduodenectomy
B. Radiotherapy
C. Whipple procedure with resection of liver segment II ?
D. Endoscopic US guided Biopsy from the pancreas

- For patients with a high likelihood of unresectability that has not been confirmed
preoperatively, staging laparoscopy permits examination of the liver and peritoneal
surfaces, as well as biopsy of any suspicious areas. If metastatic tumor is found,
laparotomy may be avoided.

2. A 35 years old man has been in ICU with sever pancreatitis, ventilator
dependence and pneumonia for 2 weeks, he is on TPN, INR 2, APPT & platelets
are normal, what of the following is most likely etiology?
A. DIC
B. Vitamin K deficiency
C. Hemophilia
D. Liver failure

3. Case scenario for pt k/c of FAP post total colectomy he develop periampulary
duodenal polyp sessile what is next?
E. Endoscopic resection
F. Nothing and follow up
G. Whipple resection
H. Segment resection
١٠٨ ‫صفحة‬
Surgery Made Easy

- A standard (not pylorus-sparing) Whipple is the procedure of choice in FAP patients


with periampullary lesions. Schwartz

4. Around 50 yrs old, with progressive jaundice, and pain, with palpable GB, what is
the most likely Dx:
A. GB ca.
B. Klatskin tumor
C. Pancreatic CA.

- Most common presentation symptoms/signs: pain, weight loss, jaundice and


Courvoisier sign (palpable, nontender gallbladder on examination). mont

5. Pt with jaundice he has distal CBD obstruction on cholangiography, CBD 12mm ,


fever, what is the clinical sign will be in his examination ?
A. palpable mass
B. palpable gall bladder
C. Shrinked liver
D. Enlarged pancreas

- Most common presentation symptoms/signs: pain, weight loss, jaundice and


Courvoisier sign (palpable, nontender gallbladder on examination). mont

6. Pt after attack of pancreatitis his CT scan show pseudocyst?


A. PTC
B. Laparotomy and drainage
C. Internal drainage

- Dx pseudocyst
- Many pseudocysts will resolve spontaneously without complications, but further
intervention may be required if the pseudocyst:
- Enlarged—pseudocysts greater than 6 cm are more likely to cause symptoms and
less likely to resolve spontaneously.
- Ideally treated 4–6 weeks after appearance so that a thick, fibrous wall can mature
around the cavity and a drainage procedure (DP) is then performed:
- Pseudocysts are treated only if symptomatic or associated with a complication
(infection, obstruction, bleeding), otherwise they are left alone.
- First modality of treatment is endoscopic drainage, either transgastric, transduodenal,
or transpapillary.
- Surgical drainage is the next best option and involves internal drainage
(cystogastrostomy, cystojejunostomy, or cystoduodenostomy) or percutaneous
external drainage (for infected pseudocysts or those with immature walls, reserved
for patients who cannot tolerate the endoscopic or surgical drainage).

7. A 55 years old female undergoes pancreaticoduodenectomy for


adenocarcinoma of the pancreases, post op day 7 she complains of
increasing abdominal pain, nausea & vomiting the appropriate next step in
management is ?
A. CT abdomen
١٠٩ ‫صفحة‬
Surgery Made Easy

B. US abdomen
C. IV erythromycin
D. MRI

8. 49 year old male with alcoholic acute pancreatitis, The most important
prognostic factor in this patient is?
A. age
B. hypercalcemia
C. hypoxemia

- Enzymes may precipitate out with calcium, resulting in ductal obstruction, and
continued enzyme release results in ductal hypertension. mont

9. What is the most important diagnostic modality in making surgical decision


regarding pancreatic cancer?
A. ERCP
B. CT abdomen
C. MRI
D. Ca 19.9 level

10. Pt has epigastric pain and n/v with Hx of sever pancreatitis 5 weeks ago with CT
finding of collection ~8 cm at lesser sac , vitals stable with mild leukocytosis
dx ? 2018
A. Pseudocyst
B. Abscess

- most common complication of pancreatitis (10% of AP, 20%–40% of chronic


pancreatitis)
- Frequently found in the lesser sac, posterior to the stomach. MONT


Spleen

1. Case scenario for pt SCA complain of LT upper abdominal pain + fever , stable
vital , O/E : LHC tenderness , CT picture show splenic abscess unilocaoular what
is Treatment?
A. Gastric aspiration
B. Upper endoscopy
C. Percutenous drainage
D. Splenectomy

- As SCD patients are at increased risk of splenic abscess, infracted splenic


parenchyma is highly susceptible to abscess formation. Fever, left upper abdominal
pain and splenic enlargement are the most common clinical manifestations. CT
scanning of abdomen has simplified the diagnostic dilemma. Medical therapy is
indicated in a few patients and percutaneous drainage could be selectively attempted.

١١٠ ‫صفحة‬
Surgery Made Easy

Splenectomy remains the treatment of choice of splenic abscess in sickle cell disease
patients.

2. 35 year old pregnant lady diagnosed to have 3 cm distal splenic aneurysm. What
is the best management option?
A. Resection of aneurysm and splenectomy
B. aneurysm ligation
C. conservative management
D. embolization

- Splenic Artery Aneurysm:


- Rupture of the aneurysm occurs usually during the 3rd trimester due to increased flow
to splenic artery (compressed aorta) and the wall is weaker
- Any aneurysm of 2 cm or larger → because rupture is associated with > 80%
maternal and fetal mortality .
- Indications for treatment include presence of symptoms, pregnancy, intention to
become pregnant, and presence of pseudoaneurysms associated with inflammatory
processes. Aneurysm resection or ligation alone is acceptable for amenable lesions in
the mid-splenic artery, but distal lesions in close proximity to the splenic hilum should
be treated with concomitant splenectomy. schwartz
- Indications for intervention include size ≥2 cm, females of child-bearing age who may
become pregnant and inflammatory pseudoaneurysms. Management depends on the
location of the aneurysm during the course of the splenic artery. Proximal and middle
third aneurysms may be excluded by proximal and distal ligation of the artery. Splenic
perfusion persists via collateralization from the short gastric vessels. For more distal
lesions proximal ligation with splenectomy is required. Alternatives treatments include
endovascular approaches with transcatheter embolization. Washington

3. Pt. with abd pain and CT done it showed patent IMA, SMA, and splenic vein
occlusion ( not visualized ) what is the management?
A. Spleenctomy
B. Portocaval shunt
C. Side to side shunt 


- One of the complication of splenic vein thrombosis is Gastric varices so Spleenctomy.

4. Pt with ITP his platelet is 20000 he is for splenectomy as u r preparing him when
to transfuse?
A. 10 units plt / pre op
B. After incision
C. Transfuse after ligation of the splenic artery
D. Transfuse 5 pre and 5 post

- in splenectomy due to Thrombocytopenic patients preferably should not undergo


transfusion before the day of surgery and ideally not before the intraoperative ligation
of the splenic artery. Schwartz

١١١ ‫صفحة‬
Surgery Made Easy

5. Blunt abdominal trauma and hematoma in the gastro-splenic ligament and


spleen is surrounded by blood, most likely affected artery?
A. Short gastric
B. Lt gastric
C. Lt gastro epi
D. Splenic a.

- The gastrosplenic ligament contains the short gastric vessels; the remaining
ligaments are avascular.Schwartz 


6. 10 days post splenectomy, 24 years old repeated vomiting associated with


abdominal distension and LUQ tenderness bowel sound normal plus 104 b/min,
BP: 90/60, temp. 37.5, RR: 24 b/min, HB: 12, WBC: 8, PH: 6, TBIL: 2.4, ALB: 4,
most likely diagnosis?
A. acute gastric dilatation
B. Mucosal gastric injury.
C. Paralytic illus
D. Slipped ligation
E. Colonic injury 


١١٢ ‫صفحة‬
Surgery Made Easy

Research, Ethics &


Professionalism and patient
safety
1. Old female with breast ca scheduled for mastectomy with chemo therapy, but the
pt’s son told you not to tell her the ttt plan.what you will do?
A. Check her mental capacity for making decisions.
B. Ask for another relative.
C.Discuss the opinion with the pt 


2. 13 years old boy with perforated appendix, while preparing the pt for surgery
which discussed with pt and his father, the pt agree for surgery but the father
refused to sign the consent and want to use herbal med?
A. Let him sign lama
B. Take the consent from the pt as he agreed already.
C. Ask for ethical committee and medico legal team
D. admit the pt and try to convince his father. 


3. During lap chole. You injured the CBD and told the pt post op what does this
mean?
A. Disclosing an apparent complication
B. Disclosing complication
C. Disclosing error
D. Disclosing an apparent error 


4. Pt with renal failure he's on the list for cadaveric renal transplant waiting for 5yrs.
his doctor offered a living donor renal transplant but pt was reluctant to agree
due to ethnical reasons, the doctor trying to talk and convince him using...?
A. non. Of donor
B. mal<benefice of recipient 


5. Calculate relative risk for two group A) 12.9 B) 3.9


A. 3.3
B. 9
C. 16.8
D. 50.4

- RR= A/B.
- if table 2X2 Want you to calculate ( epidemiological measures of association) : RR =
(a/a+b) /( c /c+d)

6. A diagram of surgical resident and non<surgical resident followed for 4 years


outcome studied: depression. Type of study?
A. cohort

١١٣ ‫صفحة‬
Surgery Made Easy

B. series case
C. control study ( retrospective )

- Cohort studies typically observe large groups of individuals, recording their exposure
to certain risk factors to find clues as to the possible causes of disease.

7. Colon ca pt and non<colon ca and eating habits. What type of study?


A. cohort ( protrospective )
B. control study
C. section study

- Case-control studies are often used to identify factors that may contribute to a
medical condition by comparing subjects who have that condition/disease (the
"cases") with patients who do not have the condition/disease but are otherwise
similar (the "controls")

8. Study on drugs, drug (A) is causing more decrease in ca level than drug (b)
concluded of author ?
A. patient based evidence
B. disease based evidence
C. evidence based medicine

9. Definition of autonomy?
A. provide health care equal for all pt
B. pt can make the decision for his treatment plan
C. patient has the right to choose his own therapy


- Goals for ethical decision making in elderly surgical patient:


- Acknowledge medical futility; physicians are not required to provide life-sustaining
treatment that is deemed medically futile.
- Clarify patients’ wishes regarding life-sustaining therapies in accordance with
Patient Self-Determination Act (i.e., advance directives, living wills, do not intubate/
resuscitate orders).
- Respect patient autonomy: right to accept and refuse treatment despite
consequences of decision.
- Ensure mental competency before establishing autonomy.
- May appoint surrogate decision maker in case of incapacitation. Schwartz

10. Meningitis pt refuse isolation?


A. speak health authority.

11. Ovarian Ca you want to start chemotherapy for her but she need to speak to
other doctor out side hospital main while you will do?
A. transfer her to another specialist
B. give her contact number for another specialist
C. let her sign DAMA
D. talk with social worker to convince her

١١٤ ‫صفحة‬
Surgery Made Easy

12. Pyelonephritis pt the family physician gave him treatment and discharge, next
day came with complication and family are complaining?
A. explain to them, that was medical errors
B. discuss with family and the physician or clinical judgment.

13. A couples wants to get In vitro fertilisation baby and asked the physician for
advice about sex selection?
A. Parents have the right of sec selection
B. It can only be done by they physician
C. Not allowed on sharia
D. Decrease sex linked disea

14. Sample collected for saudi males 400 ppl with congestive heart failure with
different variables ?
A. Age in years
B. Smoking
C. Heart dz
D. Bp measures in mmhg

15. Which of the following variable assessed in Chi-square?


A. bp & smoking
B. heart dz & smoking
C. age ..

- The chi-square test is often used to compare the distributions of two or more groups
with categorical outcome variables. not continous variables.

16. Different numbers like ((75, 89, 82, 82 83, 85, 86, 86 , 90)) Most frequent value
represent?
A. mode
B. mean

- Find the mean, median, mode, and range for the following list of values:
- 13, 18, 13, 14, 13, 16, 14, 21, 13
- The mean is the usual average, so I'll add and then divide:(13 + 18 + 13 + 14 + 13 +
16 + 14 + 21 + 13) ÷ 9 = 15, Note that the mean, in this case, isn't a value from the
original list. This is a common result. You should not assume that your mean will be
one of your original numbers.
- The median is the middle value, so first I'll have to rewrite the list in numerical order
13, 13, 13, 13, 14, 14, 16, 18, 21. There are nine numbers in the list, so the middle
one will be the (9 + 1) ÷ 2 = 10 ÷ 2 = 5th number: 13, 13, 13, 13, 14, 14, 16, 18, 21,
So the median is 14.
- The mode is the number that is repeated more often than any other, so 13 is the
mode.
- The largest value in the list is 21, and the smallest is 13, so the range is 21 – 13 = 8.
- mean: 15

median: 14

mode: 13

range: 8

١١٥ ‫صفحة‬
Surgery Made Easy

17. Null hypothesis in comparing the morality between female and males in
association with ?
A. female mortality # male mortality
B. female mortality > male mortality
C. female mortality =male mortality
D. an association between the mortality and ...

18. 2 group One on placebo and another group on analgesia P value 0.002 what
dose it mean?
A. drug is effective
B. has evidence

- For typical analysis, using the standard α = 0.05 cutoff, the null hypothesis is rejected
when p < .05 and not rejected when p > .05. The p-value does not, in itself, support
reasoning about the probabilities of hypotheses but is only a tool for deciding whether to
reject the null hypothesis.

19. Pic of diagram and what it is?


A. Histogram

- See images
- other types http://www.statisticshowto.com/types-graphs/

20. Which pt of the following should be the 1st in taken to OR?


A. Acute appendicitis
B. Young male with incarcerated hernia
C. Old lady with strangulated hernia
D. pt with pyloric hypertrophy

21. 15 y/o female case of appendicitis, came with her mother, what u will do (for the
consent)?
A. This is emergency and no need for consent
B. You will wait for her father until he arrived.
C. Inform both of them and ask mother to sign on the consent

22. Why research subjects are vulnerable?


A. poor, cannot protect their interst

23. a new treatment for disease prevent death but doesn't cause recovery from the
disease. This will?
A. increase prevalence

24. a study conducted to evaluate medication among depressed patients they were
not taking the medication regularly :
A. Unacceptable b/c this affect drug efficacy
B. acceptable because the patient signed the consent voluntary

١١٦ ‫صفحة‬
Surgery Made Easy

25. Which of the following would be the main reason to consider research
participants vulnerable?
A. Poorer than other participants
B. Unable to protect their interests
C. Persons with emotional distress
D. Mentally disabled or handicapped

26. A study was conducted to determine if there was a difference in the prevalence
of diabetes mellitus among health care workers in Riyadh. The results showed
that 30 out of 100 doctors were diabetic as compared to 50 out of 200
nurses.Which is the most appropriate statistical test?
A. T-test
B. ANOVA
C. Chi-square
D. Correlation

- The chi-square test is often used to compare the distributions of two or more groups
with categorical outcome variables. not continous variables.

27. Someone from pharmacy company came to your clinic and explain to you about
their drug and it is already used in another countries?
A. Take samples for trail
B. Prescribe the drug since it was used in another countries
C. The company should provide the efficiency of the drug

28. A study was conducted to evaluate the benefits of an intensive exercise program
in reducing subsequent mortality among persons who survive after an initial
myocardial infarction. Patients were randomized to receive either usual care
(controls) or the exercise program. Among 100 controls, 50 died within the three
year follow-up period, compared with 30 deaths among the 100 patients on the
exercise program.What was the relative risk of death for the exercise group
compared to controls?
A. 0.30
B. 0.60
C. 1.67
D. 3.33

- Control 50/100 = 0.5 exercise 30/100= 0.3


- relative risk "RR" of exercise to control = 0.3/0.5= 0.6

29.After a procedure the nurse said that the count is incomplete and the doctor
checked but he was sure that no thing missed. After that x ray done show a guze
inside the abdomen What should the doctor do? 2018
A. Abberant error.
B. Abberant complications
C. Error
D. Complication

-“NEVER EVENTS” IN SURGERY


١١٧ ‫صفحة‬
Surgery Made Easy

- Never events are errors in medical care that are clearly identifiable, preventable, and
serious in their consequences for patients and that indicate a real problem in the
safety and credibility of a healthcare facility
- National Quality Forum surgical “never events” include retained surgical items,
wrong-site surgery, and death on the day of surgery of a normal healthy patient
(American Society of Anesthesiologists Class 1).

30.After a procedure the nurse said that the count is incomplete and the doctor
checked but he was sure that no thing missed, you cheeked many times but still
count is incomplete what you should do ? 2018
A. Xray
B. Open again
C. Call for recount
D. Report as counting error 


- An x-ray at the completion of an operation is encouraged if there is any concern for a


foreign body based on confusion regarding the counts by even a single member of
the OR team or in the presence of a risk factor. Schwartz

١١٨ ‫صفحة‬
Surgery Made Easy

Transplant
1. Four weeks after a cadaveric renal transplant, the recipient returns to the
emergency department with bilateral lower- extremity edema. In spite of normal
fluid intake, he reports that he has had minimal urine output over the past 18
hours. Serum creatinine is now elevated to 1.4 mg per deciliter from the
postoperative 1.0 mg per deciliter. After failure to respond to a fluid challenge, an
ultrasound is obtained. This reveals good perfusion, minimal hydronephrosis,
and a 3x4x6-cm hypoechoic mass adjacent to the renal pelvis of the allograft.
What is the most likely cause of the patient's oliguria?
A. Lymphocele formation
B. Ureteroneocystostomy stenosis
C. Renal artery thrombosis
D. Renal artery stenosis
E. Compressive hematoma

2. two weeks after a deceased donor kidneys transplant, the recipient return to the
ER with & fullness over transplanted kidneys minimal urine output over the past
18 hours in spite of normal fluids intake, serum creatinine is elevated, US
showed, lymphocele was the final diagnosis, what is your next step?
A. Percutanous drainage
B. Fenstration to abdomen
C. Scelerosing injection
D. Observation with US

- Lymphocele – most common cause of external ureter compression


- MC occurs 3 weeks after TXP (late decreased urine output with hydronephrosis and
fluid collection)
- Tx: 1st try percutaneous drainage; if that fails, then need peritoneal window(make
hole in peritoneum, lymphatic fluid drains into peritoneum and is re-absorbed – 95%
successful)

16.Renal stone with bone ache, what is the appropriate investigation ?


A. PTH

- Dx Tertiary Hyperparathyroidism :
(A) Causative Factor
1. Persistent hyperparathyroidism after renal transplantation
2. Parathyroid gland hyperplasia with autonomous PTH production
(B) Symptoms
1. Pathologic fractures, bone pain, renal stones, peptic ulcer disease, pancreatitis, and
mental status changes
2. In these patients, both the PTH and calcium levels are high.
(C) Treatment
1. Surgical treatment for symptomatic disease or autonomous PTH greater than 1 year
after transplant
2. Subtotal or total parathyroidectomy with autotransplantation and upper thymectomy.
Mont

١١٩ ‫صفحة‬
Surgery Made Easy

4. Transplant kidney preserved, with devitalization , The cause?


A. Cold preserving more than 6 hours
B. Warming heat more than 25 minutes
C. Using preserving solution.

5. Plan for renal transplant with adenocarcinoma polyp in rectum T( in situ ) POST
EXSION ?
A. Proceed with transplant
B. Postponed after 2 years
C. Postponed after 5 years.
D. Don’t do transplant.

- Untreated and/or active malignancies are absolute contraindications to a transplant


with two exceptions: nonmelanocytic skin cancer and incidental renal cell cancer
- wait of at least 2 years after successful treatment of low-grade tumors with a low
risk of recurrence (e.g., completely locally excised low-grade squamous cell cancer of
the skin, colon cancer in a polyp absent stalk invasion).
- wait of at least 5 years after successful treatment for certain types of tumors,
especially at advanced stages or those with a high risk of recurrence (e.g.,
melanoma, lymphoma, renal cell cancer, breast cancer, colon cancer).
- the recurrence rate is 67% in patients with multiple myeloma, 53% in nonmelanocytic
skin cancer, 29% in bladder cancer, and 23% in breast cancer.Schwartz

6. patient K/C of ESRD and breast cancer, she on treatment in term of


chemotherapy total mastectomy and radiation therapy one month backs for
breast cancer planed for renal transplant ? 2018
A. Proceed with transplant
B. Postponed after 2 years
C. Postponed after 5 years.
D. Don’t do transplant.

- read Q 5

7. Pt for renal transplant as HIV screening is part of PRE op assessment but Pt has
consent about it? 2018
A. Postponed PRE op workout
B. Sign consent before do it.
C. Do not do with sign refusal
D. Tell the authorities. 


8. Pt post renal transplant when is the time of peak for infections ? 2018
A. Immediately
B. after operation After 3-6 weeks
C. 4 to 6 months
D. 1 year

- Early infections (i.e., infections occurring within 1 month posttransplant) can be due to
a wide spectrum of pathogens (bacterial, viral, and fungal). In the immediate
postoperative period, recipients are significantly compromised from the stress of the
١٢٠ ‫صفحة‬
Surgery Made Easy

operation, from induction immunosuppression, and often from initially impaired graft
function. Infections during this period can be devastating. Schwartz

١٢١ ‫صفحة‬
Surgery Made Easy

Urology
1. postsigmoidectomy for diverticulitis developed gas in urinary bladder
(Rectovesical fistula ) what is the best diagnostic test ?
A. sigmoidescopy
B. cystoscopy
C. CT SCAN
D. barium enema

- Barium enemas (and colonoscopy/flexible sigmoidoscopy) are usually avoided in the


acute setting for fear of causing perforation or peritonitis. However, they are used after
an attack has settled to exclude a coexisting carcinoma and assess the extent of
diverticular disease. Colovesical fistulae should be evaluated with cystoscopy and
biopsy in addition. Contrast examinations or CT may demonstrate the fistula clearly.
- Approximately 5% of patients with complicated diverticulitis develop fistulas between the
colon and an adjacent organ. Colovesical fistulas are most common. Schwartz
- Colonoscopy or sigmoidoscopy usually is required to rule out malignancy.

2. A 17 years old male presented with acute scrotum diagnosed as testicular


torsion, intra op right hemi scrotum had been incised by urology, they found
gangrenous segment of small bowel protruded from the inguinal region they ask
your help, What is the best approach:
A. exp.lap with resection and repair
B. supra inguinal incision , resection and repair
C. trans scrotal incision and resection 


١٢٢ ‫صفحة‬
Surgery Made Easy

Vascular
1. Post lab Chole. c/o LL redness and tenderness (sign & symptoms of DVT) what
is the management?
A. anticoagulation
B. elevation of the leg
C. Antithrombotic

- Deep vein thrombosis (DVT) and pulmonary embolism (PE) are two manifestations
of venous thromboembolism (VTE).
- There are two classes of antithrombotic drugs: anticoagulants and antiplatelet
drugs.
- Anticoagulation is the mainstay of therapy for patients with acute lower extremity
deep vein thrombosis (DVT). uptodate

2. A 68, year old man was found to have (AAA) on computed tomography (CT)
and underwent an elective open AAA repair. His postoperative course was
unremarkable, stable in ICU. Physical examination is notable for an
increasingly distended abdomen. And the nurse notes that the patient had an
episode of large bloody bowel movement your action will be ?
A. Angio
B. sigmoidscopy

- Dx ischemic colitis. One of the complication post aortic aneurysm repair :


- Suspect ischemia if patient has bowel movement during the first 24–72 hours after
surgery.
- Mucosal ischemia most common and usually manifested by mucosal sloughing;
resolves spontaneously in most cases with bowel rest.
- Requires immediate flexible sigmoidoscopy (or abdominal CT) to diagnose—most
commonly affected location is the sigmoid colon
- Transmural involvement requires reexploration and colonic resection.
- Most common after emergent open AAA repair (25%–40% of patients)
- Angiography is usually not helpful because major arterial occlusion is rare.
- Colonoscopy should be performed after recovery to evaluate strictures and to rule
out other diagnoses such as inflammatory bowel disease or malignancy. Failure to
improve after 2 to 3 days of medical management, progression of symptoms, and
deterioration in clinical condition are indications for surgical exploration.


3. Patient with lymphedema in calf of Rt leg post XRT for inguinal LN, what is
management?
A. Compression socks
B. Below knee amputation
C. Microvascular lymphatic anastomosis

4. Pt for sigmoid resection (OR pt for LAR?? ) What is the best prophylaxis for
DVT?
A. Early ambulation
B. Elastic stocking
١٢٣ ‫صفحة‬
Surgery Made Easy

C. Pneumatic decompression
D. UFH

- First, Determine the Patient’s Risk Factors


- Low risk—age less than 40 years; ambulatory or minor surgery
- Moderate risk—age greater than 40 years; abdominal, pelvic, or thoracic surgery
- High risk—age greater than 40 years; prior DVT or PE, malignancy, hip and other
orthopedic surgeries, immobility, hypercoagulable states
- Then, Prophylaxis of Choice
- Encourage early ambulation in all patients, and get physical/occupational therapy
involved early, if indicated.
- CHEST guidelines for prevention of VTE in nonorthopedic surgical patients:
- Low risk for VTE (∼1.5%)—suggest mechanical prophylaxis, preferably with
intermittent pneumatic compression (IPC).
- Moderate risk for VTE (∼3%)—suggest LMWH, low-dose unfractionated heparin,
or mechanical prophylaxis with IPC over no prophylaxis.
- High risk for VTE (∼6%)—recommend pharmacologic prophylaxis with LMWH or
low-dose unfractionated heparin over no prophylaxis. In these patients, also
recommend adding mechanical prophylaxis with IPC to pharmacologic
prophylaxis.
- Prophylaxis should be started before the initiation of anesthesia; low-dose SC
unfractionated heparin is usually given in a dose of 5000 units 2 hours preoperatively
and then every 8–12 hours postoperatively.
- High-risk patients should be watched closely for clinical signs and symptoms of DVT.
Duplex scanning is the least invasive method for screening.
- Table 24-3 Thromboembolism risk and recommended thromboprophylaxis in surgical
patients. Schwartz P919

5. A 45 years old women presented with a non healing ulcer at the medial malleolus
associated with leg edema & hyperpigmintation, management consistent of?
A. Wet to dry dressing
B. Split thickness skin graft
C. Superficial perforator ligation
D. Compression dressing

- Dx Chronic venous insufficiency (CVI) affects an estimated 600,000 people in the


United States. 90 Patients complain of leg fatigue, discomfort, and heaviness. Signs
of CVI may include varicose veins, pigmentation, lipodermatosclerosis, and venous
ulceration.
- symptom:
- Venous claudication—pain and swelling experienced while walking, which is relieved
by rest
- Physical examination:
- Skin changes—edema, hyperpigmentation, stasis dermatitis, eczema, telangiectasias
- Venous ulcers—Medial malleolus is the classic location.
- Imaging:
- Venous duplex scanning, Reflux lasting more than 0.5 seconds after cuff deflation
indicates pathologic reflux.
- Nonoperative Treatment of Chronic Venous Insufficiency:

١٢٤ ‫صفحة‬
Surgery Made Easy

- Compression Therapy. Compression therapy is most commonly achieved with


graduated elastic compression stockings.
- Surgical/Interventional Treatment of Chronic Venous Insufficiency:
- Perforator Vein Ligation.
- Superficial Venous Surgery.
- Deep Venous Valvular Reconstruction.
- Venous Stenting.

6. Obese female 120 kg with past history of DVT booked for elective surgery the
best DVT prophylactic is ?
A. LMWH
B. LMWH with intermittent pneumatic compression
C. IPCD
D. Heparin with warfarin

- See Q 4 above

. ‫ دعواتكم‬.. ‫الحمد اهلل‬

١٢٥ ‫صفحة‬

You might also like